[obm-l] Re: [obm-l] Dúvida

2021-11-16 Por tôpico Ian Barquette
Boa tarde!

Esse tipo de equação que você mandou se chama "Equações de Pell". É uma
equação diofantina, mas da forma x^2 - dy^2 = 1, em que d é um número
positivo e não-quadrado-perfeito. Também busca soluções inteiras para "x" e
"y".

Um matemático provou que esse tipo de equação tem infinitas soluções quando
d segue as restrições.

Alguns motivos dos passos da resolução tem origens em matemática mais
avançada do que sei, por isso vou os omitir.

Resolução:
A equação pode ser reescrita como (x+y*sqrt(d))*(x-y*sqrt(d)) = 1, e
podemos achar as n soluções em função do que é chamado de solução
fundamental ("1ª" solução):

x_n + y_n * sqrt(d) = (x_1 + y_1 * sqrt(d))^n
  ("x_n" significa "x índice n")

Para encontrar a solução fundamental, podemos utilizar das frações
contínuas para sqrt(d), encurtando a fração no final do período, e, assim,
o numerador e o denominador do resultado da fração vão ser o "x" e "y",
respectivamente (utilizando d = 7):

A fração contínua para sqrt(7) é [2;*1,1,1,4*], com repetição na parte em
negrito. (Vou fazer a representação linear, pois é mais complicado entender
na forma de fração)
Encurtando-a para o final do período, fica [2;1,1,1], o que equivale a 8/3.

Portanto, a solução fundamental é (8, 3).

A partir disso, pode-se encontrar os outros infinitos resultados pela
fórmula já mencionada acima:
(x_1 + y_1 * sqrt(d))^n
(8 + 3 * sqrt(7))^n

Note que para alcançar a solução (x, y) é necessário "desconsiderar" a raiz
quadrada de "d" no final, e já que se quer x e y inteiros, a solução
será (±x, ±y)

Exemplos:
(8+3 * sqrt(7))^2 = 64 + 2(24*sqrt(7)) + 63 = 127 + 48*sqrt(7)
Solução2: (±127, ±48)

(8+3 * sqrt(7))^3 = (8 + 3*sqrt(7)) * (127 + 48*sqrt(7)) = 8*127 +
8*48*sqrt(7) + 3*sqrt(7)*127 + 3*sqrt(7)*48*sqrt(7) = 1016 + 384*sqrt(7) +
381*sqrt(7) + 1008 = 2024 + 765*sqrt(7)
Solução3: (±2024, ±765)

Desculpe pelas partes sem muita explicação, mas espero que tenha entendido
como se resolve


Em seg., 15 de nov. de 2021 às 13:36, Pedro José 
escreveu:

> Boa tarde!
>
> Alguém saberia como resolver a seguinte equação:
>
> x^2-7y^2=1, x,y em Z?
>
> Fiz a-7b=1 e achei a= 8 +7k e b=1 +K
> Logo fica fácil que para k=-1 funciona x^2=1 e y^2=0.
> Também funciona para k=8 x^2=64 e y^2=9.
> Mas não sei nem como achar mais soluções nem como provar que só são essas.
> Alguém poderia me dar uma orientação?
>
> Cordialmente,
> PJMS
>
> --
> Esta mensagem foi verificada pelo sistema de antivírus e
> acredita-se estar livre de perigo.

-- 
Esta mensagem foi verificada pelo sistema de antiv�rus e
 acredita-se estar livre de perigo.



Re: [obm-l]

2021-11-14 Por tôpico Claudio Buffara
Por que vc não testa?

On Sun, Nov 14, 2021 at 9:53 AM Eric Campos Bastos Guedes <
ebastosgue...@gmail.com> wrote:

> Eu preciso de um retorno sobre o meu algoritmo que quebra o RSA resolvendo
> o problema de fatoracao.
>
> O Passo 8 talvez possa ser substituido por: PASSO 8': P = P(P+C)
>
> Em seg., 6 de set. de 2021 07:47, Eric Campos Bastos Guedes <
> ebastosgue...@gmail.com> escreveu:
>
>> Aparentemente minha conexão está raqueada por gente do Bolsonaro e eu não
>> estou recebendo respostas para minha postagem e também não estou
>> conseguindo acessar os sites de discussão sobre o RSA. Há pessoas se
>> fazendo passar por mim também.
>>
>> -- Forwarded message -
>> De: Eric Campos Bastos Guedes 
>> Date: sáb., 4 de set. de 2021 00:33
>> Subject: [obm-l]
>> To: 
>>
>>
>> Olá a todos. Gostaria de pedir licença para que vocês avaliem um
>> algoritmo que eu fiz para fatorar números grandes com fatores primos também
>> grandes. Eu acredito que esse algoritmo quebre o RSA
>>
>> O algoritmo é o seguinte:
>>
>> PASSO 1: faça A=3
>> PASSO 2: N é o inteiro a ser fatorado
>> PASSO 3: M = N**16 (potência)
>> PASSO 4: faça A=A+1
>> PASSO 5: faça P=A
>> PASSO 6: faça B=número aleatório entre 0 e 1
>> PASSO 7: se B eh maior que 0.5 faça C=1 senão faça C = -1
>> PASSO 8: faça P=(PP+CP)/2=P(P+C)/2
>> PASSO 9: se P eh menor que M  vá para o PASSO 6
>> PASSO 10: se mdc(P, N) = 1 faça M=MM e vá para o PASSO 4
>> PASSO 11: se mdc(P, N) = N faça M = raiz quadrada de M e vá para o PASSO 4
>> PASSO 12: mdc(P, N) é fator de N
>> FIM
>>
>> AUTOR: ERIC CAMPOS BASTOS GUEDES  (DIA 4 DE SETEMBRO)
>>
>> Creio ter resolvido o problema de fatoração. Alguém pode verificar isso
>> para mim.
>>
>> --
>> Esta mensagem foi verificada pelo sistema de antivírus e
>> acredita-se estar livre de perigo.
>>
>
> --
> Esta mensagem foi verificada pelo sistema de antivírus e
> acredita-se estar livre de perigo.

-- 
Esta mensagem foi verificada pelo sistema de antiv�rus e
 acredita-se estar livre de perigo.



Re: [obm-l]

2021-11-14 Por tôpico Eric Campos Bastos Guedes
Eu preciso de um retorno sobre o meu algoritmo que quebra o RSA resolvendo
o problema de fatoracao.

O Passo 8 talvez possa ser substituido por: PASSO 8': P = P(P+C)

Em seg., 6 de set. de 2021 07:47, Eric Campos Bastos Guedes <
ebastosgue...@gmail.com> escreveu:

> Aparentemente minha conexão está raqueada por gente do Bolsonaro e eu não
> estou recebendo respostas para minha postagem e também não estou
> conseguindo acessar os sites de discussão sobre o RSA. Há pessoas se
> fazendo passar por mim também.
>
> -- Forwarded message -
> De: Eric Campos Bastos Guedes 
> Date: sáb., 4 de set. de 2021 00:33
> Subject: [obm-l]
> To: 
>
>
> Olá a todos. Gostaria de pedir licença para que vocês avaliem um algoritmo
> que eu fiz para fatorar números grandes com fatores primos também grandes.
> Eu acredito que esse algoritmo quebre o RSA
>
> O algoritmo é o seguinte:
>
> PASSO 1: faça A=3
> PASSO 2: N é o inteiro a ser fatorado
> PASSO 3: M = N**16 (potência)
> PASSO 4: faça A=A+1
> PASSO 5: faça P=A
> PASSO 6: faça B=número aleatório entre 0 e 1
> PASSO 7: se B eh maior que 0.5 faça C=1 senão faça C = -1
> PASSO 8: faça P=(PP+CP)/2=P(P+C)/2
> PASSO 9: se P eh menor que M  vá para o PASSO 6
> PASSO 10: se mdc(P, N) = 1 faça M=MM e vá para o PASSO 4
> PASSO 11: se mdc(P, N) = N faça M = raiz quadrada de M e vá para o PASSO 4
> PASSO 12: mdc(P, N) é fator de N
> FIM
>
> AUTOR: ERIC CAMPOS BASTOS GUEDES  (DIA 4 DE SETEMBRO)
>
> Creio ter resolvido o problema de fatoração. Alguém pode verificar isso
> para mim.
>
> --
> Esta mensagem foi verificada pelo sistema de antivírus e
> acredita-se estar livre de perigo.
>

-- 
Esta mensagem foi verificada pelo sistema de antiv�rus e
 acredita-se estar livre de perigo.



Re: [obm-l] OBMEP 2021 - Fase 2 - N3

2021-11-11 Por tôpico Matheus Bezerra Luna
Legal esse raciocínio, simplifica bastante.
Na prova não consegui explicar bem a minha solução por falta de tempo, mas
fiz algo mais ou menos assim:
Se no tempo T+1 o ponteiro estiver em uma coroa e a moeda antes do ponteiro
for cara, no tempo T o ponteiro estava em uma cara e a moeda seguinte era
coroa (determinado). Se no T+1 o ponteiro está numa coroa e a moeda
anterior é também coroa, no T o ponteiro estava em coroa e a seguinte era
cara (determinado). Assim, as posições em que o ponteiro está numa coroa
são reversíveis.
Basta provar que alguma posição com ponteiro em coroa se repete. Suponha,
por absurdo, que não, e seja S' a primeira posição repetida referente a uma
configuração S, que tem ponteiro em cara. Por hipótese, as posições após S'
só podem ter o ponteiro em caras, pois as posições seguintes a S' são
equivalentes às seguintes a S e então são repetições, não podendo ter
ponteiro em coroa. Porém, isso equivale a dizer vai chegar um momento em
que todas as moedas serão caras, já que quando o ponteiro está em cara as
moedas se preservam como estão e o ponteiro a partir de um ponto sempre
está em caras. Absurdo pelo item c da questão, o que finaliza o raciocínio.
*Matheus BL*


On Tue, Nov 9, 2021 at 6:46 PM Ralph Costa Teixeira 
wrote:

> Oi, Matheus.
>
> Concordo, olhando apenas as moedas sob o ponteiro, não dá para reverter
> mas olhando as vizinhas, ou seja olhando TODO o sistema, TODAS AS MOEDAS a
> todo o tempo, dá sim!
>
> Mais exatamente, posso denotar o estado do sistema assim:
>
> ABC(D*)EFGHIJ
>
> onde cada A, B, C, ... assumem o valor "Cara=0" ou "Coroa=1", e o * marca
> onde o ponteiro aponta nesse momento. Ou seja, nesta notação começaria com:
>
> Tempo 0: (1*)1
> Tempo 1: 1(1*)0111
> Tempo 2: 11(0*)111
> Tempo 3: 110(1*)01
> Tempo 4: 1101(0*)1
> Tempo 5: 11010(1*)0111
> ...
>
> Pois bem, se no tempo (n+1) for, digamos
> ABC(D*)EFGHIJ
> entao no tempo n tinha que ser...
> AB(C*)DXFGHIJ
> onde a unica moeda que eu tenho que descobrir eh X (as outras não mudam de
> n para n+1). Mas eu descubro X olhando para **D e E juntas** (nao apenas
> uma delas)!
>
> Abraço, Ralph.
>
>
>
> On Tue, Nov 9, 2021 at 3:24 PM Matheus Bezerra Luna <
> matheusbezerr...@gmail.com> wrote:
>
>> Não é completamente reversível não, vai ter que usar o item C para
>> concluir o D. Se num tempo T o ponteiro está em uma cara, no tempo T-1 ele
>> poderia estar tanto numa cara (pois então nesse tempo não aconteceu nada e
>> a moeda seguinte permanceu cara) ou então coroa (o ponteiro em uma coroa
>> sendo a moeda seguinte também coroa)
>>
>> On Tue, Nov 9, 2021, 13:47 Pedro Júnior 
>> wrote:
>>
>>> Obrigado, Ralph!
>>>
>>> Em ter., 9 de nov. de 2021 às 13:21, Ralph Costa Teixeira <
>>> ralp...@gmail.com> escreveu:
>>>
 Suponho que (A) e (B) sejam fáceis -- basta seguir o algoritmo na mão e
 ver o que acontece.

 Para facilitar a conversa, vou pensar em "tempo" como o número de
 movimentos feitos... Ou seja, o tempo 0 corresponde à posição inicial; o
 tempo 1 seria logo após o primeiro movimento; etc.

 Para (C), pense assim: se o sistema tem alguma coroa no tempo (n), eu
 afirmo que vai ter alguma coroa no tempo (n+1). De fato:
 -- Se o ponteiro aponta para uma cara no tempo (n), o sistema não muda,
 e a tal coroa continua ali;
 -- Se o ponteiro aponta para uma coroa no tempo (n), ESTA coroa vai
 ficar presente no tempo (n+1).
 Portanto, sempre teremos coroas.
 (Talvez seja mais natural pensar assim: como que o sistema passaria de
 "ter coroas" para "não ter coroas"? Bom, para ele mudar o ponteiro tem que
 apontar para alguma coroa, e esta coroa NÃO MUDA. Ou seja,
 impossível passar de "ter coroas" para "não ter coroas".)

 Para (D), note que o sistema tem apenas (2^10) * 10 configurações
 possíveis (o número não interessa tanto, o que importa é que é FINITO; note
 que incluo ali as posições das moedas E a do ponteiro), enquanto o tempo
 avança sempre, então em algum momento alguma configuração vai ter que
 repetir.
 Mas pense como "desfazer" o último movimento realizado e você vai
 perceber que existe apenas um jeito de "voltar no tempo" (deixo para você
 descrever exatamente isso)! Ou seja, o sistema é reversível (olhando como
 ficou o sistema no tempo (n+1), você consegue deduzir como ele estava no
 tempo (n), revertendo o último movimento, de maneira única). Portanto, se o
 sistema tinha a mesma configuração nos tempos A e A+T, revertendo os
 movimentos, concluímos que vai ter a mesma configuração nos tempos 0 e T;
 ou seja, no tempo T tínhamos todas coroas como no tempo 0 (e o ponteiro
 apontando para A! Bônus!)

 Abraço, Ralph.

 On Tue, Nov 9, 2021 at 12:22 PM Pedro Júnior <
 pedromatematic...@gmail.com> wrote:

> Olá pessoal, alguém aí conseguiu fazer essa questão da prova da OBMEP
> 2021 N3, fase 2? Se puder, ajuda aí... 

Re: [obm-l] OBMEP 2021 - Fase 2 - N3

2021-11-09 Por tôpico Ralph Costa Teixeira
Oi, Matheus.

Concordo, olhando apenas as moedas sob o ponteiro, não dá para reverter mas
olhando as vizinhas, ou seja olhando TODO o sistema, TODAS AS MOEDAS a todo
o tempo, dá sim!

Mais exatamente, posso denotar o estado do sistema assim:

ABC(D*)EFGHIJ

onde cada A, B, C, ... assumem o valor "Cara=0" ou "Coroa=1", e o * marca
onde o ponteiro aponta nesse momento. Ou seja, nesta notação começaria com:

Tempo 0: (1*)1
Tempo 1: 1(1*)0111
Tempo 2: 11(0*)111
Tempo 3: 110(1*)01
Tempo 4: 1101(0*)1
Tempo 5: 11010(1*)0111
...

Pois bem, se no tempo (n+1) for, digamos
ABC(D*)EFGHIJ
entao no tempo n tinha que ser...
AB(C*)DXFGHIJ
onde a unica moeda que eu tenho que descobrir eh X (as outras não mudam de
n para n+1). Mas eu descubro X olhando para **D e E juntas** (nao apenas
uma delas)!

Abraço, Ralph.



On Tue, Nov 9, 2021 at 3:24 PM Matheus Bezerra Luna <
matheusbezerr...@gmail.com> wrote:

> Não é completamente reversível não, vai ter que usar o item C para
> concluir o D. Se num tempo T o ponteiro está em uma cara, no tempo T-1 ele
> poderia estar tanto numa cara (pois então nesse tempo não aconteceu nada e
> a moeda seguinte permanceu cara) ou então coroa (o ponteiro em uma coroa
> sendo a moeda seguinte também coroa)
>
> On Tue, Nov 9, 2021, 13:47 Pedro Júnior 
> wrote:
>
>> Obrigado, Ralph!
>>
>> Em ter., 9 de nov. de 2021 às 13:21, Ralph Costa Teixeira <
>> ralp...@gmail.com> escreveu:
>>
>>> Suponho que (A) e (B) sejam fáceis -- basta seguir o algoritmo na mão e
>>> ver o que acontece.
>>>
>>> Para facilitar a conversa, vou pensar em "tempo" como o número de
>>> movimentos feitos... Ou seja, o tempo 0 corresponde à posição inicial; o
>>> tempo 1 seria logo após o primeiro movimento; etc.
>>>
>>> Para (C), pense assim: se o sistema tem alguma coroa no tempo (n), eu
>>> afirmo que vai ter alguma coroa no tempo (n+1). De fato:
>>> -- Se o ponteiro aponta para uma cara no tempo (n), o sistema não muda,
>>> e a tal coroa continua ali;
>>> -- Se o ponteiro aponta para uma coroa no tempo (n), ESTA coroa vai
>>> ficar presente no tempo (n+1).
>>> Portanto, sempre teremos coroas.
>>> (Talvez seja mais natural pensar assim: como que o sistema passaria de
>>> "ter coroas" para "não ter coroas"? Bom, para ele mudar o ponteiro tem que
>>> apontar para alguma coroa, e esta coroa NÃO MUDA. Ou seja,
>>> impossível passar de "ter coroas" para "não ter coroas".)
>>>
>>> Para (D), note que o sistema tem apenas (2^10) * 10 configurações
>>> possíveis (o número não interessa tanto, o que importa é que é FINITO; note
>>> que incluo ali as posições das moedas E a do ponteiro), enquanto o tempo
>>> avança sempre, então em algum momento alguma configuração vai ter que
>>> repetir.
>>> Mas pense como "desfazer" o último movimento realizado e você vai
>>> perceber que existe apenas um jeito de "voltar no tempo" (deixo para você
>>> descrever exatamente isso)! Ou seja, o sistema é reversível (olhando como
>>> ficou o sistema no tempo (n+1), você consegue deduzir como ele estava no
>>> tempo (n), revertendo o último movimento, de maneira única). Portanto, se o
>>> sistema tinha a mesma configuração nos tempos A e A+T, revertendo os
>>> movimentos, concluímos que vai ter a mesma configuração nos tempos 0 e T;
>>> ou seja, no tempo T tínhamos todas coroas como no tempo 0 (e o ponteiro
>>> apontando para A! Bônus!)
>>>
>>> Abraço, Ralph.
>>>
>>> On Tue, Nov 9, 2021 at 12:22 PM Pedro Júnior <
>>> pedromatematic...@gmail.com> wrote:
>>>
 Olá pessoal, alguém aí conseguiu fazer essa questão da prova da OBMEP
 2021 N3, fase 2? Se puder, ajuda aí... Valeu!

 6) há 10 moedas em um círculo nomeadas de A a J, inicialmente todas com
 a face coroa virada para cima. No centro desse círculo, há um ponteiro que
 inicialmente aponta para a moeda A. Esse ponteiro se movimenta, girando no
 sentido anti-horário (A->B->C->...->J->A->...). Ao movimentar-se, há duas
 opções:
 •Quando o ponteiro termina o movimento apontando para uma moeda com a
 face coroa virada para cima, a moeda seguinte é virada.
 •Quando o ponteiro termina o movimento apontando para uma moeda com a
 face cara virada para cima, nada acontece.

 Há exemplo, no primeiro movimento (de A para B), o ponteiro termina em
 B, e assim, vira-se a moeda C, que fica com a face cara para cima.

 Letra A) o que acontece com as moedas C e D após o segundo movimento?

 Letra B) Depois do 12º movimento, quais moedas estão com a face coroa
 virada para cima?

 Letra C) mostre que é impossível que, após certo número de movimentos,
 todas as moedas fiquem com a face cara para cima.

 Letra D) Mostre que, após um certo número de movimentos, todas as
 moedas voltarão a ficar com a face coroa para cima.



 --
 Esta mensagem foi verificada pelo sistema de antivírus e
 acredita-se estar livre de perigo.
>>>
>>>
>>> --
>>> Esta mensagem foi verificada pelo 

Re: [obm-l] OBMEP 2021 - Fase 2 - N3

2021-11-09 Por tôpico Matheus Bezerra Luna
Não é completamente reversível não, vai ter que usar o item C para concluir
o D. Se num tempo T o ponteiro está em uma cara, no tempo T-1 ele poderia
estar tanto numa cara (pois então nesse tempo não aconteceu nada e a moeda
seguinte permanceu cara) ou então coroa (o ponteiro em uma coroa sendo a
moeda seguinte também coroa)

On Tue, Nov 9, 2021, 13:47 Pedro Júnior  wrote:

> Obrigado, Ralph!
>
> Em ter., 9 de nov. de 2021 às 13:21, Ralph Costa Teixeira <
> ralp...@gmail.com> escreveu:
>
>> Suponho que (A) e (B) sejam fáceis -- basta seguir o algoritmo na mão e
>> ver o que acontece.
>>
>> Para facilitar a conversa, vou pensar em "tempo" como o número de
>> movimentos feitos... Ou seja, o tempo 0 corresponde à posição inicial; o
>> tempo 1 seria logo após o primeiro movimento; etc.
>>
>> Para (C), pense assim: se o sistema tem alguma coroa no tempo (n), eu
>> afirmo que vai ter alguma coroa no tempo (n+1). De fato:
>> -- Se o ponteiro aponta para uma cara no tempo (n), o sistema não muda, e
>> a tal coroa continua ali;
>> -- Se o ponteiro aponta para uma coroa no tempo (n), ESTA coroa vai ficar
>> presente no tempo (n+1).
>> Portanto, sempre teremos coroas.
>> (Talvez seja mais natural pensar assim: como que o sistema passaria de
>> "ter coroas" para "não ter coroas"? Bom, para ele mudar o ponteiro tem que
>> apontar para alguma coroa, e esta coroa NÃO MUDA. Ou seja,
>> impossível passar de "ter coroas" para "não ter coroas".)
>>
>> Para (D), note que o sistema tem apenas (2^10) * 10 configurações
>> possíveis (o número não interessa tanto, o que importa é que é FINITO; note
>> que incluo ali as posições das moedas E a do ponteiro), enquanto o tempo
>> avança sempre, então em algum momento alguma configuração vai ter que
>> repetir.
>> Mas pense como "desfazer" o último movimento realizado e você vai
>> perceber que existe apenas um jeito de "voltar no tempo" (deixo para você
>> descrever exatamente isso)! Ou seja, o sistema é reversível (olhando como
>> ficou o sistema no tempo (n+1), você consegue deduzir como ele estava no
>> tempo (n), revertendo o último movimento, de maneira única). Portanto, se o
>> sistema tinha a mesma configuração nos tempos A e A+T, revertendo os
>> movimentos, concluímos que vai ter a mesma configuração nos tempos 0 e T;
>> ou seja, no tempo T tínhamos todas coroas como no tempo 0 (e o ponteiro
>> apontando para A! Bônus!)
>>
>> Abraço, Ralph.
>>
>> On Tue, Nov 9, 2021 at 12:22 PM Pedro Júnior 
>> wrote:
>>
>>> Olá pessoal, alguém aí conseguiu fazer essa questão da prova da OBMEP
>>> 2021 N3, fase 2? Se puder, ajuda aí... Valeu!
>>>
>>> 6) há 10 moedas em um círculo nomeadas de A a J, inicialmente todas com
>>> a face coroa virada para cima. No centro desse círculo, há um ponteiro que
>>> inicialmente aponta para a moeda A. Esse ponteiro se movimenta, girando no
>>> sentido anti-horário (A->B->C->...->J->A->...). Ao movimentar-se, há duas
>>> opções:
>>> •Quando o ponteiro termina o movimento apontando para uma moeda com a
>>> face coroa virada para cima, a moeda seguinte é virada.
>>> •Quando o ponteiro termina o movimento apontando para uma moeda com a
>>> face cara virada para cima, nada acontece.
>>>
>>> Há exemplo, no primeiro movimento (de A para B), o ponteiro termina em
>>> B, e assim, vira-se a moeda C, que fica com a face cara para cima.
>>>
>>> Letra A) o que acontece com as moedas C e D após o segundo movimento?
>>>
>>> Letra B) Depois do 12º movimento, quais moedas estão com a face coroa
>>> virada para cima?
>>>
>>> Letra C) mostre que é impossível que, após certo número de movimentos,
>>> todas as moedas fiquem com a face cara para cima.
>>>
>>> Letra D) Mostre que, após um certo número de movimentos, todas as moedas
>>> voltarão a ficar com a face coroa para cima.
>>>
>>>
>>>
>>> --
>>> Esta mensagem foi verificada pelo sistema de antivírus e
>>> acredita-se estar livre de perigo.
>>
>>
>> --
>> Esta mensagem foi verificada pelo sistema de antivírus e
>> acredita-se estar livre de perigo.
>
>
>
> --
>
> Pedro Jerônimo S. de O. Júnior
>
> Professor de Matemática
>
> Geo João Pessoa – PB
>
> --
> Esta mensagem foi verificada pelo sistema de antivírus e
> acredita-se estar livre de perigo.

-- 
Esta mensagem foi verificada pelo sistema de antiv�rus e
 acredita-se estar livre de perigo.



Re: [obm-l] OBMEP 2021 - Fase 2 - N3

2021-11-09 Por tôpico Pedro Júnior
Obrigado, Ralph!

Em ter., 9 de nov. de 2021 às 13:21, Ralph Costa Teixeira 
escreveu:

> Suponho que (A) e (B) sejam fáceis -- basta seguir o algoritmo na mão e
> ver o que acontece.
>
> Para facilitar a conversa, vou pensar em "tempo" como o número de
> movimentos feitos... Ou seja, o tempo 0 corresponde à posição inicial; o
> tempo 1 seria logo após o primeiro movimento; etc.
>
> Para (C), pense assim: se o sistema tem alguma coroa no tempo (n), eu
> afirmo que vai ter alguma coroa no tempo (n+1). De fato:
> -- Se o ponteiro aponta para uma cara no tempo (n), o sistema não muda, e
> a tal coroa continua ali;
> -- Se o ponteiro aponta para uma coroa no tempo (n), ESTA coroa vai ficar
> presente no tempo (n+1).
> Portanto, sempre teremos coroas.
> (Talvez seja mais natural pensar assim: como que o sistema passaria de
> "ter coroas" para "não ter coroas"? Bom, para ele mudar o ponteiro tem que
> apontar para alguma coroa, e esta coroa NÃO MUDA. Ou seja,
> impossível passar de "ter coroas" para "não ter coroas".)
>
> Para (D), note que o sistema tem apenas (2^10) * 10 configurações
> possíveis (o número não interessa tanto, o que importa é que é FINITO; note
> que incluo ali as posições das moedas E a do ponteiro), enquanto o tempo
> avança sempre, então em algum momento alguma configuração vai ter que
> repetir.
> Mas pense como "desfazer" o último movimento realizado e você vai perceber
> que existe apenas um jeito de "voltar no tempo" (deixo para você descrever
> exatamente isso)! Ou seja, o sistema é reversível (olhando como ficou o
> sistema no tempo (n+1), você consegue deduzir como ele estava no tempo (n),
> revertendo o último movimento, de maneira única). Portanto, se o sistema
> tinha a mesma configuração nos tempos A e A+T, revertendo os movimentos,
> concluímos que vai ter a mesma configuração nos tempos 0 e T; ou seja, no
> tempo T tínhamos todas coroas como no tempo 0 (e o ponteiro apontando para
> A! Bônus!)
>
> Abraço, Ralph.
>
> On Tue, Nov 9, 2021 at 12:22 PM Pedro Júnior 
> wrote:
>
>> Olá pessoal, alguém aí conseguiu fazer essa questão da prova da OBMEP
>> 2021 N3, fase 2? Se puder, ajuda aí... Valeu!
>>
>> 6) há 10 moedas em um círculo nomeadas de A a J, inicialmente todas com a
>> face coroa virada para cima. No centro desse círculo, há um ponteiro que
>> inicialmente aponta para a moeda A. Esse ponteiro se movimenta, girando no
>> sentido anti-horário (A->B->C->...->J->A->...). Ao movimentar-se, há duas
>> opções:
>> •Quando o ponteiro termina o movimento apontando para uma moeda com a
>> face coroa virada para cima, a moeda seguinte é virada.
>> •Quando o ponteiro termina o movimento apontando para uma moeda com a
>> face cara virada para cima, nada acontece.
>>
>> Há exemplo, no primeiro movimento (de A para B), o ponteiro termina em B,
>> e assim, vira-se a moeda C, que fica com a face cara para cima.
>>
>> Letra A) o que acontece com as moedas C e D após o segundo movimento?
>>
>> Letra B) Depois do 12º movimento, quais moedas estão com a face coroa
>> virada para cima?
>>
>> Letra C) mostre que é impossível que, após certo número de movimentos,
>> todas as moedas fiquem com a face cara para cima.
>>
>> Letra D) Mostre que, após um certo número de movimentos, todas as moedas
>> voltarão a ficar com a face coroa para cima.
>>
>>
>>
>> --
>> Esta mensagem foi verificada pelo sistema de antivírus e
>> acredita-se estar livre de perigo.
>
>
> --
> Esta mensagem foi verificada pelo sistema de antivírus e
> acredita-se estar livre de perigo.



-- 

Pedro Jerônimo S. de O. Júnior

Professor de Matemática

Geo João Pessoa – PB

-- 
Esta mensagem foi verificada pelo sistema de antiv�rus e
 acredita-se estar livre de perigo.



Re: [obm-l] OBMEP 2021 - Fase 2 - N3

2021-11-09 Por tôpico Ralph Costa Teixeira
Suponho que (A) e (B) sejam fáceis -- basta seguir o algoritmo na mão e ver
o que acontece.

Para facilitar a conversa, vou pensar em "tempo" como o número de
movimentos feitos... Ou seja, o tempo 0 corresponde à posição inicial; o
tempo 1 seria logo após o primeiro movimento; etc.

Para (C), pense assim: se o sistema tem alguma coroa no tempo (n), eu
afirmo que vai ter alguma coroa no tempo (n+1). De fato:
-- Se o ponteiro aponta para uma cara no tempo (n), o sistema não muda, e a
tal coroa continua ali;
-- Se o ponteiro aponta para uma coroa no tempo (n), ESTA coroa vai ficar
presente no tempo (n+1).
Portanto, sempre teremos coroas.
(Talvez seja mais natural pensar assim: como que o sistema passaria de "ter
coroas" para "não ter coroas"? Bom, para ele mudar o ponteiro tem que
apontar para alguma coroa, e esta coroa NÃO MUDA. Ou seja,
impossível passar de "ter coroas" para "não ter coroas".)

Para (D), note que o sistema tem apenas (2^10) * 10 configurações possíveis
(o número não interessa tanto, o que importa é que é FINITO; note que
incluo ali as posições das moedas E a do ponteiro), enquanto o tempo avança
sempre, então em algum momento alguma configuração vai ter que repetir.
Mas pense como "desfazer" o último movimento realizado e você vai perceber
que existe apenas um jeito de "voltar no tempo" (deixo para você descrever
exatamente isso)! Ou seja, o sistema é reversível (olhando como ficou o
sistema no tempo (n+1), você consegue deduzir como ele estava no tempo (n),
revertendo o último movimento, de maneira única). Portanto, se o sistema
tinha a mesma configuração nos tempos A e A+T, revertendo os movimentos,
concluímos que vai ter a mesma configuração nos tempos 0 e T; ou seja, no
tempo T tínhamos todas coroas como no tempo 0 (e o ponteiro apontando para
A! Bônus!)

Abraço, Ralph.

On Tue, Nov 9, 2021 at 12:22 PM Pedro Júnior 
wrote:

> Olá pessoal, alguém aí conseguiu fazer essa questão da prova da OBMEP 2021
> N3, fase 2? Se puder, ajuda aí... Valeu!
>
> 6) há 10 moedas em um círculo nomeadas de A a J, inicialmente todas com a
> face coroa virada para cima. No centro desse círculo, há um ponteiro que
> inicialmente aponta para a moeda A. Esse ponteiro se movimenta, girando no
> sentido anti-horário (A->B->C->...->J->A->...). Ao movimentar-se, há duas
> opções:
> •Quando o ponteiro termina o movimento apontando para uma moeda com a face
> coroa virada para cima, a moeda seguinte é virada.
> •Quando o ponteiro termina o movimento apontando para uma moeda com a face
> cara virada para cima, nada acontece.
>
> Há exemplo, no primeiro movimento (de A para B), o ponteiro termina em B,
> e assim, vira-se a moeda C, que fica com a face cara para cima.
>
> Letra A) o que acontece com as moedas C e D após o segundo movimento?
>
> Letra B) Depois do 12º movimento, quais moedas estão com a face coroa
> virada para cima?
>
> Letra C) mostre que é impossível que, após certo número de movimentos,
> todas as moedas fiquem com a face cara para cima.
>
> Letra D) Mostre que, após um certo número de movimentos, todas as moedas
> voltarão a ficar com a face coroa para cima.
>
>
>
> --
> Esta mensagem foi verificada pelo sistema de antivírus e
> acredita-se estar livre de perigo.

-- 
Esta mensagem foi verificada pelo sistema de antiv�rus e
 acredita-se estar livre de perigo.



Re: [obm-l] Lei dos cossenos e Lei dos senos

2021-10-15 Por tôpico Anderson Torres
Smells like argumentação circular.

Em qui., 30 de set. de 2021 às 17:09, Israel Meireles Chrisostomo
 escreveu:
>
> Olá, ultimamente fiz uma prova para lei dos cossenos e senos, mas não sei se 
> está correta, alguém poderia por favor  me ajudar na correção?
> O link com a solução segue abaixo
> https://www.overleaf.com/read/zfcqwwmgxnrt
>
> --
> Esta mensagem foi verificada pelo sistema de antivírus e
> acredita-se estar livre de perigo.

-- 
Esta mensagem foi verificada pelo sistema de antiv�rus e
 acredita-se estar livre de perigo.


=
Instru��es para entrar na lista, sair da lista e usar a lista em
http://www.mat.puc-rio.br/~obmlistas/obm-l.html
=


Re: [obm-l] geometria

2021-09-28 Por tôpico Israel Meireles Chrisostomo
Muito obrigado

Em seg, 27 de set de 2021 21:25, Claudio Buffara 
escreveu:

> O caso LLL de congruência implica que, dados 3 segmentos que obedecem aa
> desigualdade triangular, o triângulo que os tem como lados é unicamente
> determinado, a menos de uma isometria.
>
> Enviado do meu iPhone
>
> > Em 27 de set. de 2021, à(s) 19:50, Israel Meireles Chrisostomo <
> israelmchrisost...@gmail.com> escreveu:
> >
> > 
> >
> > Olá pessoal. como faço para provar que o triângulo é um polígono
> rígido?
> >
> >
> > Abraços, muito obrigado
> > --
> > Israel Meireles Chrisostomo
> >
> > --
> > Esta mensagem foi verificada pelo sistema de antivírus e
> > acredita-se estar livre de perigo.
>
> --
> Esta mensagem foi verificada pelo sistema de antivírus e
>  acredita-se estar livre de perigo.
>
>
> =
> Instru�ões para entrar na lista, sair da lista e usar a lista em
> http://www.mat.puc-rio.br/~obmlistas/obm-l.html
> =
>

-- 
Esta mensagem foi verificada pelo sistema de antiv�rus e
 acredita-se estar livre de perigo.



Re: [obm-l] geometria

2021-09-27 Por tôpico Claudio Buffara
O caso LLL de congruência implica que, dados 3 segmentos que obedecem aa 
desigualdade triangular, o triângulo que os tem como lados é unicamente 
determinado, a menos de uma isometria.

Enviado do meu iPhone

> Em 27 de set. de 2021, à(s) 19:50, Israel Meireles Chrisostomo 
>  escreveu:
> 
> 
> 
> Olá pessoal. como faço para provar que o triângulo é um polígono rígido?
> 
> 
> Abraços, muito obrigado
> -- 
> Israel Meireles Chrisostomo
> 
> -- 
> Esta mensagem foi verificada pelo sistema de antivírus e 
> acredita-se estar livre de perigo.

-- 
Esta mensagem foi verificada pelo sistema de antiv�rus e
 acredita-se estar livre de perigo.


=
Instru��es para entrar na lista, sair da lista e usar a lista em
http://www.mat.puc-rio.br/~obmlistas/obm-l.html
=


Re: [obm-l] Conjuntos

2021-09-27 Por tôpico Pacini Bores
 

10% 

Em 26/09/2021 3:47, marcone augusto araújo borges escreveu: 

> Uma pessoa cética em relação às boas intenções da humanidade acredita que 70% 
> dos homens são violentos, 70% são desonestos e 70% são intolerantes. Se essa 
> pessoa estiver certa, em uma amostra ideal de 100 homens, quantos são, no 
> mínimo, simultaneamente desonestos, violentos e intolerantes? 
> -- 
> Esta mensagem foi verificada pelo sistema de antivírus e 
> acredita-se estar livre de perigo.

 
-- 
Esta mensagem foi verificada pelo sistema de antiv�rus e
 acredita-se estar livre de perigo.



Re: [obm-l] lei dos senos

2021-09-25 Por tôpico Israel Meireles Chrisostomo
Muito obrigado

Em sex, 24 de set de 2021 14:33, Claudio Buffara 
escreveu:

> Se os ângulos do triângulo são dados, então o triângulo fica determinado a
> menos de uma semelhança.
> Daí, dado um lado, os outros ficam unicamente determinados, e
> necessariamente obedecem à lei dos senos.
>
> Ou seja, dados a, b, c ângulos de um triângulo, e o lado de medida m,
> oposto ao ângulo a, os lados de medidas n e o ficam unicamente
> determinados, por:
> n = m*sen(b)/sen(a)   e   o = m*sen(c)/sen(a).
>
> On Fri, Sep 24, 2021 at 1:07 AM Israel Meireles Chrisostomo <
> israelmchrisost...@gmail.com> wrote:
>
>> Olá pessoal, eu estava resolvendo um problema  daí então surgiu uma
>> dúvida.A dúvida é a seguinte: sejam a,b,c ângulos  de um triângulo e m,n,o
>> lados de um triângulo qualquer , como provar que se m/sen(a)= n/sen(b)=
>> o/sen(c) então a,b,c e m,n,o pertencem ao mesmo triângulo. ou seja vale a
>> lei dos senos.
>>
>> --
>> Israel Meireles Chrisostomo
>>
>> --
>> Esta mensagem foi verificada pelo sistema de antivírus e
>> acredita-se estar livre de perigo.
>
>
> --
> Esta mensagem foi verificada pelo sistema de antivírus e
> acredita-se estar livre de perigo.

-- 
Esta mensagem foi verificada pelo sistema de antiv�rus e
 acredita-se estar livre de perigo.



Re: [obm-l] lei dos senos

2021-09-24 Por tôpico Claudio Buffara
Se os ângulos do triângulo são dados, então o triângulo fica determinado a
menos de uma semelhança.
Daí, dado um lado, os outros ficam unicamente determinados, e
necessariamente obedecem à lei dos senos.

Ou seja, dados a, b, c ângulos de um triângulo, e o lado de medida m,
oposto ao ângulo a, os lados de medidas n e o ficam unicamente
determinados, por:
n = m*sen(b)/sen(a)   e   o = m*sen(c)/sen(a).

On Fri, Sep 24, 2021 at 1:07 AM Israel Meireles Chrisostomo <
israelmchrisost...@gmail.com> wrote:

> Olá pessoal, eu estava resolvendo um problema  daí então surgiu uma
> dúvida.A dúvida é a seguinte: sejam a,b,c ângulos  de um triângulo e m,n,o
> lados de um triângulo qualquer , como provar que se m/sen(a)= n/sen(b)=
> o/sen(c) então a,b,c e m,n,o pertencem ao mesmo triângulo. ou seja vale a
> lei dos senos.
>
> --
> Israel Meireles Chrisostomo
>
> --
> Esta mensagem foi verificada pelo sistema de antivírus e
> acredita-se estar livre de perigo.

-- 
Esta mensagem foi verificada pelo sistema de antiv�rus e
 acredita-se estar livre de perigo.



[obm-l] Re: [obm-l] Re: [obm-l] Re: [obm-l] Re: matemática discreta

2021-09-20 Por tôpico Anderson Torres
Não consegui entender esse texto.

Em seg., 20 de set. de 2021 às 22:37, Israel Meireles Chrisostomo
 escreveu:
>
> Obrigado
>
> Em seg, 20 de set de 2021 22:00, Israel Meireles Chrisostomo 
>  escreveu:
>>
>> Tome n maior que n
>>
>> Em seg, 20 de set de 2021 20:49, Marcelo Salhab Brogliato 
>>  escreveu:
>>>
>>> Oi Israel,
>>>
>>> Não consegui entender a questão.
>>>
>>> Exemplo:
>>>
>>> n = 10, m = 3, Fib(10 - 3 + 1) = Fib(8) = 21
>>>
>>> (alpha**(2*n)) / (alpha**(n - m)) = alpha**(n + m) = 521.0019193787257
>>>
>>> Pela sua igualdade, alpha**(n + m) deveria ser 1/21, correto?
>>>
>>> Abraços,
>>> Marcelo
>>>
>>> Il giorno lun 20 set 2021 alle ore 15:54 Israel Meireles Chrisostomo 
>>>  ha scritto:

 já tentei de tudo, por favor me ajudem.

 Em seg., 20 de set. de 2021 às 19:39, Israel Meireles Chrisostomo 
  escreveu:
>
> Alguém poderia resolver o problema no link abaixo?
>
> https://mathoverflow.net/questions/404417/alpha2n-fracf-n-m1-alphan-m-1-how-to-prove-that-equality-is-true
>
> --
> Israel Meireles Chrisostomo



 --
 Israel Meireles Chrisostomo

 --
 Esta mensagem foi verificada pelo sistema de antivírus e
 acredita-se estar livre de perigo.
>>>
>>>
>>> --
>>> Esta mensagem foi verificada pelo sistema de antivírus e
>>> acredita-se estar livre de perigo.
>
>
> --
> Esta mensagem foi verificada pelo sistema de antivírus e
> acredita-se estar livre de perigo.

-- 
Esta mensagem foi verificada pelo sistema de antiv�rus e
 acredita-se estar livre de perigo.


=
Instru��es para entrar na lista, sair da lista e usar a lista em
http://www.mat.puc-rio.br/~obmlistas/obm-l.html
=


[obm-l] Re: [obm-l] Re: [obm-l] Re: matemática discreta

2021-09-20 Por tôpico Israel Meireles Chrisostomo
Obrigado

Em seg, 20 de set de 2021 22:00, Israel Meireles Chrisostomo <
israelmchrisost...@gmail.com> escreveu:

> Tome n maior que n
>
> Em seg, 20 de set de 2021 20:49, Marcelo Salhab Brogliato <
> msbro...@gmail.com> escreveu:
>
>> Oi Israel,
>>
>> Não consegui entender a questão.
>>
>> Exemplo:
>>
>> n = 10, m = 3, Fib(10 - 3 + 1) = Fib(8) = 21
>>
>> (alpha**(2*n)) / (alpha**(n - m)) = alpha**(n + m) = 521.0019193787257
>>
>> Pela sua igualdade, alpha**(n + m) deveria ser 1/21, correto?
>>
>> Abraços,
>> Marcelo
>>
>> Il giorno lun 20 set 2021 alle ore 15:54 Israel Meireles Chrisostomo <
>> israelmchrisost...@gmail.com> ha scritto:
>>
>>> já tentei de tudo, por favor me ajudem.
>>>
>>> Em seg., 20 de set. de 2021 às 19:39, Israel Meireles Chrisostomo <
>>> israelmchrisost...@gmail.com> escreveu:
>>>
 Alguém poderia resolver o problema no link abaixo?


 https://mathoverflow.net/questions/404417/alpha2n-fracf-n-m1-alphan-m-1-how-to-prove-that-equality-is-true

 --
 Israel Meireles Chrisostomo

>>>
>>>
>>> --
>>> Israel Meireles Chrisostomo
>>>
>>> --
>>> Esta mensagem foi verificada pelo sistema de antivírus e
>>> acredita-se estar livre de perigo.
>>
>>
>> --
>> Esta mensagem foi verificada pelo sistema de antivírus e
>> acredita-se estar livre de perigo.
>
>

-- 
Esta mensagem foi verificada pelo sistema de antiv�rus e
 acredita-se estar livre de perigo.



[obm-l] Re: [obm-l] Re: [obm-l] Re: matemática discreta

2021-09-20 Por tôpico Israel Meireles Chrisostomo
Tome n maior que n

Em seg, 20 de set de 2021 20:49, Marcelo Salhab Brogliato <
msbro...@gmail.com> escreveu:

> Oi Israel,
>
> Não consegui entender a questão.
>
> Exemplo:
>
> n = 10, m = 3, Fib(10 - 3 + 1) = Fib(8) = 21
>
> (alpha**(2*n)) / (alpha**(n - m)) = alpha**(n + m) = 521.0019193787257
>
> Pela sua igualdade, alpha**(n + m) deveria ser 1/21, correto?
>
> Abraços,
> Marcelo
>
> Il giorno lun 20 set 2021 alle ore 15:54 Israel Meireles Chrisostomo <
> israelmchrisost...@gmail.com> ha scritto:
>
>> já tentei de tudo, por favor me ajudem.
>>
>> Em seg., 20 de set. de 2021 às 19:39, Israel Meireles Chrisostomo <
>> israelmchrisost...@gmail.com> escreveu:
>>
>>> Alguém poderia resolver o problema no link abaixo?
>>>
>>>
>>> https://mathoverflow.net/questions/404417/alpha2n-fracf-n-m1-alphan-m-1-how-to-prove-that-equality-is-true
>>>
>>> --
>>> Israel Meireles Chrisostomo
>>>
>>
>>
>> --
>> Israel Meireles Chrisostomo
>>
>> --
>> Esta mensagem foi verificada pelo sistema de antivírus e
>> acredita-se estar livre de perigo.
>
>
> --
> Esta mensagem foi verificada pelo sistema de antivírus e
> acredita-se estar livre de perigo.

-- 
Esta mensagem foi verificada pelo sistema de antiv�rus e
 acredita-se estar livre de perigo.



[obm-l] Re: [obm-l] Re: matemática discreta

2021-09-20 Por tôpico Marcelo Salhab Brogliato
Oi Israel,

Não consegui entender a questão.

Exemplo:

n = 10, m = 3, Fib(10 - 3 + 1) = Fib(8) = 21

(alpha**(2*n)) / (alpha**(n - m)) = alpha**(n + m) = 521.0019193787257

Pela sua igualdade, alpha**(n + m) deveria ser 1/21, correto?

Abraços,
Marcelo

Il giorno lun 20 set 2021 alle ore 15:54 Israel Meireles Chrisostomo <
israelmchrisost...@gmail.com> ha scritto:

> já tentei de tudo, por favor me ajudem.
>
> Em seg., 20 de set. de 2021 às 19:39, Israel Meireles Chrisostomo <
> israelmchrisost...@gmail.com> escreveu:
>
>> Alguém poderia resolver o problema no link abaixo?
>>
>>
>> https://mathoverflow.net/questions/404417/alpha2n-fracf-n-m1-alphan-m-1-how-to-prove-that-equality-is-true
>>
>> --
>> Israel Meireles Chrisostomo
>>
>
>
> --
> Israel Meireles Chrisostomo
>
> --
> Esta mensagem foi verificada pelo sistema de antivírus e
> acredita-se estar livre de perigo.

-- 
Esta mensagem foi verificada pelo sistema de antiv�rus e
 acredita-se estar livre de perigo.



[obm-l] Re: [obm-l] Re: [obm-l] cálculo

2021-09-15 Por tôpico Israel Meireles Chrisostomo
Muito obrigado

Em qua, 15 de set de 2021 11:36, Esdras Muniz 
escreveu:

> O ponto é que tanto o conjunto dos números racionais quanto o conjunto dos
> números irracionais são densos em R. Portanto, para todo intervalo não
> degenerado, o máximo de f será 1 e o mínimo de f será zero. Daí, a integral
> superior será sempre maior que a integral inferior, portanto a função não é
> integravel.
>
> Em qua, 15 de set de 2021 00:11, Israel Meireles Chrisostomo <
> israelmchrisost...@gmail.com> escreveu:
>
>> Olá pessoal. eu estou me esforçando para entender esse exemplo do
>> guidorizzi, alguém poderia me explicar?Aqui vai:
>> Seja f uma função, tal que se x é racional então f igual a 1, se x é
>> irracional então f igual a zero. Mostre que a função não é riemann
>> integrável.
>>
>> --
>> Israel Meireles Chrisostomo
>>
>> --
>> Esta mensagem foi verificada pelo sistema de antivírus e
>> acredita-se estar livre de perigo.
>
>
> --
> Esta mensagem foi verificada pelo sistema de antivírus e
> acredita-se estar livre de perigo.

-- 
Esta mensagem foi verificada pelo sistema de antiv�rus e
 acredita-se estar livre de perigo.



[obm-l] Re: [obm-l] cálculo

2021-09-15 Por tôpico Esdras Muniz
O ponto é que tanto o conjunto dos números racionais quanto o conjunto dos
números irracionais são densos em R. Portanto, para todo intervalo não
degenerado, o máximo de f será 1 e o mínimo de f será zero. Daí, a integral
superior será sempre maior que a integral inferior, portanto a função não é
integravel.

Em qua, 15 de set de 2021 00:11, Israel Meireles Chrisostomo <
israelmchrisost...@gmail.com> escreveu:

> Olá pessoal. eu estou me esforçando para entender esse exemplo do
> guidorizzi, alguém poderia me explicar?Aqui vai:
> Seja f uma função, tal que se x é racional então f igual a 1, se x é
> irracional então f igual a zero. Mostre que a função não é riemann
> integrável.
>
> --
> Israel Meireles Chrisostomo
>
> --
> Esta mensagem foi verificada pelo sistema de antivírus e
> acredita-se estar livre de perigo.

-- 
Esta mensagem foi verificada pelo sistema de antiv�rus e
 acredita-se estar livre de perigo.



[obm-l] Re: [obm-l] cálculo

2021-09-15 Por tôpico Pedro Angelo
A definição de integrabilidade Riemann passa por verificar que, para
partições P suficientemente finas, a soma superior S(f;P) é parecida
com a soma inferior s(f;P).

Faça o que sempre deve ser feito nesse tipo de problema: calcule
exemplos concretos. Escolha partições quaisquer (pequenas, pois vc
quer conseguir fazer as contas na mão), e calcule as somas inferior e
superior para cada partição escolhida. O que acontece à medida que as
partições vão ficando cada vez mais finas?

On Wed, Sep 15, 2021 at 12:11 AM Israel Meireles Chrisostomo
 wrote:
>
> Olá pessoal. eu estou me esforçando para entender esse exemplo do guidorizzi, 
> alguém poderia me explicar?Aqui vai:
> Seja f uma função, tal que se x é racional então f igual a 1, se x é 
> irracional então f igual a zero. Mostre que a função não é riemann integrável.
>
> --
> Israel Meireles Chrisostomo
>
> --
> Esta mensagem foi verificada pelo sistema de antivírus e
> acredita-se estar livre de perigo.

-- 
Esta mensagem foi verificada pelo sistema de antiv�rus e
 acredita-se estar livre de perigo.


=
Instru��es para entrar na lista, sair da lista e usar a lista em
http://www.mat.puc-rio.br/~obmlistas/obm-l.html
=


[obm-l] Re: [obm-l] Ajuda numa questão da OBM 1987

2021-08-21 Por tôpico Anderson Torres
Em ter., 20 de jul. de 2021 às 18:25, Prof. Douglas Oliveira
 escreveu:
>
> Tem-se um bolo em forma de prisma triangular, cuja base está em um plano 
> horizontal. Dois indivíduos vão dividir o bolo de acordo com a seguinte 
> regra: o primeiro escolhe um ponto na base superior do bolo e o segundo corta 
> o bolo por um plano vertical à sua escolha, passando porém pelo ponto 
> escolhido, e seleciona para si um dos pedaços em que dividiu o bolo. Qual 
> deve ser a estratégia para o primeiro e qual deve ser a fração do volume do 
> bolo que ele espera obter?

Primeira dica: tente resolver o mesmo problema, mas para um triângulo
equilátero de lado 100.
Afinal de contas, uma transformação afim leva isso para qualquer triângulo.

>
> Abraço do Douglas.
>
> --
> Esta mensagem foi verificada pelo sistema de antivírus e
> acredita-se estar livre de perigo.

-- 
Esta mensagem foi verificada pelo sistema de antiv�rus e
 acredita-se estar livre de perigo.


=
Instru��es para entrar na lista, sair da lista e usar a lista em
http://www.mat.puc-rio.br/~obmlistas/obm-l.html
=


[obm-l] Re: [obm-l] "números biquadrados"

2021-08-13 Por tôpico Anderson Torres
Em qui, 12 de ago de 2021 21:17, marcone augusto araújo borges <
marconeborge...@hotmail.com> escreveu:

> 1233 = 12^2 + 33^2
> Em uma prova da bom nível 2, o número 1233 foi apresentado como
> "biquadrado" e foi pedido outro número biquadrado
> Eu pensei
> A^2+ B^2 = 100A + B
> A^2 - 100A + B^2 - B = 0
> Seriam dois valores para A cuja soma é 100, então se um deles é 12 o outro
> é 88
> Observei que 8833 = 88^2 + 33^2
> Se não fosse dado o 1233, daria para calcular os dois números...
> Como resolver A^2 + B^2 = 100A + B, com A e B inteiros positivos?
>

A^2 - 100A + B^2 - B = 0

4A^2 - 2*100*2A + 4B^2 - 2*2B = 0

(2A)^2 - 2*100*2A + 100^2 + (2B)^2 - 2*2B +1 = 100^2+1

(2A-100)^2+(2B-1)^2 = 10001

Agora é calcular mecanicamente todas as possibilidades para A e B.



> --
> Esta mensagem foi verificada pelo sistema de antivírus e
>


> acredita-se estar livre de perigo.
>

-- 
Esta mensagem foi verificada pelo sistema de antiv�rus e
 acredita-se estar livre de perigo.



[obm-l] Re: [obm-l] Re: [obm-l] Álgebra

2021-07-27 Por tôpico Anderson Torres
Em dom., 25 de jul. de 2021 às 15:23, Ralph Costa Teixeira
 escreveu:
>
> Sem ser muito formal: (a,b) e (c,d) sao dois vetores do plano, unitários e 
> ortogonais. Ou seja, um deles eh igual ao outro girado de 90 graus. Assim 
> (c,d)=(-b,a) ou (c,d)=(b,-a). De um jeito ou de outro, cd=-ab, ou seja, 
> resposta 0.
>
> On Sun, Jul 25, 2021 at 10:03 AM marcone augusto araújo borges 
>  wrote:
>>
>> a, b, c, d são números reais tais que a^2+b^2 = c^2 + d^2 = 1, ac + bd = 0. 
>> Calcule ab + cd
>> Desde já agradeço

Poderíamos escrever a=sinX, c=cosY. Assim sendo, b=cosX, d=sinY e daí
0=ac+bd=sinXcosY + cosXsinY = sin(X+Y), assim podemos usar X=-Y e daí
sinXcosX+sinYcosY = sinXcosX-sinXcosX=0
>>
>> --
>> Esta mensagem foi verificada pelo sistema de antivírus e
>> acredita-se estar livre de perigo.
>
>
> --
> Esta mensagem foi verificada pelo sistema de antivírus e
> acredita-se estar livre de perigo.

-- 
Esta mensagem foi verificada pelo sistema de antiv�rus e
 acredita-se estar livre de perigo.


=
Instru��es para entrar na lista, sair da lista e usar a lista em
http://www.mat.puc-rio.br/~obmlistas/obm-l.html
=


[obm-l] Re: [obm-l] Re: [obm-l] Álgebra

2021-07-25 Por tôpico Pacini Bores
 

Vi também assim : 

(ac+bd)(ad+bc) = cd(a^2+b^2)+ab(c^2+d^2). 

0= cd.1 + ab.1, logo ab+cd =0. 

É claro que a solução do Ralph é mais elegante... 

Abraços 

Pacini 

Em 25/07/2021 15:10, Ralph Costa Teixeira escreveu: 

> Sem ser muito formal: (a,b) e (c,d) sao dois vetores do plano, unitários e 
> ortogonais. Ou seja, um deles eh igual ao outro girado de 90 graus. Assim 
> (c,d)=(-b,a) ou (c,d)=(b,-a). De um jeito ou de outro, cd=-ab, ou seja, 
> resposta 0. 
> 
> On Sun, Jul 25, 2021 at 10:03 AM marcone augusto araújo borges 
>  wrote: 
> 
>> a, b, c, d são números reais tais que a^2+b^2 = c^2 + d^2 = 1, ac + bd = 0. 
>> Calcule ab + cd 
>> Desde já agradeço 
>> -- 
>> Esta mensagem foi verificada pelo sistema de antivírus e 
>> acredita-se estar livre de perigo.
> 
> -- 
> Esta mensagem foi verificada pelo sistema de antivrus e 
> acredita-se estar livre de perigo.

 
-- 
Esta mensagem foi verificada pelo sistema de antiv�rus e
 acredita-se estar livre de perigo.



[obm-l] RE: [obm-l] Novos avanços sobre a Hipótese do Continuum

2021-07-25 Por tôpico Paulo Santa Rita
Oi Bouskela e demais
membros desta lista... obm-l !

No mínimo interessante...  Na verdade, dizer que a cardinalidade do contínuo é 
"C" é apenas uma convenção e demonstração de ignorância, pois não sabemos 
(ainda) a que álefe da sequência do Cantor este "C" corresponde...

Se "C" for igual ao terceiro álefe, isto é, ao "álefe índice dois", é 
inevitável a pergunta sobre exemplos de conjunto com a cardinalidade do segundo 
álefe... O Erdos conseguiu um resultado interessante neste sentido, se não me 
falha a memória, sobre conjunto de funções analíticas.

Outro fato notável que ilustra a (maravilhosa) intuição do Godel é a previsão 
de que esta discussão toda sobre a hipótese do contínuo levaria a um conjunto 
de axiomas tipo ZFC onde as técnicas de forcing estariam "devidamente domadas" 
e novos e produtivos axiomas seriam descobertos.

Realmente interessante o artigo que você destacou.

Por oportuno e para não fugir ao espírito desta lista, alguém aqui conhece uma 
prova elementar (sem usar função zeta) da identidade entre a função seno e o 
produto de Euler  ?

sen(x) = x*[1  -  (x/pi)^2]*[1  -  (x/(2*pi))^2]*[1  -  (x/(3*pi))^2]*...*[1  - 
 (x/n*pi)^2]*...

Um abraço a Todos !
Paulo Santa Rita
1250721151E


Um abraço a todos !



De: owner-ob...@mat.puc-rio.br  em nome de 
bousk...@gmail.com 
Enviado: domingo, 18 de julho de 2021 17:06
Para: obm-l@mat.puc-rio.br 
Assunto: [obm-l] Novos avanços sobre a Hipótese do Continuum


Recebi da Quanta Magazine o artigo identificado a seguir:



How Many Numbers Exist? Infinity Proof Moves Math Closer to an Answer

For 50 years, mathematicians have believed that the total number of real 
numbers is unknowable. A new proof suggests otherwise.



Este artigo foi escrito por Natalie Wolchover em 15JUL2021 e, particularmente, 
o achei interessantíssimo.

--
Esta mensagem foi verificada pelo sistema de antivírus e
acredita-se estar livre de perigo.

-- 
Esta mensagem foi verificada pelo sistema de antivírus e
 acredita-se estar livre de perigo.



[obm-l] Re: [obm-l] Álgebra

2021-07-25 Por tôpico Ralph Costa Teixeira
Sem ser muito formal: (a,b) e (c,d) sao dois vetores do plano, unitários e
ortogonais. Ou seja, um deles eh igual ao outro girado de 90 graus. Assim
(c,d)=(-b,a) ou (c,d)=(b,-a). De um jeito ou de outro, cd=-ab, ou seja,
resposta 0.

On Sun, Jul 25, 2021 at 10:03 AM marcone augusto araújo borges <
marconeborge...@hotmail.com> wrote:

> a, b, c, d são números reais tais que a^2+b^2 = c^2 + d^2 = 1, ac + bd =
> 0. Calcule ab + cd
> Desde já agradeço
>
> --
> Esta mensagem foi verificada pelo sistema de antivírus e
> acredita-se estar livre de perigo.
>

-- 
Esta mensagem foi verificada pelo sistema de antiv�rus e
 acredita-se estar livre de perigo.



[obm-l] Re: [obm-l] Ajuda numa questão da OBM 1987

2021-07-20 Por tôpico joao pedro b menezes
Eu pensaria em trabalhar com os pontos notáveis, talvez o baricentro, e
argumentar que em qualquer outro ponto é possível realizar um corte que o
prejudique mais. Isso é só uma teoria e, portanto, é possível que esteja
totalmente errada.

-- 
Esta mensagem foi verificada pelo sistema de antiv�rus e
 acredita-se estar livre de perigo.



Re: [obm-l] Limites

2021-06-30 Por tôpico Anderson Torres
Em sex., 25 de jun. de 2021 às 23:38, Israel Meireles Chrisostomo
 escreveu:
>
> Alguém aí consegue provar o teorema do confronto?Em caso afirmativo por favor 
>  prove-o
>

??

> --
> Esta mensagem foi verificada pelo sistema de antivírus e
> acredita-se estar livre de perigo.

-- 
Esta mensagem foi verificada pelo sistema de antiv�rus e
 acredita-se estar livre de perigo.


=
Instru��es para entrar na lista, sair da lista e usar a lista em
http://www.mat.puc-rio.br/~obmlistas/obm-l.html
=


Re: [obm-l] Limites

2021-06-26 Por tôpico Israel Meireles Chrisostomo
muito obrigado por ser tão educado

Em sáb., 26 de jun. de 2021 às 11:27, Maikel Andril Marcelino <
maikel.marcel...@ifrn.edu.br> escreveu:

> https://www.youtube.com/watch?v=CWCVmgbePWY​
>
>
> Atenciosamente,
>
> *Maikel Andril Marcelino*
>
> *Assistente de Aluno - Secretaria Acadêmica -​ SEAC/SPP - Ramal: 7629 *
> *Coordenadoria de Apoio Acadêmico - COAPAC/IFRN-SPP*
> *Instituto Federal do Rio Grande do Norte*
> *Campus São Paulo do Potengi*
>
> *+55 **(84) 98851-3451*
> --
> *De:* owner-ob...@mat.puc-rio.br  em nome de
> Israel Meireles Chrisostomo 
> *Enviado:* sexta-feira, 25 de junho de 2021 23:27
> *Para:* obm-l
> *Assunto:* [obm-l] Limites
>
> Alguém aí consegue provar o teorema do confronto?Em caso afirmativo por
> favor  prove-o
>
> --
> Esta mensagem foi verificada pelo sistema de antiv�rus e
> acredita-se estar livre de perigo.
>
> --
> Esta mensagem foi verificada pelo sistema de antivírus e
> acredita-se estar livre de perigo.
>


-- 
Israel Meireles Chrisostomo

-- 
Esta mensagem foi verificada pelo sistema de antiv�rus e
 acredita-se estar livre de perigo.



Re: [obm-l] Limites

2021-06-26 Por tôpico Maikel Andril Marcelino
https://www.youtube.com/watch?v=CWCVmgbePWY?


Atenciosamente,

Maikel Andril Marcelino
Assistente de Aluno - Secretaria Acadêmica -? SEAC/SPP - Ramal: 7629
Coordenadoria de Apoio Acadêmico - COAPAC/IFRN-SPP
Instituto Federal do Rio Grande do Norte
Campus São Paulo do Potengi

+55 (84) 98851-3451

De: owner-ob...@mat.puc-rio.br  em nome de Israel 
Meireles Chrisostomo 
Enviado: sexta-feira, 25 de junho de 2021 23:27
Para: obm-l
Assunto: [obm-l] Limites

Alguém aí consegue provar o teorema do confronto?Em caso afirmativo por favor  
prove-o

--
Esta mensagem foi verificada pelo sistema de antiv?rus e
acredita-se estar livre de perigo.

-- 
Esta mensagem foi verificada pelo sistema de antivírus e
 acredita-se estar livre de perigo.



Re: [obm-l] probabilidade condicional

2021-06-25 Por tôpico Daniel Jelin
Obrigado, Ralph!

Em qui, 24 de jun de 2021 23:55, Ralph Costa Teixeira 
escreveu:

> Sim, são falsas!
>
> Seu exemplo mata o problema! Seus eventos A e B são independentes, mas:
>
> Em (1), P (A | B e C) = 0, enquanto P(A | C) = 1/2.
> Em (2), P(A e B | C) = 0, enquanto P(A | C) = P (B | C) = 1/2.
>
> Em suma, quando uma nova informação (C) chega, eventos (A) e (B) que eram
> independentes podem deixar de sê-lo!
>
> Abraco, Ralph.
>
> On Thu, Jun 24, 2021 at 9:57 PM Daniel Jelin 
> wrote:
>
>> Caros, duas dúvidas elementares sobre probabilidade condicional, quem
>> sabe possam me ajudar. Leio, em mais de um lugar, que:
>>
>> 1) Se A e B são independentes, então P(A | B e C) = P (A | C)
>>
>> A explicação parece fazer sentido: se A não depende de B, tanto que faz
>> que B seja dado ou não.
>>
>> Em conexão com esse problema, leio também que:
>>
>> 2) Se A e B são independentes, então P(A e B | C)=P(A | C)*P(B | C).
>>
>> A explicação, que tb parece boa, é que se P(A e B)=P(A)*P(B), então
>> podemos "condicionar" toda a igualdade a C, e ela ainda será verdadeira.
>>
>> Tenho tentado demonstrar essas afirmações, usando Bayes, mas não chego a
>> lugar nenhum... Além disso, penso que haja contra-exemplos simples pra
>> essas duas afirmações. Por exemplo: lanço dois dados e faço A={o primeiro
>> dado é par}, B={o segundo dado é par}, C={a soma dos dois dados é ímpar}. O
>> que acontece aqui? Essas afirmações fazem mesmo sentido?
>>
>> abs
>>
>> --
>> Esta mensagem foi verificada pelo sistema de antivírus e
>> acredita-se estar livre de perigo.
>
>
> --
> Esta mensagem foi verificada pelo sistema de antivírus e
> acredita-se estar livre de perigo.

-- 
Esta mensagem foi verificada pelo sistema de antiv�rus e
 acredita-se estar livre de perigo.



Re: [obm-l] probabilidade condicional

2021-06-24 Por tôpico Ralph Costa Teixeira
Sim, são falsas!

Seu exemplo mata o problema! Seus eventos A e B são independentes, mas:

Em (1), P (A | B e C) = 0, enquanto P(A | C) = 1/2.
Em (2), P(A e B | C) = 0, enquanto P(A | C) = P (B | C) = 1/2.

Em suma, quando uma nova informação (C) chega, eventos (A) e (B) que eram
independentes podem deixar de sê-lo!

Abraco, Ralph.

On Thu, Jun 24, 2021 at 9:57 PM Daniel Jelin  wrote:

> Caros, duas dúvidas elementares sobre probabilidade condicional, quem sabe
> possam me ajudar. Leio, em mais de um lugar, que:
>
> 1) Se A e B são independentes, então P(A | B e C) = P (A | C)
>
> A explicação parece fazer sentido: se A não depende de B, tanto que faz
> que B seja dado ou não.
>
> Em conexão com esse problema, leio também que:
>
> 2) Se A e B são independentes, então P(A e B | C)=P(A | C)*P(B | C).
>
> A explicação, que tb parece boa, é que se P(A e B)=P(A)*P(B), então
> podemos "condicionar" toda a igualdade a C, e ela ainda será verdadeira.
>
> Tenho tentado demonstrar essas afirmações, usando Bayes, mas não chego a
> lugar nenhum... Além disso, penso que haja contra-exemplos simples pra
> essas duas afirmações. Por exemplo: lanço dois dados e faço A={o primeiro
> dado é par}, B={o segundo dado é par}, C={a soma dos dois dados é ímpar}. O
> que acontece aqui? Essas afirmações fazem mesmo sentido?
>
> abs
>
> --
> Esta mensagem foi verificada pelo sistema de antivírus e
> acredita-se estar livre de perigo.

-- 
Esta mensagem foi verificada pelo sistema de antiv�rus e
 acredita-se estar livre de perigo.



Re: [obm-l] Produto infinito do seno

2021-06-18 Por tôpico Israel Meireles Chrisostomo
Esse daqui(da imagem abaixo)
 [image: image.png]


Livre
de vírus. www.avast.com
.
<#DAB4FAD8-2DD7-40BB-A1B8-4E2AA1F9FDF2>

Em sex., 18 de jun. de 2021 às 13:51, Israel Meireles Chrisostomo <
israelmchrisost...@gmail.com> escreveu:

> falo do produto infinito do seno de euler
>
>
> 
>  Livre
> de vírus. www.avast.com
> .
> <#m_-3237286192599090848_DAB4FAD8-2DD7-40BB-A1B8-4E2AA1F9FDF2>
>
> Em qua., 19 de mai. de 2021 às 10:33, Anderson Torres <
> torres.anderson...@gmail.com> escreveu:
>
>> Em seg., 17 de mai. de 2021 Ã s 18:58, Israel Meireles Chrisostomo
>>  escreveu:
>> >
>> > Alguém aí sabe quantas provas existem para se verificar q d fato o
>> produto infinito do seno é verdadeiro?Eu tenho uma.
>> >
>>
>> HEIN
>>
>> > --
>> > Esta mensagem foi verificada pelo sistema de antivírus e
>> > acredita-se estar livre de perigo.
>>
>> --
>> Esta mensagem foi verificada pelo sistema de antivírus e
>>  acredita-se estar livre de perigo.
>>
>>
>> =
>> Instruções para entrar na lista, sair da lista e usar a lista em
>> http://www.mat.puc-rio.br/~obmlistas/obm-l.html
>> =
>>
>
>
> --
> Israel Meireles Chrisostomo
>


-- 
Israel Meireles Chrisostomo

-- 
Esta mensagem foi verificada pelo sistema de antiv�rus e
 acredita-se estar livre de perigo.



Re: [obm-l] Produto infinito do seno

2021-06-18 Por tôpico Israel Meireles Chrisostomo
falo do produto infinito do seno de euler


Livre
de vírus. www.avast.com
.
<#DAB4FAD8-2DD7-40BB-A1B8-4E2AA1F9FDF2>

Em qua., 19 de mai. de 2021 às 10:33, Anderson Torres <
torres.anderson...@gmail.com> escreveu:

> Em seg., 17 de mai. de 2021 Ã s 18:58, Israel Meireles Chrisostomo
>  escreveu:
> >
> > Alguém aí sabe quantas provas existem para se verificar q d fato o
> produto infinito do seno é verdadeiro?Eu tenho uma.
> >
>
> HEIN
>
> > --
> > Esta mensagem foi verificada pelo sistema de antivírus e
> > acredita-se estar livre de perigo.
>
> --
> Esta mensagem foi verificada pelo sistema de antivírus e
>  acredita-se estar livre de perigo.
>
>
> =
> Instruções para entrar na lista, sair da lista e usar a lista em
> http://www.mat.puc-rio.br/~obmlistas/obm-l.html
> =
>


-- 
Israel Meireles Chrisostomo

-- 
Esta mensagem foi verificada pelo sistema de antiv�rus e
 acredita-se estar livre de perigo.



[obm-l] Re: [obm-l] Re: [obm-l] Re: [obm-l] Re: [obm-l] Função

2021-05-29 Por tôpico Israel Meireles Chrisostomo
obrigado


Livre
de vírus. www.avast.com
.
<#DAB4FAD8-2DD7-40BB-A1B8-4E2AA1F9FDF2>

Em qua., 19 de mai. de 2021 às 10:56, Anderson Torres <
torres.anderson...@gmail.com> escreveu:

> Em seg., 26 de abr. de 2021 às 17:18, Israel Meireles Chrisostomo
>  escreveu:
> >
> > Mas aí então a+bi e b+ai são os mesmos números
>
> Não são.
>
> 4+5i e 5+4i são diferentes, e 4+5i < 5+4i por essas regras.
>
> >
> > Em seg, 26 de abr de 2021 13:36, Anderson Torres <
> torres.anderson...@gmail.com> escreveu:
> >>
> >> Em qui., 22 de abr. de 2021 às 07:19, Israel Meireles Chrisostomo
> >>  escreveu:
> >> >
> >> > Me desculpem se eu estou falando bobagem, mas considere uma função
> com domínio complexo, então essa função não pode ser bijetora, pois toda
> função bijetora ou é crescente ou é decrescente, mas não há ordem nos
> complexos
>
> Você não entendeu nada aqui, suponho. Primeiramente, funções não são
> coisas limitadas a números.
>
> Segundamente, quando usamos esse teorema de que funções contínuas são
> monótonas, é óbvio que estamos supondo de antemão que estamos
> trabalhando com um sistema numérico que admita a ideia de ordem.
> Especialmente, a de um corpo ordenado completo.
>
> Por exemplo, não faz sentido falar de "continuidade" quando se fala de
> funções de naturais para naturais, porque números naturais não formam
> um sistema numérico contínuo.
>
> >>
> >> Não é correto dizer que não existe ordem nos complexos. É só atribuir
> >> o seguinte: o complexo A é maior que o complexo B se e somente se ou o
> >> módulo de A é maior que o de B ou os módulos são iguais mas o
> >> argumento de A é maior que o de B (tomando este módulo no intervalo de
> >> 0 a tau).
> >>
> >> >
> >> > --
> >> > Esta mensagem foi verificada pelo sistema de antivírus e
> >> > acredita-se estar livre de perigo.
> >>
> >> --
> >> Esta mensagem foi verificada pelo sistema de antivírus e
> >>  acredita-se estar livre de perigo.
> >>
> >>
> >>
> =
> >> Instru�ões para entrar na lista, sair da lista e usar a lista em
> >> http://www.mat.puc-rio.br/~obmlistas/obm-l.html
> >>
> =
> >
> >
> > --
> > Esta mensagem foi verificada pelo sistema de antivírus e
> > acredita-se estar livre de perigo.
>
> --
> Esta mensagem foi verificada pelo sistema de antivírus e
>  acredita-se estar livre de perigo.
>
>
> =
> Instru�ões para entrar na lista, sair da lista e usar a lista em
> http://www.mat.puc-rio.br/~obmlistas/obm-l.html
> =
>


-- 
Israel Meireles Chrisostomo

-- 
Esta mensagem foi verificada pelo sistema de antiv�rus e
 acredita-se estar livre de perigo.



[obm-l] Re: [obm-l] base de numeração

2021-05-19 Por tôpico Anderson Torres
Em ter., 27 de abr. de 2021 às 17:40, Daniel Quevedo
 escreveu:
>
> Os oito últimos algarismos do número 27^1986 quando escrito na base 2 são:
> a) 11011001
> b) 11011101
> c) 1001
> d) 11011011
> e) 10011001
>
> gab: A

Calcule o resto da divisão de 27^1986 por 2^8, depois converta para binário.

>
> --
> Fiscal: Daniel Quevedo
>
> --
> Esta mensagem foi verificada pelo sistema de antivírus e
> acredita-se estar livre de perigo.

-- 
Esta mensagem foi verificada pelo sistema de antiv�rus e
 acredita-se estar livre de perigo.


=
Instru��es para entrar na lista, sair da lista e usar a lista em
http://www.mat.puc-rio.br/~obmlistas/obm-l.html
=


[obm-l] Re: [obm-l] Re: [obm-l] Re: [obm-l] Função

2021-05-19 Por tôpico Anderson Torres
Em seg., 26 de abr. de 2021 às 17:18, Israel Meireles Chrisostomo
 escreveu:
>
> Mas aí então a+bi e b+ai são os mesmos números

Não são.

4+5i e 5+4i são diferentes, e 4+5i < 5+4i por essas regras.

>
> Em seg, 26 de abr de 2021 13:36, Anderson Torres 
>  escreveu:
>>
>> Em qui., 22 de abr. de 2021 às 07:19, Israel Meireles Chrisostomo
>>  escreveu:
>> >
>> > Me desculpem se eu estou falando bobagem, mas considere uma função com 
>> > domínio complexo, então essa função não pode ser bijetora, pois toda 
>> > função bijetora ou é crescente ou é decrescente, mas não há ordem nos 
>> > complexos

Você não entendeu nada aqui, suponho. Primeiramente, funções não são
coisas limitadas a números.

Segundamente, quando usamos esse teorema de que funções contínuas são
monótonas, é óbvio que estamos supondo de antemão que estamos
trabalhando com um sistema numérico que admita a ideia de ordem.
Especialmente, a de um corpo ordenado completo.

Por exemplo, não faz sentido falar de "continuidade" quando se fala de
funções de naturais para naturais, porque números naturais não formam
um sistema numérico contínuo.

>>
>> Não é correto dizer que não existe ordem nos complexos. É só atribuir
>> o seguinte: o complexo A é maior que o complexo B se e somente se ou o
>> módulo de A é maior que o de B ou os módulos são iguais mas o
>> argumento de A é maior que o de B (tomando este módulo no intervalo de
>> 0 a tau).
>>
>> >
>> > --
>> > Esta mensagem foi verificada pelo sistema de antivírus e
>> > acredita-se estar livre de perigo.
>>
>> --
>> Esta mensagem foi verificada pelo sistema de antivírus e
>>  acredita-se estar livre de perigo.
>>
>>
>> =
>> Instru�ões para entrar na lista, sair da lista e usar a lista em
>> http://www.mat.puc-rio.br/~obmlistas/obm-l.html
>> =
>
>
> --
> Esta mensagem foi verificada pelo sistema de antivírus e
> acredita-se estar livre de perigo.

-- 
Esta mensagem foi verificada pelo sistema de antiv�rus e
 acredita-se estar livre de perigo.


=
Instru��es para entrar na lista, sair da lista e usar a lista em
http://www.mat.puc-rio.br/~obmlistas/obm-l.html
=


Re: [obm-l] Produto infinito do seno

2021-05-19 Por tôpico Anderson Torres
Em seg., 17 de mai. de 2021 às 18:58, Israel Meireles Chrisostomo
 escreveu:
>
> Alguém aí sabe quantas provas existem para se verificar q d fato o produto 
> infinito do seno é verdadeiro?Eu tenho uma.
>

HEIN

> --
> Esta mensagem foi verificada pelo sistema de antivírus e
> acredita-se estar livre de perigo.

-- 
Esta mensagem foi verificada pelo sistema de antiv�rus e
 acredita-se estar livre de perigo.


=
Instru��es para entrar na lista, sair da lista e usar a lista em
http://www.mat.puc-rio.br/~obmlistas/obm-l.html
=


Re: [obm-l] f(x + y) = f(x) + f(y)

2021-05-10 Por tôpico Bruno Visnadi
A princípio, não há nada que garanta que f seja derivável ou mesmo que o
limite exista para esta prova valer. Mas, de fato, se o domínio está
restrito a Q, você pode mostrar que f(x) = ax para algum a. Um caminho é
definir f(1) = a e mostrar que f(1/n) = a/n, para então chegar em f(m/n) =
ma/n.

Se o domínio fosse R, no entanto, não seguiria que f(x) = ax. O máximo que
poderia-se dizer é que se T ∈ R é tal que existe um r de modo que para todo
x ∈ T, x/r ∈ Q, existe um b tal que f(x) = bx para todo x ∈ T. Ou seja,
dentro de determinadas classes de R, vale f(x) = ax, mas o valor de a pode
mudar para cada classe.

Em qua., 5 de mai. de 2021 às 11:14, Claudio Buffara <
claudio.buff...@gmail.com> escreveu:

> f(x) = ax + b só satisfaz isso se b = 0.
> Tente com x+1, por exemplo.
> E mais: sem alguma outra condição (do tipo continuidade ou monotonicidade)
> ainda assim a expressão não implica que f(x) = ax.
>
> Abs,
> Cláudio.
>
>
> Enviado do meu iPhone
>
> > Em 5 de mai. de 2021, à(s) 09:13, joao pedro b menezes <
> joaopedrobmene...@gmail.com> escreveu:
> >
> > Eu estava fazendo um exercício de equações funcionais e me deparei
> com essa expressão. Não sei o que aconteceu, mas tive uma crise
> existencial e decidi provar que isso implica f(x) = ax + b( ou pelo menos
> acho que implica). Essa prova estaria certa?:
> > (obs: a função é definida nos racionais)
> > f(x + 0) = f(x) + f(0) => f(0) = 0.
> > f(x + h) = f(x) + f(h) ->
> > (f(x + h) - f(x))/h = f(h)/h = (f(h) - f(0))/h
> > agora basta fazer lim h -> 0 e obtemosÂ
> > f’(x) = f’(0) . Mas f’(0) é uma constante, logo f(x) = ax + b
> > (obs: tenho quase certeza que ela seria válida para os reais, porém
> como a função é limitada aos racionais, estou em dúvida)
> >
> > --
> > Esta mensagem foi verificada pelo sistema de antivírus e
> > acredita-se estar livre de perigo.
>
> --
> Esta mensagem foi verificada pelo sistema de antivírus e
>  acredita-se estar livre de perigo.
>
>
> =
> Instru�ões para entrar na lista, sair da lista e usar a lista em
> http://www.mat.puc-rio.br/~obmlistas/obm-l.html
> =
>

-- 
Esta mensagem foi verificada pelo sistema de antiv�rus e
 acredita-se estar livre de perigo.



Assunto: Re: [obm-l] f(x + y) = f(x) + f(y)

2021-05-05 Por tôpico Artur Costa Steiner
Se vc adicionar a hipótese de que f é contínua em algum real x0, a conclusão 
desejada torna-se válida.
Se vc quiser elocubrar um pouco, pode seguir os seguintes passos,:
Mostre que continuidade em x0 implica continuidade em 0 que, por sua vez, 
implica continuidade em toda a reta real.
Mostre que f(x) = ax vale para todo inteiro x.
Mostre  que f(x) = ax vale para todo x da forma x = 1/n, n inteiro não nulo. 
Disso conclua que vale para todo racional x
Veja wue g(x) = ax é contínua e coincide com f nos racionais, que são densos em 
R. Logo, f = g em R.
Se a hipótese adicionada for de monotonicidade, então o conjunto das 
descontinuidades de f é enumerável, o que implica que o conjunto das 
continuidades seja não enumerável, logo não vazio. E o caso anterior se aplica.
AbrsArtur


Enviado do Yahoo Mail no Android 
 
  Em qua., 5 5e mai. 5e 2021 às 11:45, Claudio 
Buffara escreveu:   f(x) = ax + b só satisfaz isso 
se b = 0.
Tente com x+1, por exemplo.
E mais: sem alguma outra condição (do tipo continuidade ou monotonicidade) 
ainda assim a expressão não implica que f(x) = ax.

Abs,
Cláudio.


Enviado do meu iPhone

> Em 5 de mai. de 2021, à(s) 09:13, joao pedro b menezes 
>  escreveu:
> 
> Eu estava fazendo um exercício de equações funcionais e me deparei com 
> essa expressão. Não sei o que aconteceu, mas tive uma crise existencial e 
> decidi provar que isso implica f(x) = ax + b( ou pelo menos acho que 
> implica). Essa prova estaria certa?:
> (obs: a função é definida nos racionais)
> f(x + 0) = f(x) + f(0) => f(0) = 0.
> f(x + h) = f(x) + f(h) ->
> (f(x + h) - f(x))/h = f(h)/h = (f(h) - f(0))/h
> agora basta fazer lim h -> 0 e obtemos 
> f’(x) = f’(0) . Mas f’(0) é uma constante, logo f(x) = ax + b
> (obs: tenho quase certeza que ela seria válida para os reais, porém como a 
> função é limitada aos racionais, estou em dúvida)
> 
> -- 
> Esta mensagem foi verificada pelo sistema de antivírus e 
> acredita-se estar livre de perigo.

-- 
Esta mensagem foi verificada pelo sistema de antiv�rus e
 acredita-se estar livre de perigo.


=
Instru��es para entrar na lista, sair da lista e usar a lista em
http://www.mat.puc-rio.br/~obmlistas/obm-l.html
=
  

-- 
Esta mensagem foi verificada pelo sistema de antiv�rus e
 acredita-se estar livre de perigo.



Re: [obm-l] f(x + y) = f(x) + f(y)

2021-05-05 Por tôpico Claudio Buffara
f(x) = ax + b só satisfaz isso se b = 0.
Tente com x+1, por exemplo.
E mais: sem alguma outra condição (do tipo continuidade ou monotonicidade) 
ainda assim a expressão não implica que f(x) = ax.

Abs,
Cláudio.


Enviado do meu iPhone

> Em 5 de mai. de 2021, à(s) 09:13, joao pedro b menezes 
>  escreveu:
> 
> Eu estava fazendo um exercício de equações funcionais e me deparei com 
> essa expressão. Não sei o que aconteceu, mas tive uma crise existencial e 
> decidi provar que isso implica f(x) = ax + b( ou pelo menos acho que 
> implica). Essa prova estaria certa?:
> (obs: a função é definida nos racionais)
> f(x + 0) = f(x) + f(0) => f(0) = 0.
> f(x + h) = f(x) + f(h) ->
> (f(x + h) - f(x))/h = f(h)/h = (f(h) - f(0))/h
> agora basta fazer lim h -> 0 e obtemos 
> f’(x) = f’(0) . Mas f’(0) é uma constante, logo f(x) = ax + b
> (obs: tenho quase certeza que ela seria válida para os reais, porém como a 
> função é limitada aos racionais, estou em dúvida)
> 
> -- 
> Esta mensagem foi verificada pelo sistema de antivírus e 
> acredita-se estar livre de perigo.

-- 
Esta mensagem foi verificada pelo sistema de antiv�rus e
 acredita-se estar livre de perigo.


=
Instru��es para entrar na lista, sair da lista e usar a lista em
http://www.mat.puc-rio.br/~obmlistas/obm-l.html
=


[obm-l] Re: [obm-l] Re: [obm-l] Função

2021-04-26 Por tôpico Israel Meireles Chrisostomo
Mas aí então a+bi e b+ai são os mesmos números

Em seg, 26 de abr de 2021 13:36, Anderson Torres <
torres.anderson...@gmail.com> escreveu:

> Em qui., 22 de abr. de 2021 às 07:19, Israel Meireles Chrisostomo
>  escreveu:
> >
> > Me desculpem se eu estou falando bobagem, mas considere uma função com
> domínio complexo, então essa função não pode ser bijetora, pois toda função
> bijetora ou é crescente ou é decrescente, mas não há ordem nos complexos
>
> Não é correto dizer que não existe ordem nos complexos. É só atribuir
> o seguinte: o complexo A é maior que o complexo B se e somente se ou o
> módulo de A é maior que o de B ou os módulos são iguais mas o
> argumento de A é maior que o de B (tomando este módulo no intervalo de
> 0 a tau).
>
> >
> > --
> > Esta mensagem foi verificada pelo sistema de antivírus e
> > acredita-se estar livre de perigo.
>
> --
> Esta mensagem foi verificada pelo sistema de antivírus e
>  acredita-se estar livre de perigo.
>
>
> =
> Instru�ões para entrar na lista, sair da lista e usar a lista em
> http://www.mat.puc-rio.br/~obmlistas/obm-l.html
> =
>

-- 
Esta mensagem foi verificada pelo sistema de antiv�rus e
 acredita-se estar livre de perigo.



[obm-l] Re: [obm-l] Função

2021-04-26 Por tôpico Anderson Torres
Em qui., 22 de abr. de 2021 às 07:19, Israel Meireles Chrisostomo
 escreveu:
>
> Me desculpem se eu estou falando bobagem, mas considere uma função com 
> domínio complexo, então essa função não pode ser bijetora, pois toda função 
> bijetora ou é crescente ou é decrescente, mas não há ordem nos complexos

Não é correto dizer que não existe ordem nos complexos. É só atribuir
o seguinte: o complexo A é maior que o complexo B se e somente se ou o
módulo de A é maior que o de B ou os módulos são iguais mas o
argumento de A é maior que o de B (tomando este módulo no intervalo de
0 a tau).

>
> --
> Esta mensagem foi verificada pelo sistema de antivírus e
> acredita-se estar livre de perigo.

-- 
Esta mensagem foi verificada pelo sistema de antiv�rus e
 acredita-se estar livre de perigo.


=
Instru��es para entrar na lista, sair da lista e usar a lista em
http://www.mat.puc-rio.br/~obmlistas/obm-l.html
=


Re: [obm-l]

2021-04-26 Por tôpico Anderson Torres
Em dom., 25 de abr. de 2021 às 14:34, Artur Costa Steiner
 escreveu:
>
> Oh, no meu email anterior, onde se
> lê raiz(3), leia-se raiz_cúbica(2). Tô fazendo um tratamento na vista e ando 
> com dificuldade para digitar num celular.
> Um cara de 69 anos como eu não deveria mais participar deste grupo

Como se estivéssemos disputando uma Medalha Fields!

>
> Artur
>
>
>
> Em dom., 25 de abr. de 2021 14:16, Artur Costa Steiner 
>  escreveu:
>>
>> raiz(2)) e raiz(3) são inteiros algébricos, visto serem raízes de x^2 - 2 e 
>> x^3 - 2, respectivamente. Segundo um clássico teorema da Teoria dos Números, 
>> a soma de dois inteiros algébricos é inteira algébrica. E um inteiro 
>> algébrico é racional se, e somente se, for inteiro. Como, conforme já 
>> comentado, este não é o caso de raiz(2) + raiz(3), segue-se que está soma é 
>> irracional.
>>
>> Abraços
>> Artur
>>
>>
>> Em sex., 23 de abr. de 2021 17:43, Marcos Martinelli 
>>  escreveu:
>>>
>>> Legal, Matheus.
>>>
>>> Minha ideia foi encontrar um polinômio em m.n (m = raiz(2) e 
>>> n=raiz_cúbica(2)) de coeficientes racionais. Pra isso desenvolvi m^k + n^k 
>>> (k >= 0) até k=6 e encontrei um de grau 6 com coeficientes dependendo só de 
>>> m+n.
>>>
>>> Se m+n for racional, usei o fato de se a + beta (a racional e beta 
>>> irracional com beta^j  também irracional (1=< j <= grau do polinômio- 1) 
>>> for raiz desse polinômio então a - beta também seria.
>>>
>>> Mas essa sua ficou bem elegante.
>>>
>>> Brigado.
>>>
>>> Em sex., 23 de abr. de 2021 às 17:18, Matheus Secco 
>>>  escreveu:

 Oi, Marcos. Não é difícil verificar que raiz(2) + raiz_cubica(2) é uma 
 raiz do polinômio x^6 - 6 x^4 - 4 x^3 + 12 x^2 - 24 x - 4. Com isso, pelo 
 teorema das raízes racionais, se raiz(2) + raiz_cubica(2) fosse racional, 
 teria que ser um inteiro e é fácil verificar que 2 < raiz(2) + 
 raiz_cubica(2) < 3.

 Abraços

 On Fri, Apr 23, 2021 at 4:43 PM Marcos Martinelli 
  wrote:
>
> Opa, pessoal. Pensei nos últimos dias no problema seguinte. Cheguei a uma 
> solução um pouco mais genérica, mas me deu trabalho. Gostaria de estudar 
> outras abordagens.
>
> Problema) Prove que raiz (2) + raiz_cúbica (2) é irracional.
>
> Na sequência posto um rascunho do que pensei.
>
> Obrigado.
>
> --
> Esta mensagem foi verificada pelo sistema de antivírus e
> acredita-se estar livre de perigo.


 --
 Esta mensagem foi verificada pelo sistema de antivírus e
 acredita-se estar livre de perigo.
>>>
>>>
>>> --
>>> Esta mensagem foi verificada pelo sistema de antivírus e
>>> acredita-se estar livre de perigo.
>
>
> --
> Esta mensagem foi verificada pelo sistema de antivírus e
> acredita-se estar livre de perigo.

-- 
Esta mensagem foi verificada pelo sistema de antiv�rus e
 acredita-se estar livre de perigo.


=
Instru��es para entrar na lista, sair da lista e usar a lista em
http://www.mat.puc-rio.br/~obmlistas/obm-l.html
=


Re: [obm-l] Magnitude

2021-04-26 Por tôpico Anderson Torres
Em seg., 26 de abr. de 2021 às 00:16, Pedro Lazéra
 escreveu:
>
> Boa noite, Maikel.
>
> A quantidade de algarismos de um número x (na base 10) é "1 + piso de 
> log(x)", em que "log" é a função logaritmo na base 10. Você pode verificar 
> isso assim: 10^n, para n inteiro >= 0, é o menor número do mundo com n+1 
> algarismos. Além disso, log(10^n) = n. Por fim, log é uma função crescente.
>
> Usando que log(a*b) = log(a) + log(b), uma solução computeira é fazer um 
> programa que calcula log(i), para i inteiro entre 1 e 100, e soma esses 
> valores a uma variável s inicializada com o valor ZERO.
>
> Uma solução computeira (provavelmente) errada é calcular x = 100! e depois 
> achar o log(x). Esse valor x não cabe nas estruturas de dados que a maioria 
> das linguagens usa para representar números.
>
> Bom, a soma dá 157.97, conforme o Anderson Torres falou antes de mim. A gente 
> sabe que os computadores até erram (truncam) o valor exato de log(x), mas o 
> erro é bem pequeno e só estamos somando 100 aplicações da função log, daí 
> sabemos que esse 157.97 pode até estar errado, mas é por muito pouco (menos 
> do que 0,01, por exemplo).

Na pior hipótese, é só pegar cada log com 3 casas de precisão. Isso
significa que estaríamos somando os erros da terceira casa depois da
vírgula, que vezes 100 só chegariam na primeira casa depois da vírgula
no máximo: se cada erro fosse 0.009, vezes 100 isso daria um erro de
0.9.
Então nem tem com o que se preocupar aqui.

>
> Finalmente, 100! tem 1 + piso(157.97) = 158 algarismos.
>
> Abraços,
> Pedro
>
> On Sun, Apr 11, 2021 at 12:29 AM Anderson Torres 
>  wrote:
>>
>> Em sáb., 3 de abr. de 2021 às 01:13, Maikel Andril Marcelino
>>  escreveu:
>> >
>> > Quantos algarismos tem o número (100!) ?
>>
>> Em outras palavras, qual é o log(100!)/log(10). O Google me diz que
>> isso é 157,97 - logo, 158 dígitos.
>>
>> >
>> >
>> > Atenciosamente,
>> >
>> > Maikel Andril Marcelino
>> > Assistente de Aluno - Biblioteca - Ramal: 7616
>> > Coordenadoria de Apoio Acadêmico - COAPAC/IFRN-SPP
>> > Instituto Federal do Rio Grande do Norte
>> > Campus São Paulo do Potengi
>> >
>> > +55 (84) 8851-3451
>> >
>> > --
>> > Esta mensagem foi verificada pelo sistema de antivírus e
>> > acredita-se estar livre de perigo.
>>
>> --
>> Esta mensagem foi verificada pelo sistema de antivírus e
>>  acredita-se estar livre de perigo.
>>
>>
>> =
>> Instru�ões para entrar na lista, sair da lista e usar a lista em
>> http://www.mat.puc-rio.br/~obmlistas/obm-l.html
>> =
>
>
> --
> Esta mensagem foi verificada pelo sistema de antivírus e
> acredita-se estar livre de perigo.

-- 
Esta mensagem foi verificada pelo sistema de antiv�rus e
 acredita-se estar livre de perigo.


=
Instru��es para entrar na lista, sair da lista e usar a lista em
http://www.mat.puc-rio.br/~obmlistas/obm-l.html
=


Re: [obm-l] Magnitude

2021-04-25 Por tôpico Pedro Lazéra
Boa noite, Maikel.

A quantidade de algarismos de um número x (na base 10) é "1 + piso de
log(x)", em que "log" é a função logaritmo na base 10. Você pode verificar
isso assim: 10^n, para n inteiro >= 0, é o menor número do mundo com n+1
algarismos. Além disso, log(10^n) = n. Por fim, log é uma função crescente.

Usando que log(a*b) = log(a) + log(b), uma solução computeira é fazer um
programa que calcula log(i), para i inteiro entre 1 e 100, e soma esses
valores a uma variável s inicializada com o valor ZERO.

Uma solução computeira (provavelmente) errada é calcular x = 100! e depois
achar o log(x). Esse valor x não cabe nas estruturas de dados que a maioria
das linguagens usa para representar números.

Bom, a soma dá 157.97, conforme o Anderson Torres falou antes de mim. A
gente sabe que os computadores até erram (truncam) o valor exato de log(x),
mas o erro é bem pequeno e só estamos somando 100 aplicações da função log,
daí sabemos que esse 157.97 pode até estar errado, mas é por muito pouco
(menos do que 0,01, por exemplo).

Finalmente, 100! tem 1 + piso(157.97) = 158 algarismos.

Abraços,
Pedro

On Sun, Apr 11, 2021 at 12:29 AM Anderson Torres <
torres.anderson...@gmail.com> wrote:

> Em sáb., 3 de abr. de 2021 às 01:13, Maikel Andril Marcelino
>  escreveu:
> >
> > Quantos algarismos tem o número (100!) ?
>
> Em outras palavras, qual é o log(100!)/log(10). O Google me diz que
> isso é 157,97 - logo, 158 dígitos.
>
> >
> >
> > Atenciosamente,
> >
> > Maikel Andril Marcelino
> > Assistente de Aluno - Biblioteca - Ramal: 7616
> > Coordenadoria de Apoio Acadêmico - COAPAC/IFRN-SPP
> > Instituto Federal do Rio Grande do Norte
> > Campus São Paulo do Potengi
> >
> > +55 (84) 8851-3451
> >
> > --
> > Esta mensagem foi verificada pelo sistema de antivírus e
> > acredita-se estar livre de perigo.
>
> --
> Esta mensagem foi verificada pelo sistema de antivírus e
>  acredita-se estar livre de perigo.
>
>
> =
> Instru�ões para entrar na lista, sair da lista e usar a lista em
> http://www.mat.puc-rio.br/~obmlistas/obm-l.html
> =
>

-- 
Esta mensagem foi verificada pelo sistema de antiv�rus e
 acredita-se estar livre de perigo.



Re: [obm-l]

2021-04-25 Por tôpico Artur Costa Steiner
Oh, no meu email anterior, onde se
lê raiz(3), leia-se raiz_cúbica(2). Tô fazendo um tratamento na vista e
ando com dificuldade para digitar num celular.
Um cara de 69 anos como eu não deveria mais participar deste grupo

Artur



Em dom., 25 de abr. de 2021 14:16, Artur Costa Steiner <
artur.costa.stei...@gmail.com> escreveu:

> raiz(2)) e raiz(3) são inteiros algébricos, visto serem raízes de x^2 - 2
> e x^3 - 2, respectivamente. Segundo um clássico teorema da Teoria dos
> Números, a soma de dois inteiros algébricos é inteira algébrica. E um
> inteiro algébrico é racional se, e somente se, for inteiro. Como, conforme
> já comentado, este não é o caso de raiz(2) + raiz(3), segue-se que está
> soma é irracional.
>
> Abraços
> Artur
>
>
> Em sex., 23 de abr. de 2021 17:43, Marcos Martinelli <
> mffmartine...@gmail.com> escreveu:
>
>> Legal, Matheus.
>>
>> Minha ideia foi encontrar um polinômio em m.n (m = raiz(2) e
>> n=raiz_cúbica(2)) de coeficientes racionais. Pra isso desenvolvi m^k + n^k
>> (k >= 0) até k=6 e encontrei um de grau 6 com coeficientes dependendo só de
>> m+n.
>>
>> Se m+n for racional, usei o fato de se a + beta (a racional e beta
>> irracional com beta^j  também irracional (1=< j <= grau do polinômio- 1)
>> for raiz desse polinômio então a - beta também seria.
>>
>> Mas essa sua ficou bem elegante.
>>
>> Brigado.
>>
>> Em sex., 23 de abr. de 2021 às 17:18, Matheus Secco <
>> matheusse...@gmail.com> escreveu:
>>
>>> Oi, Marcos. Não é difícil verificar que raiz(2) + raiz_cubica(2) é uma
>>> raiz do polinômio x^6 - 6 x^4 - 4 x^3 + 12 x^2 - 24 x - 4. Com isso, pelo
>>> teorema das raízes racionais, se raiz(2) + raiz_cubica(2) fosse racional,
>>> teria que ser um inteiro e é fácil verificar que 2 < raiz(2) +
>>> raiz_cubica(2) < 3.
>>>
>>> Abraços
>>>
>>> On Fri, Apr 23, 2021 at 4:43 PM Marcos Martinelli <
>>> mffmartine...@gmail.com> wrote:
>>>
 Opa, pessoal. Pensei nos últimos dias no problema seguinte. Cheguei a
 uma solução um pouco mais genérica, mas me deu trabalho. Gostaria de
 estudar outras abordagens.

 Problema) Prove que raiz (2) + raiz_cúbica (2) é irracional.

 Na sequência posto um rascunho do que pensei.

 Obrigado.

 --
 Esta mensagem foi verificada pelo sistema de antivírus e
 acredita-se estar livre de perigo.
>>>
>>>
>>> --
>>> Esta mensagem foi verificada pelo sistema de antivírus e
>>> acredita-se estar livre de perigo.
>>
>>
>> --
>> Esta mensagem foi verificada pelo sistema de antivírus e
>> acredita-se estar livre de perigo.
>
>

-- 
Esta mensagem foi verificada pelo sistema de antiv�rus e
 acredita-se estar livre de perigo.



Re: [obm-l]

2021-04-25 Por tôpico Artur Costa Steiner
raiz(2)) e raiz(3) são inteiros algébricos, visto serem raízes de x^2 - 2 e
x^3 - 2, respectivamente. Segundo um clássico teorema da Teoria dos
Números, a soma de dois inteiros algébricos é inteira algébrica. E um
inteiro algébrico é racional se, e somente se, for inteiro. Como, conforme
já comentado, este não é o caso de raiz(2) + raiz(3), segue-se que está
soma é irracional.

Abraços
Artur


Em sex., 23 de abr. de 2021 17:43, Marcos Martinelli <
mffmartine...@gmail.com> escreveu:

> Legal, Matheus.
>
> Minha ideia foi encontrar um polinômio em m.n (m = raiz(2) e
> n=raiz_cúbica(2)) de coeficientes racionais. Pra isso desenvolvi m^k + n^k
> (k >= 0) até k=6 e encontrei um de grau 6 com coeficientes dependendo só de
> m+n.
>
> Se m+n for racional, usei o fato de se a + beta (a racional e beta
> irracional com beta^j  também irracional (1=< j <= grau do polinômio- 1)
> for raiz desse polinômio então a - beta também seria.
>
> Mas essa sua ficou bem elegante.
>
> Brigado.
>
> Em sex., 23 de abr. de 2021 às 17:18, Matheus Secco <
> matheusse...@gmail.com> escreveu:
>
>> Oi, Marcos. Não é difícil verificar que raiz(2) + raiz_cubica(2) é uma
>> raiz do polinômio x^6 - 6 x^4 - 4 x^3 + 12 x^2 - 24 x - 4. Com isso, pelo
>> teorema das raízes racionais, se raiz(2) + raiz_cubica(2) fosse racional,
>> teria que ser um inteiro e é fácil verificar que 2 < raiz(2) +
>> raiz_cubica(2) < 3.
>>
>> Abraços
>>
>> On Fri, Apr 23, 2021 at 4:43 PM Marcos Martinelli <
>> mffmartine...@gmail.com> wrote:
>>
>>> Opa, pessoal. Pensei nos últimos dias no problema seguinte. Cheguei a
>>> uma solução um pouco mais genérica, mas me deu trabalho. Gostaria de
>>> estudar outras abordagens.
>>>
>>> Problema) Prove que raiz (2) + raiz_cúbica (2) é irracional.
>>>
>>> Na sequência posto um rascunho do que pensei.
>>>
>>> Obrigado.
>>>
>>> --
>>> Esta mensagem foi verificada pelo sistema de antivírus e
>>> acredita-se estar livre de perigo.
>>
>>
>> --
>> Esta mensagem foi verificada pelo sistema de antivírus e
>> acredita-se estar livre de perigo.
>
>
> --
> Esta mensagem foi verificada pelo sistema de antivírus e
> acredita-se estar livre de perigo.

-- 
Esta mensagem foi verificada pelo sistema de antiv�rus e
 acredita-se estar livre de perigo.



[obm-l] Re: [obm-l] RES: [obm-l] Re: [obm-l] INFLAÇÂO MÁXIMA

2021-04-24 Por tôpico Pedro Júnior
Sobre a gramática, verdade!
Sobre a Matemática, impecável.
Consegui ver onde eu estava errando.

Obrigado, professor.

Em sex, 23 de abr de 2021 14:24,  escreveu:

> Olá!
>
> Para começar, esta questão deveria ter sido anulada. “… não HAJAM perdas
> reais?” é um assassinato da nossa língua.
>
>
>
> Juros “reais” (JR), de 10%, significam juros acima da inflação (IF).
>
> No período de 1 ano, o ganho bruto de capital (GB) será: GB = 1.000
> (1+10%)(1+IF) - 1.000
>
> Descontando o imposto, o ganho líquido (GL) será: GL =  (1-40%)GB
>
> Condição de contorno: não “hão” (coerência com o linguajar da questão)
> perdas reais: 1.000(1+IF) = 1.000+GL = 1.000 + (1-40%)GB = 1.000 + (1-40%)(
> 1.000 (1+10%)(1+IF) - 1.000 )
>
> Daí: IF = 17,6470…%
>
>
>
> *Albert Bouskelá*
>
> bousk...@gmail.com
>
>
>
> *De:* owner-ob...@mat.puc-rio.br  *Em nome de
> *Daniel Jelin
> *Enviada em:* sexta-feira, 23 de abril de 2021 12:30
> *Para:* obm-l@mat.puc-rio.br
> *Assunto:* [obm-l] Re: [obm-l] INFLAÇÂO MÁXIMA
>
>
>
> Curioso, pra mim deu muito perto, 17,6470...%
>
> Resolvi a seguinte inequação, com x = 1 + (inflação):
>
> 1.1*1000x - (1.1*1000x - 1000)*0.4>=1000x
> 1.1 x - 0.44 x + 0.4 >= x
> x<=0.4/0.34= 1.176470...
>
> Parece simples. O que tá escapando aqui?
>
>
>
> On Fri, Apr 23, 2021 at 11:23 AM Pedro Júnior 
> wrote:
>
> Olá pessoal, acabei me enrolando nesse probleminha da Olimpíada Brasileira
> de Economia. Será que alguém pode me ajudar? Vai junto o gabarito da
> competição, isso foi em 2020.
>
>
>
> *01)* Um título comprado por mil reais promete pagar juros reais de 10%
> a.a. A alíquota de imposto é de 40%. Qual a inflação máxima no período para
> que não hajam perdas reais?
>
> Resp.: 17,62%
>
>
>
> --
>
> Pedro Jerônimo S. de O. Júnior
>
> Professor de Matemática
>
>
>
>
> --
> Esta mensagem foi verificada pelo sistema de antivírus e
> acredita-se estar livre de perigo.
>
>
> --
> Esta mensagem foi verificada pelo sistema de antiv?s e
> acredita-se estar livre de perigo.
>
> --
> Esta mensagem foi verificada pelo sistema de antivírus e
> acredita-se estar livre de perigo.
>

-- 
Esta mensagem foi verificada pelo sistema de antiv�rus e
 acredita-se estar livre de perigo.



Re: [obm-l] Probabilidade

2021-04-23 Por tôpico Pacini Bores
 

Obrigado Ralph pela explicação didática. 

Ficou esclarecida a minha dúvida 

Abraços 

Pacini 

Em 23/04/2021 16:59, Ralph Costa Teixeira escreveu: 

> Ah, Pacini, você levanta um ponto interessante... 
> 
> Primeiro, deixa eu esclarecer: eu usei p(n) = Pr (A vai vencer o jogo | A tem 
> n pontos a mais do que B agora); ou seja, não seria exatamente o que você 
> interpretou ali. 
> 
> Daqui meu argumento de simetria: a partir do momento em que A tem 0 pontos a 
> mais do que B, ou seja, eles estão empatados, o jogo é completamente 
> simétrico, ou seja, eu posso permutar A e B sem alterar nenhuma 
> probabilidade. Por isso eu digo que: 
> 
> p(0) = Pr (A vencer | empatados agora) = Pr (B vencer | empatados agora) 
> 
> Aqui entra o seu ponto interessante: É POSSÍVEL QUE ESTE JOGO CONTINUE PARA 
> SEMPRE, SEM QUE HAJA VENCEDOR. De fato, se os lançamentos a partir de agora 
> forem CKCKCKCK..., o jogo nunca termina. 
> 
> Entao eu deveria escrever Pr (A vencer | empatados agora) + Pr (B vencer | 
> empatados agora) + Pr (jogo nunca terminar | empatados agora) = 1. Para eu 
> poder afirmar que os dois primeiros termos valem 1/2, **eu tenho que te 
> convencer primeiro que o terceiro termo vale 0**. 
> 
> Bom, vale 0 sim, mas eu usei isso baseado em experiência prévia com este tipo 
> de experimento; por exemplo, sei que: 
> 
> ---///--- 
> LEMA: Lance uma moeda infinitas vezes, onde cada lançamento é independente 
> dos outros e tem probabilidade p de dar "Cara" e 1-p de dar "Koroa", com 
> 0 momento da sequência é 1. 
> 
> PROVA: Escreva "sucesso" = "obter N caras consecutivas", e "fracasso" = "nao 
> obter N caras consecutivas". Temos: 
> Pr (fracasso nos lançamentos 1 a N) = 1-p^N = a, onde 0 Pr (fracasso nos lançamentos N+1 a 2N) = a. 
> Pr (fracasso nos lançamentos 2N+1 a 3N) = a. 
> ... 
> Pois bem, fracasso na sequência toda IMPLICA fracasso em cada uma das 
> subsequências que escolhi acima. Como tomei sequências disjuntas de 
> lançamentos, posso multiplicar tudo e obter: 
> Pr (fracasso nos lançamentos de 1 a kN) <= a^k. 
> 
> Quando k->Inf, isso vai para 0, portanto a probabilidade de fracasso nos 
> "infinitos" lançamentos vale 0. 
> ---///--- 
> 
> O que isso tem a ver com nosso problema? No nosso problema, note que se 
> tivermos 7 lances consecutivos onde A marca ponto mas B não (deixa eu chamar 
> isso de "cara"), certamente A vai vencer em algum momento desta sequência. 
> 
> Assim, "jogo nunca terminar" IMPLICA "nunca existe uma sequência de 7 caras". 
> Portanto: 
> Pr (jogo não terminar) <= Pr(nunca ter sequência com 7 "caras") = 0 
> e assim eu posso completar o argumento que eu usei, afirmando que p(0)=1/2. 
> Ufa! 
> 
> (Note que este argumento vale mesmo no caso em que cada "lance" tem 4 opções 
> (1,0); (0,1); (0,0); (1,1) para o número de pontos que A e B ganham; aqui 
> teríamos p("cara")=1/4, continua valendo!) 
> 
> ---///--- 
> 
> Enfim, antes que alguém estranhe isso, deixa eu explicitar algo que pode 
> parecer estranho: 
> -- SIM, é possível que o jogo nunca termine... 
> -- ...e a probabilidade disso acontecer vale 0. 
> Os axiomas da probabilidade dizem que Pr(vazio)=0; SE um evento é impossível 
> ENTÃO ele tem probabilidade 0. Mas nunca dizem a volta disso! Podemos ter 
> Pr(A)=0 sem ter A=vazio nem impossível! Eventos POSSÍVEIS podem ter 
> probabilidade 0 sim senhor. 
> Exemplo simples: jogando uma moeda justa infinitas vezes, qual a 
> probabilidade de todas as vezes darem cara? Reposta: ZERO. PODE acontecer... 
> mas, huh, eu não apostaria nisso. :D 
> Pior: eventos de probabilidade 0 ACONTECEM. Exemplo: jogue a moeda infinitas 
> vezes, anote a sequência exata que saiu, na ordem. A probabilidade de sair 
> exatamente esta sequência era ZERO antes de você fazer o experimento... mas 
> aconteceu. :P 
> 
> On Fri, Apr 23, 2021 at 9:48 AM Pacini Bores  wrote: 
> 
> Desculpe Ralph, 
> 
> O que não ficou claro pra mim foi o fato de que p(0) =1/2 , já que p(0) 
> traduz a probabilidade de de ficar com diferença de zero ponto agora ou 
> depois, ou seja, partindo de zero ponto de diferença entre os dois jogadores, 
> poderia ficar assim a vida toda, não ? Em que estou pensando errado. 
> 
> Agradeço desde já ( acho que tenho que estudar mais) 
> 
> Pacini 
> 
> Em 03/04/2021 18:08, Ralph Costa Teixeira escreveu: 
> 
> Vou dizer que "o jogo está na posicao n" quando A tem n pontos de vantagem; e 
> vou chamar de p(n) a probabilidade de A vencer o jogo (agora ou depois) 
> sabendo que (agora) A tem n pontos mais do que B. 
> 
> Por exemplo, p(3)=1, p(-3)=0 e p(0)=1/2 (por simetria). 
> 
> Aliás, por simetria, vemos que p(1)=1-p(-1) e p(2)=1-p(-2). Vou chamar a=p(1) 
> e b=p(2) para facilitar a escrita (o "p(n)" seria util para jogos maiores, 
> quando a gente escreveria tudo em forma matricial -- mas aqui nem vou 
> precisar). 
> 
> A partir da posicao 1, no próximo "lance", temos 50% de chance de ir para 2 
> (e dali chance b de A ganhar) e 50% de chance de ir para 0 

Re: [obm-l]

2021-04-23 Por tôpico Marcos Martinelli
Legal, Matheus.

Minha ideia foi encontrar um polinômio em m.n (m = raiz(2) e
n=raiz_cúbica(2)) de coeficientes racionais. Pra isso desenvolvi m^k + n^k
(k >= 0) até k=6 e encontrei um de grau 6 com coeficientes dependendo só de
m+n.

Se m+n for racional, usei o fato de se a + beta (a racional e beta
irracional com beta^j  também irracional (1=< j <= grau do polinômio- 1)
for raiz desse polinômio então a - beta também seria.

Mas essa sua ficou bem elegante.

Brigado.

Em sex., 23 de abr. de 2021 às 17:18, Matheus Secco 
escreveu:

> Oi, Marcos. Não é difícil verificar que raiz(2) + raiz_cubica(2) é uma
> raiz do polinômio x^6 - 6 x^4 - 4 x^3 + 12 x^2 - 24 x - 4. Com isso, pelo
> teorema das raízes racionais, se raiz(2) + raiz_cubica(2) fosse racional,
> teria que ser um inteiro e é fácil verificar que 2 < raiz(2) +
> raiz_cubica(2) < 3.
>
> Abraços
>
> On Fri, Apr 23, 2021 at 4:43 PM Marcos Martinelli 
> wrote:
>
>> Opa, pessoal. Pensei nos últimos dias no problema seguinte. Cheguei a uma
>> solução um pouco mais genérica, mas me deu trabalho. Gostaria de estudar
>> outras abordagens.
>>
>> Problema) Prove que raiz (2) + raiz_cúbica (2) é irracional.
>>
>> Na sequência posto um rascunho do que pensei.
>>
>> Obrigado.
>>
>> --
>> Esta mensagem foi verificada pelo sistema de antivírus e
>> acredita-se estar livre de perigo.
>
>
> --
> Esta mensagem foi verificada pelo sistema de antivírus e
> acredita-se estar livre de perigo.

-- 
Esta mensagem foi verificada pelo sistema de antiv�rus e
 acredita-se estar livre de perigo.



[obm-l] Re: [obm-l] Re: [obm-l] Função

2021-04-23 Por tôpico Israel Meireles Chrisostomo
Obrigado

Em qui, 22 de abr de 2021 11:25, Artur Costa Steiner <
artur.costa.stei...@gmail.com> escreveu:

> O que vc disse só vale para funções contínuas de R em R. No domínio
> complexo, não vale.
> Nos complexos, uma função inteira é injetora se, e somente se, for um
> mapeamento afim não constante, caso em que é bijetora.
>
> Artur
>
>
> Em qui., 22 de abr. de 2021 07:19, Israel Meireles Chrisostomo <
> israelmchrisost...@gmail.com> escreveu:
>
>> Me desculpem se eu estou falando bobagem, mas considere uma função com
>> domínio complexo, então essa função não pode ser bijetora, pois toda função
>> bijetora ou é crescente ou é decrescente, mas não há ordem nos complexos
>>
>> --
>> Esta mensagem foi verificada pelo sistema de antivírus e
>> acredita-se estar livre de perigo.
>
>
> --
> Esta mensagem foi verificada pelo sistema de antivírus e
> acredita-se estar livre de perigo.

-- 
Esta mensagem foi verificada pelo sistema de antiv�rus e
 acredita-se estar livre de perigo.



Re: [obm-l]

2021-04-23 Por tôpico Matheus Secco
Oi, Marcos. Não é difícil verificar que raiz(2) + raiz_cubica(2) é uma raiz
do polinômio x^6 - 6 x^4 - 4 x^3 + 12 x^2 - 24 x - 4. Com isso, pelo
teorema das raízes racionais, se raiz(2) + raiz_cubica(2) fosse racional,
teria que ser um inteiro e é fácil verificar que 2 < raiz(2) +
raiz_cubica(2) < 3.

Abraços

On Fri, Apr 23, 2021 at 4:43 PM Marcos Martinelli 
wrote:

> Opa, pessoal. Pensei nos últimos dias no problema seguinte. Cheguei a uma
> solução um pouco mais genérica, mas me deu trabalho. Gostaria de estudar
> outras abordagens.
>
> Problema) Prove que raiz (2) + raiz_cúbica (2) é irracional.
>
> Na sequência posto um rascunho do que pensei.
>
> Obrigado.
>
> --
> Esta mensagem foi verificada pelo sistema de antivírus e
> acredita-se estar livre de perigo.

-- 
Esta mensagem foi verificada pelo sistema de antiv�rus e
 acredita-se estar livre de perigo.



Re: [obm-l] Probabilidade

2021-04-23 Por tôpico Ralph Costa Teixeira
Ah, Pacini, você levanta um ponto interessante...

Primeiro, deixa eu esclarecer: eu usei p(n) = Pr (A vai vencer o jogo | A
tem n pontos a mais do que B agora); ou seja, não seria exatamente o que
você interpretou ali.

Daqui meu argumento de simetria: a partir do momento em que A tem 0 pontos
a mais do que B, ou seja, eles estão empatados, o jogo é completamente
simétrico, ou seja, eu posso permutar A e B sem alterar nenhuma
probabilidade. Por isso eu digo que:

p(0) = Pr (A vencer | empatados agora) = Pr (B vencer | empatados agora)

Aqui entra o seu ponto interessante: É POSSÍVEL QUE ESTE JOGO CONTINUE PARA
SEMPRE, SEM QUE HAJA VENCEDOR. De fato, se os lançamentos a partir de agora
forem CKCKCKCK..., o jogo nunca termina.

Entao eu deveria escrever Pr (A vencer | empatados agora) + Pr (B vencer |
empatados agora) + Pr (jogo nunca terminar | empatados agora) = 1. Para eu
poder afirmar que os dois primeiros termos valem 1/2, **eu tenho que te
convencer primeiro que o terceiro termo vale 0**.

Bom, vale 0 sim, mas eu usei isso baseado em experiência prévia com este
tipo de experimento; por exemplo, sei que:

---///---
LEMA: Lance uma moeda infinitas vezes, onde cada lançamento é independente
dos outros e tem probabilidade p de dar "Cara" e 1-p de dar "Koroa", com
0Inf, isso vai para 0, portanto a probabilidade de fracasso nos
"infinitos" lançamentos vale 0.
---///---

O que isso tem a ver com nosso problema? No nosso problema, note que se
tivermos 7 lances consecutivos onde A marca ponto mas B não (deixa eu
chamar isso de "cara"), certamente A vai vencer em algum momento desta
sequência.

Assim, "jogo nunca terminar" IMPLICA "nunca existe uma sequência de 7
caras". Portanto:
Pr (jogo não terminar) <= Pr(nunca ter sequência com 7 "caras") = 0
e assim eu posso completar o argumento que eu usei, afirmando que p(0)=1/2.
Ufa!

(Note que este argumento vale mesmo no caso em que cada "lance" tem 4
opções (1,0); (0,1); (0,0); (1,1) para o número de pontos que A e B ganham;
aqui teríamos p("cara")=1/4, continua valendo!)

---///---

Enfim, antes que alguém estranhe isso, deixa eu explicitar algo que pode
parecer estranho:
-- SIM, é possível que o jogo nunca termine...
-- ...e a probabilidade disso acontecer vale 0.
Os axiomas da probabilidade dizem que Pr(vazio)=0; SE um evento é
impossível ENTÃO ele tem probabilidade 0. Mas nunca dizem a volta disso!
Podemos ter Pr(A)=0 sem ter A=vazio nem impossível! Eventos POSSÍVEIS podem
ter probabilidade 0 sim senhor.
Exemplo simples: jogando uma moeda justa infinitas vezes, qual a
probabilidade de todas as vezes darem cara? Reposta: ZERO. PODE
acontecer... mas, huh, eu não apostaria nisso. :D
Pior: eventos de probabilidade 0 ACONTECEM. Exemplo: jogue a moeda
infinitas vezes, anote a sequência exata que saiu, na ordem. A
probabilidade de sair exatamente esta sequência era ZERO antes de você
fazer o experimento... mas aconteceu. :P





On Fri, Apr 23, 2021 at 9:48 AM Pacini Bores  wrote:

> Desculpe  Ralph,
>
> O que não ficou claro pra mim  foi o fato de que p(0) =1/2 , já que p(0)
> traduz a probabilidade de de ficar com diferença de  zero ponto  agora ou
> depois, ou seja, partindo de zero ponto de diferença entre os dois
> jogadores, poderia ficar assim a vida toda, não ? Em que estou pensando
> errado.
>
> Agradeço desde já ( acho que tenho que estudar mais)
>
> Pacini
>
> Em 03/04/2021 18:08, Ralph Costa Teixeira escreveu:
>
> Vou dizer que "o jogo está na posicao n" quando A tem n pontos de
> vantagem; e vou chamar de p(n) a probabilidade de A vencer o jogo (agora ou
> depois) sabendo que (agora) A tem n pontos mais do que B.
>
> Por exemplo, p(3)=1, p(-3)=0 e p(0)=1/2 (por simetria).
>
> Aliás, por simetria, vemos que p(1)=1-p(-1) e p(2)=1-p(-2). Vou chamar
> a=p(1) e b=p(2) para facilitar a escrita (o "p(n)" seria util para jogos
> maiores, quando a gente escreveria tudo em forma matricial -- mas aqui nem
> vou precisar).
>
> A partir da posicao 1, no próximo "lance", temos 50% de chance de ir para
> 2 (e dali chance b de A ganhar) e 50% de chance de ir para 0 (e dali chance
> 50% de A ganhar). Portanto:
>
> a= 1/2 . b + 1/2. 1/2
>
> Analogamente, a partir de 2, temos 50% de chance de ir para 1 e 50% de
> chance de termos vitória de A, portanto:
>
> b=1/2 + 1/2.a
>
> Resolvendo o sistema, vem a=2/3  e b = 5/6. Resposta (B)?
>
> Abraco, Ralph.
>
> P.S.: Em geral seria : p(n)=p_A . p(n+1) + (1-p_A) . p(n-1), e as regras
> sobre a vitória determinam "condições de contorno". Ou seja, considerando o
> vetor v = (p(-m), p(-m+1), ... p(0), ... p(m)), temos v=Mv onde M é uma
> matriz tridiagonal (de fato, com 0s na diagonal). Ou seja, no fundo no
> fundo estamos falando de um problema de achar o autovetor associado ao
> autovalor 1 da matriz M, e as condicoes de contorno apenas normalizam v.
>
>
>
>
> On Sat, Apr 3, 2021 at 3:22 PM Pacini Bores 
> wrote:
>
>> Olá pessoal, Encontrei uma resposta que não está entre as opções desta
>> questão do Canguru.
>>
>> " um certo 

[obm-l] RES: [obm-l] Re: [obm-l] INFLAÇÂO MÁXIMA

2021-04-23 Por tôpico bouskela
Olá!

Para começar, esta questão deveria ter sido anulada. “… não HAJAM perdas 
reais?” é um assassinato da nossa língua.

 

Juros “reais” (JR), de 10%, significam juros acima da inflação (IF).

No período de 1 ano, o ganho bruto de capital (GB) será: GB = 1.000 
(1+10%)(1+IF) - 1.000

Descontando o imposto, o ganho líquido (GL) será: GL =  (1-40%)GB

Condição de contorno: não “hão” (coerência com o linguajar da questão) perdas 
reais: 1.000(1+IF) = 1.000+GL = 1.000 + (1-40%)GB = 1.000 + (1-40%)( 1.000 
(1+10%)(1+IF) - 1.000 )

Daí: IF = 17,6470…%

 

Albert Bouskelá

 <mailto:bousk...@gmail.com> bousk...@gmail.com

 

De: owner-ob...@mat.puc-rio.br  Em nome de Daniel 
Jelin
Enviada em: sexta-feira, 23 de abril de 2021 12:30
Para: obm-l@mat.puc-rio.br
Assunto: [obm-l] Re: [obm-l] INFLAÇÂO MÁXIMA

 

Curioso, pra mim deu muito perto, 17,6470...%

Resolvi a seguinte inequação, com x = 1 + (inflação):

1.1*1000x - (1.1*1000x - 1000)*0.4>=1000x
1.1 x - 0.44 x + 0.4 >= x
x<=0.4/0.34= 1.176470... 

Parece simples. O que tá escapando aqui?

 

On Fri, Apr 23, 2021 at 11:23 AM Pedro Júnior mailto:pedromatematic...@gmail.com> > wrote:

Olá pessoal, acabei me enrolando nesse probleminha da Olimpíada Brasileira de 
Economia. Será que alguém pode me ajudar? Vai junto o gabarito da competição, 
isso foi em 2020.

 

01) Um título comprado por mil reais promete pagar juros reais de 10% a.a. A 
alíquota de imposto é de 40%. Qual a inflação máxima no período para que não 
hajam perdas reais?

Resp.: 17,62%


 

-- 

Pedro Jerônimo S. de O. Júnior

Professor de Matemática

 


-- 
Esta mensagem foi verificada pelo sistema de antivírus e 
acredita-se estar livre de perigo. 


-- 
Esta mensagem foi verificada pelo sistema de antiv�s e 
acredita-se estar livre de perigo. 


-- 
Esta mensagem foi verificada pelo sistema de antiv�rus e
 acredita-se estar livre de perigo.



Re: [obm-l] Probabilidade

2021-04-23 Por tôpico Pacini Bores
 

Desculpe Ralph, 

O que não ficou claro pra mim foi o fato de que p(0) =1/2 , já que p(0)
traduz a probabilidade de de ficar com diferença de zero ponto agora ou
depois, ou seja, partindo de zero ponto de diferença entre os dois
jogadores, poderia ficar assim a vida toda, não ? Em que estou pensando
errado. 

Agradeço desde já ( acho que tenho que estudar mais) 

Pacini 

Em 03/04/2021 18:08, Ralph Costa Teixeira escreveu: 

> Vou dizer que "o jogo está na posicao n" quando A tem n pontos de vantagem; e 
> vou chamar de p(n) a probabilidade de A vencer o jogo (agora ou depois) 
> sabendo que (agora) A tem n pontos mais do que B. 
> 
> Por exemplo, p(3)=1, p(-3)=0 e p(0)=1/2 (por simetria). 
> 
> Aliás, por simetria, vemos que p(1)=1-p(-1) e p(2)=1-p(-2). Vou chamar a=p(1) 
> e b=p(2) para facilitar a escrita (o "p(n)" seria util para jogos maiores, 
> quando a gente escreveria tudo em forma matricial -- mas aqui nem vou 
> precisar). 
> 
> A partir da posicao 1, no próximo "lance", temos 50% de chance de ir para 2 
> (e dali chance b de A ganhar) e 50% de chance de ir para 0 (e dali chance 50% 
> de A ganhar). Portanto: 
> 
> a= 1/2 . b + 1/2. 1/2 
> 
> Analogamente, a partir de 2, temos 50% de chance de ir para 1 e 50% de chance 
> de termos vitória de A, portanto: 
> 
> b=1/2 + 1/2.a 
> 
> Resolvendo o sistema, vem a=2/3 e b = 5/6. Resposta (B)? 
> 
> Abraco, Ralph. 
> 
> P.S.: Em geral seria : p(n)=p_A . p(n+1) + (1-p_A) . p(n-1), e as regras 
> sobre a vitória determinam "condições de contorno". Ou seja, considerando o 
> vetor v = (p(-m), p(-m+1), ... p(0), ... p(m)), temos v=Mv onde M é uma 
> matriz tridiagonal (de fato, com 0s na diagonal). Ou seja, no fundo no fundo 
> estamos falando de um problema de achar o autovetor associado ao autovalor 1 
> da matriz M, e as condicoes de contorno apenas normalizam v.
> 
> On Sat, Apr 3, 2021 at 3:22 PM Pacini Bores  wrote: 
> 
>> Olá pessoal, Encontrei uma resposta que não está entre as opções desta 
>> questão do Canguru. 
>> 
>> " um certo jogo tem um vencedor quando este atinge 3 pontos a frente do 
>> oponente. Dois jogadores A e B estão jogando e, num determinado momento, A 
>> está 1 ponto a frente de B. Os jogadores têm probabilidades iguais de obter 
>> 1 ponto. Qual a probabilidade de A vencer o jogo ? 
>> 
>> (A) 1/2 (B) 2/3 (C) 3/4 (D) 4/5 (E) 5/6 
>> 
>> O que vocês acham ? 
>> 
>> Pacini 
>> 
>> -- 
>> Esta mensagem foi verificada pelo sistema de antivírus e 
>> acredita-se estar livre de perigo.
> 
> -- 
> Esta mensagem foi verificada pelo sistema de antivrus e 
> acredita-se estar livre de perigo.

 
-- 
Esta mensagem foi verificada pelo sistema de antiv�rus e
 acredita-se estar livre de perigo.



[obm-l] Re: [obm-l] INFLAÇÂO MÁXIMA

2021-04-23 Por tôpico Daniel Jelin
Curioso, pra mim deu muito perto, 17,6470...%

Resolvi a seguinte inequação, com x = 1 + (inflação):

1.1*1000x - (1.1*1000x - 1000)*0.4>=1000x
1.1 x - 0.44 x + 0.4 >= x
x<=0.4/0.34= 1.176470...

Parece simples. O que tá escapando aqui?

On Fri, Apr 23, 2021 at 11:23 AM Pedro Júnior 
wrote:

> Olá pessoal, acabei me enrolando nesse probleminha da Olimpíada Brasileira
> de Economia. Será que alguém pode me ajudar? Vai junto o gabarito da
> competição, isso foi em 2020.
>
> *01)* Um título comprado por mil reais promete pagar juros reais de 10%
> a.a. A alíquota de imposto é de 40%. Qual a inflação máxima no período para
> que não hajam perdas reais?
> Resp.: 17,62%
>
> --
>
> Pedro Jerônimo S. de O. Júnior
>
> Professor de Matemática
>
>
>
> --
> Esta mensagem foi verificada pelo sistema de antivírus e
> acredita-se estar livre de perigo.

-- 
Esta mensagem foi verificada pelo sistema de antiv�rus e
 acredita-se estar livre de perigo.



[obm-l] Re: [obm-l] Função

2021-04-22 Por tôpico Artur Costa Steiner
O que vc disse só vale para funções contínuas de R em R. No domínio
complexo, não vale.
Nos complexos, uma função inteira é injetora se, e somente se, for um
mapeamento afim não constante, caso em que é bijetora.

Artur


Em qui., 22 de abr. de 2021 07:19, Israel Meireles Chrisostomo <
israelmchrisost...@gmail.com> escreveu:

> Me desculpem se eu estou falando bobagem, mas considere uma função com
> domínio complexo, então essa função não pode ser bijetora, pois toda função
> bijetora ou é crescente ou é decrescente, mas não há ordem nos complexos
>
> --
> Esta mensagem foi verificada pelo sistema de antivírus e
> acredita-se estar livre de perigo.

-- 
Esta mensagem foi verificada pelo sistema de antiv�rus e
 acredita-se estar livre de perigo.



[obm-l] Re: [obm-l] Função

2021-04-22 Por tôpico Eduardo Henrique Rodrigues do Nascimento
Cara, toda função real contínua e bijetora é monótona. Como contraexemplo
se f não for contínua:

  x+1 para x no intervalo [0,1[
f(x)={x, para x≥2 e x<0
  x-1 para x no intervalo [1,2[

então f não é crescente em todo o seu domínio: 1/2<3/2; mas
f(1/2)=3/2>1/2=f(3/2).

 além disso, a função complexa f(z)=z é claramente bijetora.

Em qui, 22 de abr de 2021 07:19, Israel Meireles Chrisostomo <
israelmchrisost...@gmail.com> escreveu:

> Me desculpem se eu estou falando bobagem, mas considere uma função com
> domínio complexo, então essa função não pode ser bijetora, pois toda função
> bijetora ou é crescente ou é decrescente, mas não há ordem nos complexos
>
> --
> Esta mensagem foi verificada pelo sistema de antivírus e
> acredita-se estar livre de perigo.

-- 
Esta mensagem foi verificada pelo sistema de antiv�rus e
 acredita-se estar livre de perigo.



[obm-l] Re: [obm-l] Re: [obm-l] Questão sobre desigualdades

2021-04-14 Por tôpico Carlos Monteiro
De onde saiu essa desigualdade?

Em qua., 14 de abr. de 2021 às 20:39, Anderson Torres <
torres.anderson...@gmail.com> escreveu:

> Em qua., 14 de abr. de 2021 às 15:54, Carlos Monteiro
>  escreveu:
> >
> > Encontre os valores máximo e mínimo da expressão:  x/(x^2+1) + y/(y^2+1)
> + z/(z^2+1) , onde x, y e z são números reais que satisfazem x+y+z = 1.
> >
> >
>
> Verifica-se que 3(12x+1)/50 >= x/(x^2+1), e assim o valor máximo é 3/10
>
> >
> >
> > --
> > Esta mensagem foi verificada pelo sistema de antivírus e
> > acredita-se estar livre de perigo.
>
> --
> Esta mensagem foi verificada pelo sistema de antivírus e
>  acredita-se estar livre de perigo.
>
>
> =
> Instru�ões para entrar na lista, sair da lista e usar a lista em
> http://www.mat.puc-rio.br/~obmlistas/obm-l.html
> =
>

-- 
Esta mensagem foi verificada pelo sistema de antiv�rus e
 acredita-se estar livre de perigo.



[obm-l] Re: [obm-l] Questão sobre desigualdades

2021-04-14 Por tôpico Anderson Torres
Em qua., 14 de abr. de 2021 às 15:54, Carlos Monteiro
 escreveu:
>
> Encontre os valores máximo e mínimo da expressão:  x/(x^2+1) + y/(y^2+1) + 
> z/(z^2+1) , onde x, y e z são números reais que satisfazem x+y+z = 1.
>
>

Verifica-se que 3(12x+1)/50 >= x/(x^2+1), e assim o valor máximo é 3/10

>
>
> --
> Esta mensagem foi verificada pelo sistema de antivírus e
> acredita-se estar livre de perigo.

-- 
Esta mensagem foi verificada pelo sistema de antiv�rus e
 acredita-se estar livre de perigo.


=
Instru��es para entrar na lista, sair da lista e usar a lista em
http://www.mat.puc-rio.br/~obmlistas/obm-l.html
=


Re: [obm-l] transcendencia

2021-04-10 Por tôpico Anderson Torres
Em qui., 1 de abr. de 2021 às 18:02, Israel Meireles Chrisostomo
 escreveu:
>
>
> Como posso provar que se u é um número transcendente e a_k são números 
> algébricos quaisquer, para todo k natural, então  ua_0+ ua_1+ ua_2+...+ ua_n 
> não pode ser igual a zero.

Fatorando U.

> --
> Israel Meireles Chrisostomo
>
> --
> Esta mensagem foi verificada pelo sistema de antivírus e
> acredita-se estar livre de perigo.

-- 
Esta mensagem foi verificada pelo sistema de antiv�rus e
 acredita-se estar livre de perigo.


=
Instru��es para entrar na lista, sair da lista e usar a lista em
http://www.mat.puc-rio.br/~obmlistas/obm-l.html
=


Re: [obm-l] Magnitude

2021-04-10 Por tôpico Anderson Torres
Em sáb., 3 de abr. de 2021 às 01:13, Maikel Andril Marcelino
 escreveu:
>
> Quantos algarismos tem o número (100!) ?

Em outras palavras, qual é o log(100!)/log(10). O Google me diz que
isso é 157,97 - logo, 158 dígitos.

>
>
> Atenciosamente,
>
> Maikel Andril Marcelino
> Assistente de Aluno - Biblioteca - Ramal: 7616
> Coordenadoria de Apoio Acadêmico - COAPAC/IFRN-SPP
> Instituto Federal do Rio Grande do Norte
> Campus São Paulo do Potengi
>
> +55 (84) 8851-3451
>
> --
> Esta mensagem foi verificada pelo sistema de antivírus e
> acredita-se estar livre de perigo.

-- 
Esta mensagem foi verificada pelo sistema de antiv�rus e
 acredita-se estar livre de perigo.


=
Instru��es para entrar na lista, sair da lista e usar a lista em
http://www.mat.puc-rio.br/~obmlistas/obm-l.html
=


[obm-l] Re: [obm-l] Algébricos

2021-04-10 Por tôpico Anderson Torres
Em seg., 5 de abr. de 2021 às 21:57, Israel Meireles Chrisostomo
 escreveu:
>
> O número i é algebricamente dependente de pi?
>

O que é algebricamente dependente?

> --
> Esta mensagem foi verificada pelo sistema de antivírus e
> acredita-se estar livre de perigo.

-- 
Esta mensagem foi verificada pelo sistema de antiv�rus e
 acredita-se estar livre de perigo.


=
Instru��es para entrar na lista, sair da lista e usar a lista em
http://www.mat.puc-rio.br/~obmlistas/obm-l.html
=


Re: [obm-l] Probabilidade

2021-04-09 Por tôpico Ralph Costa Teixeira
Primeiro: sim, Albert tem razão, eu assumi que em cada rodada apenas um
entre A e B marcariam pontos, portanto ignorei os casos (A=B), e nada dizia
isso claramente no enunciado.

Mas a conta do Daniel revela que não importa, o que é bem interessante
E, agora, depois de ver a conta, digo: era de se esperar! Afinal, no jogo
"tipo Bouskela", as rodadas onde A e B marcam pontos juntos podem ser
jogadas fora, pois o fim do jogo é determinado por quantos pontos um
jogador tem A MAIS do que o outro, e tais rodadas não tem efeito nenhum
nisso. Como estas rodadas do tipo A=B podem ser jogadas fora, o jogo "tipo
Bouskela" é de fato equivalente ao jogo "tipo Ralph" que eu analisei (bom,
pelo menos com relação a determinar QUEM ganha; se a gente perguntasse algo
do tipo "QUANDO" ganha, os jogos seriam bem distintos).

(Outra coisa: eu tinha dito que achar essas probabilidades era equivalente
a achar os autovetores de uma certa matriz M; note que a minha matriz M tem
probabilidades de transição entre estados. Se a gente incluir as
"transições tipo Bouskela" no jogo, a gente de fato estah colocando algumas
probabilidades p na diagonal principal, e re-escalando correspondentemente
as probabilidades do jogo do "tipo Ralph". Ou seja, estamos trocando M por
X=(1-p).M+p.I. Mas M e X=(1-p).M+pI tem os mesmos autovetores, que eh a
maneira "Algebra Linear" de explicar porque a resposta não muda! :D :D )



On Sat, Apr 10, 2021 at 1:19 AM Daniel Jelin  wrote:

> Me parece que a interpretação dada não muda a resposta, se entendi
> direito. Teríamos: 50% de chance de continuar na mesma posição (ponto pros
> dois ou ponto pra ninguém), 25% de avançar (ponto pra um), 25% de recuar
> (ponto pro adversário). Assim, acho que dá para usar o esquema do Ralph:
> a=(1/4)*b+(1/4)*(1/2)+(1/2)*a
> b=(1/4)+(1/4)*a+(1/2)*b
> E resolvendo, temos os mesmos a=2/3 e b=5/6.
>
> Ainda que as probabilidades de fazer e de não fazer o ponto fossem
> diferentes, creio que dá na mesma. Seja x a probabilidade de A fazer 1
> ponto, então, pelo enunciado, x também é a probabilidade de B fazer 1
> ponto. Aí a probabilidade de A não fazer ponto é 1-x, e a de B não fazer
> ponto são os mesmos 1-x. Então:
>
> a=(x)*(1-x)*b + (1-x)*(x)*1/2 + (x)(x)*a+(1-x)*(1-x)*a
> b=(x)*(1-x) + (1-x)*(x)*a + (x)(x)*b+(1-x)*(1-x)*b
> E resolvendo, eliminamos x e voltamos a a=2/3 e b=5/6.
>
> On Thu, Apr 8, 2021 at 8:27 PM  wrote:
>
>> Este é um problema bastante interessante, contudo o seu enunciado, tal
>> como está, apresenta uma falha: - É necessário fixar quais são os
>> resultados possíveis numa determinada rodada do jogo! Dito assim, o
>> enunciado admite, para cada rodada 4 possibilidades: (A=1, B=1); (A=1,
>> B=0); (A=0, B=1); (A=0, B=0).
>>
>>
>>
>> *Albert Bouskelá*
>>
>> bousk...@gmail.com
>>
>>
>>
>> *De:* owner-ob...@mat.puc-rio.br  *Em nome
>> de *Professor Vanderlei Nemitz
>> *Enviada em:* quinta-feira, 8 de abril de 2021 14:34
>> *Para:* OBM 
>> *Assunto:* Re: [obm-l] Probabilidade
>>
>>
>>
>> Muito legal esse tipo de problema.
>>
>> Em que ano caiu, você sabe, Pacini?
>>
>>
>>
>> Em sáb., 3 de abr. de 2021 às 15:22, Pacini Bores 
>> escreveu:
>>
>> Olá pessoal, Encontrei uma resposta que não está entre as opções desta
>> questão do Canguru.
>>
>> " um certo jogo tem um vencedor quando este atinge 3 pontos a frente do
>> oponente. Dois jogadores A e B estão jogando e, num determinado momento, A
>> está 1 ponto a frente de B. Os jogadores  têm probabilidades iguais de
>> obter 1 ponto. Qual a probabilidade de A vencer o jogo ?
>>
>> (A) 1/2   (B) 2/3  (C) 3/4   (D) 4/5  (E) 5/6
>>
>>
>>
>> O que vocês acham ?
>>
>>  Pacini
>>
>>
>>
>>
>> --
>> Esta mensagem foi verificada pelo sistema de antivírus e
>> acredita-se estar livre de perigo.
>>
>>
>> --
>> Esta mensagem foi verificada pelo sistema de antiv?s e
>> acredita-se estar livre de perigo.
>>
>> --
>> Esta mensagem foi verificada pelo sistema de antivírus e
>> acredita-se estar livre de perigo.
>>
>
>
> <https://www.avast.com/sig-email?utm_medium=email_source=link_campaign=sig-email_content=webmail>
>  Livre
> de vírus. www.avast.com
> <https://www.avast.com/sig-email?utm_medium=email_source=link_campaign=sig-email_content=webmail>.
> <#m_-7953325398812603413_DAB4FAD8-2DD7-40BB-A1B8-4E2AA1F9FDF2>
>
> --
> Esta mensagem foi verificada pelo sistema de antivírus e
> acredita-se estar livre de perigo.

-- 
Esta mensagem foi verificada pelo sistema de antiv�rus e
 acredita-se estar livre de perigo.



Re: [obm-l] Probabilidade

2021-04-09 Por tôpico Daniel Jelin
Me parece que a interpretação dada não muda a resposta, se entendi direito.
Teríamos: 50% de chance de continuar na mesma posição (ponto pros dois ou
ponto pra ninguém), 25% de avançar (ponto pra um), 25% de recuar (ponto pro
adversário). Assim, acho que dá para usar o esquema do Ralph:
a=(1/4)*b+(1/4)*(1/2)+(1/2)*a
b=(1/4)+(1/4)*a+(1/2)*b
E resolvendo, temos os mesmos a=2/3 e b=5/6.

Ainda que as probabilidades de fazer e de não fazer o ponto fossem
diferentes, creio que dá na mesma. Seja x a probabilidade de A fazer 1
ponto, então, pelo enunciado, x também é a probabilidade de B fazer 1
ponto. Aí a probabilidade de A não fazer ponto é 1-x, e a de B não fazer
ponto são os mesmos 1-x. Então:

a=(x)*(1-x)*b + (1-x)*(x)*1/2 + (x)(x)*a+(1-x)*(1-x)*a
b=(x)*(1-x) + (1-x)*(x)*a + (x)(x)*b+(1-x)*(1-x)*b
E resolvendo, eliminamos x e voltamos a a=2/3 e b=5/6.

On Thu, Apr 8, 2021 at 8:27 PM  wrote:

> Este é um problema bastante interessante, contudo o seu enunciado, tal
> como está, apresenta uma falha: - É necessário fixar quais são os
> resultados possíveis numa determinada rodada do jogo! Dito assim, o
> enunciado admite, para cada rodada 4 possibilidades: (A=1, B=1); (A=1,
> B=0); (A=0, B=1); (A=0, B=0).
>
>
>
> *Albert Bouskelá*
>
> bousk...@gmail.com
>
>
>
> *De:* owner-ob...@mat.puc-rio.br  *Em nome de
> *Professor Vanderlei Nemitz
> *Enviada em:* quinta-feira, 8 de abril de 2021 14:34
> *Para:* OBM 
> *Assunto:* Re: [obm-l] Probabilidade
>
>
>
> Muito legal esse tipo de problema.
>
> Em que ano caiu, você sabe, Pacini?
>
>
>
> Em sáb., 3 de abr. de 2021 às 15:22, Pacini Bores 
> escreveu:
>
> Olá pessoal, Encontrei uma resposta que não está entre as opções desta
> questão do Canguru.
>
> " um certo jogo tem um vencedor quando este atinge 3 pontos a frente do
> oponente. Dois jogadores A e B estão jogando e, num determinado momento, A
> está 1 ponto a frente de B. Os jogadores  têm probabilidades iguais de
> obter 1 ponto. Qual a probabilidade de A vencer o jogo ?
>
> (A) 1/2   (B) 2/3  (C) 3/4   (D) 4/5  (E) 5/6
>
>
>
> O que vocês acham ?
>
>  Pacini
>
>
>
>
> --
> Esta mensagem foi verificada pelo sistema de antivírus e
> acredita-se estar livre de perigo.
>
>
> --
> Esta mensagem foi verificada pelo sistema de antiv?s e
> acredita-se estar livre de perigo.
>
> --
> Esta mensagem foi verificada pelo sistema de antivírus e
> acredita-se estar livre de perigo.
>

<https://www.avast.com/sig-email?utm_medium=email_source=link_campaign=sig-email_content=webmail>
Livre
de vírus. www.avast.com
<https://www.avast.com/sig-email?utm_medium=email_source=link_campaign=sig-email_content=webmail>.
<#DAB4FAD8-2DD7-40BB-A1B8-4E2AA1F9FDF2>

-- 
Esta mensagem foi verificada pelo sistema de antiv�rus e
 acredita-se estar livre de perigo.



Re: [obm-l] Probabilidade

2021-04-09 Por tôpico Pacini Bores
 

Acredito que foi este ano. Passaram pra mim desta forma. 

Pacini 

Em 08/04/2021 14:33, Professor Vanderlei Nemitz escreveu: 

> Muito legal esse tipo de problema. 
> Em que ano caiu, você sabe, Pacini? 
> 
> Em sáb., 3 de abr. de 2021 às 15:22, Pacini Bores  
> escreveu: 
> 
>> Olá pessoal, Encontrei uma resposta que não está entre as opções desta 
>> questão do Canguru. 
>> 
>> " um certo jogo tem um vencedor quando este atinge 3 pontos a frente do 
>> oponente. Dois jogadores A e B estão jogando e, num determinado momento, A 
>> está 1 ponto a frente de B. Os jogadores têm probabilidades iguais de obter 
>> 1 ponto. Qual a probabilidade de A vencer o jogo ? 
>> 
>> (A) 1/2 (B) 2/3 (C) 3/4 (D) 4/5 (E) 5/6 
>> 
>> O que vocês acham ? 
>> 
>> Pacini 
>> 
>> -- 
>> Esta mensagem foi verificada pelo sistema de antivírus e 
>> acredita-se estar livre de perigo.
> 
> -- 
> Esta mensagem foi verificada pelo sistema de antivrus e 
> acredita-se estar livre de perigo.

 
-- 
Esta mensagem foi verificada pelo sistema de antiv�rus e
 acredita-se estar livre de perigo.



Re: [obm-l] Probabilidade

2021-04-08 Por tôpico Professor Vanderlei Nemitz
Muito legal esse tipo de problema.
Em que ano caiu, você sabe, Pacini?

Em sáb., 3 de abr. de 2021 às 15:22, Pacini Bores 
escreveu:

> Olá pessoal, Encontrei uma resposta que não está entre as opções desta
> questão do Canguru.
>
> " um certo jogo tem um vencedor quando este atinge 3 pontos a frente do
> oponente. Dois jogadores A e B estão jogando e, num determinado momento, A
> está 1 ponto a frente de B. Os jogadores  têm probabilidades iguais de
> obter 1 ponto. Qual a probabilidade de A vencer o jogo ?
>
> (A) 1/2   (B) 2/3  (C) 3/4   (D) 4/5  (E) 5/6
>
>
>
> O que vocês acham ?
>
>  Pacini
>
>
> --
> Esta mensagem foi verificada pelo sistema de antivírus e
> acredita-se estar livre de perigo.
>

-- 
Esta mensagem foi verificada pelo sistema de antiv�rus e
 acredita-se estar livre de perigo.



Re: [obm-l] Magnitude

2021-04-07 Por tôpico qedtexte

Sauda,c~oes,

Com o WA, 158.

Lus





Data: 03/04/2021
De: Maikel Andril Marcelino maikel.marcel...@ifrn.edu.br
Para: obm-l@mat.puc-rio.br
Assunto: [obm-l] Magnitude Quantos algarismos tem o nmero (100!) ? 



Atenciosamente, 

Maikel Andril Marcelino 
Assistente de Aluno - Biblioteca - Ramal: 7616 
Coordenadoria de Apoio Acadmico - COAPAC/IFRN-SPP 
Instituto Federal do Rio Grande do Norte 
Campus So Paulo do Potengi 

+55 (84) 8851-3451 


--
Esta mensagem foi verificada pelo sistema de antivrus e 
acredita-se estar livre de perigo.

--
Esta mensagem foi verificada pelo sistema de antiv�rus e
acredita-se estar livre de perigo.



Re: [obm-l] Probabilidade

2021-04-03 Por tôpico Pacini Bores
 

Obrigado Ralph 

Abraços 

Em 03/04/2021 18:08, Ralph Costa Teixeira escreveu: 

> Vou dizer que "o jogo está na posicao n" quando A tem n pontos de vantagem; e 
> vou chamar de p(n) a probabilidade de A vencer o jogo (agora ou depois) 
> sabendo que (agora) A tem n pontos mais do que B. 
> 
> Por exemplo, p(3)=1, p(-3)=0 e p(0)=1/2 (por simetria). 
> 
> Aliás, por simetria, vemos que p(1)=1-p(-1) e p(2)=1-p(-2). Vou chamar a=p(1) 
> e b=p(2) para facilitar a escrita (o "p(n)" seria util para jogos maiores, 
> quando a gente escreveria tudo em forma matricial -- mas aqui nem vou 
> precisar). 
> 
> A partir da posicao 1, no próximo "lance", temos 50% de chance de ir para 2 
> (e dali chance b de A ganhar) e 50% de chance de ir para 0 (e dali chance 50% 
> de A ganhar). Portanto: 
> 
> a= 1/2 . b + 1/2. 1/2 
> 
> Analogamente, a partir de 2, temos 50% de chance de ir para 1 e 50% de chance 
> de termos vitória de A, portanto: 
> 
> b=1/2 + 1/2.a 
> 
> Resolvendo o sistema, vem a=2/3 e b = 5/6. Resposta (B)? 
> 
> Abraco, Ralph. 
> 
> P.S.: Em geral seria : p(n)=p_A . p(n+1) + (1-p_A) . p(n-1), e as regras 
> sobre a vitória determinam "condições de contorno". Ou seja, considerando o 
> vetor v = (p(-m), p(-m+1), ... p(0), ... p(m)), temos v=Mv onde M é uma 
> matriz tridiagonal (de fato, com 0s na diagonal). Ou seja, no fundo no fundo 
> estamos falando de um problema de achar o autovetor associado ao autovalor 1 
> da matriz M, e as condicoes de contorno apenas normalizam v.
> 
> On Sat, Apr 3, 2021 at 3:22 PM Pacini Bores  wrote: 
> 
>> Olá pessoal, Encontrei uma resposta que não está entre as opções desta 
>> questão do Canguru. 
>> 
>> " um certo jogo tem um vencedor quando este atinge 3 pontos a frente do 
>> oponente. Dois jogadores A e B estão jogando e, num determinado momento, A 
>> está 1 ponto a frente de B. Os jogadores têm probabilidades iguais de obter 
>> 1 ponto. Qual a probabilidade de A vencer o jogo ? 
>> 
>> (A) 1/2 (B) 2/3 (C) 3/4 (D) 4/5 (E) 5/6 
>> 
>> O que vocês acham ? 
>> 
>> Pacini 
>> 
>> -- 
>> Esta mensagem foi verificada pelo sistema de antivírus e 
>> acredita-se estar livre de perigo.
> 
> -- 
> Esta mensagem foi verificada pelo sistema de antivrus e 
> acredita-se estar livre de perigo.

 
-- 
Esta mensagem foi verificada pelo sistema de antiv�rus e
 acredita-se estar livre de perigo.



Re: [obm-l] Probabilidade

2021-04-03 Por tôpico Ralph Costa Teixeira
Vou dizer que "o jogo está na posicao n" quando A tem n pontos de vantagem;
e vou chamar de p(n) a probabilidade de A vencer o jogo (agora ou depois)
sabendo que (agora) A tem n pontos mais do que B.

Por exemplo, p(3)=1, p(-3)=0 e p(0)=1/2 (por simetria).

Aliás, por simetria, vemos que p(1)=1-p(-1) e p(2)=1-p(-2). Vou chamar
a=p(1) e b=p(2) para facilitar a escrita (o "p(n)" seria util para jogos
maiores, quando a gente escreveria tudo em forma matricial -- mas aqui nem
vou precisar).

A partir da posicao 1, no próximo "lance", temos 50% de chance de ir para 2
(e dali chance b de A ganhar) e 50% de chance de ir para 0 (e dali chance
50% de A ganhar). Portanto:

a= 1/2 . b + 1/2. 1/2

Analogamente, a partir de 2, temos 50% de chance de ir para 1 e 50% de
chance de termos vitória de A, portanto:

b=1/2 + 1/2.a

Resolvendo o sistema, vem a=2/3  e b = 5/6. Resposta (B)?

Abraco, Ralph.

P.S.: Em geral seria : p(n)=p_A . p(n+1) + (1-p_A) . p(n-1), e as regras
sobre a vitória determinam "condições de contorno". Ou seja, considerando o
vetor v = (p(-m), p(-m+1), ... p(0), ... p(m)), temos v=Mv onde M é uma
matriz tridiagonal (de fato, com 0s na diagonal). Ou seja, no fundo no
fundo estamos falando de um problema de achar o autovetor associado ao
autovalor 1 da matriz M, e as condicoes de contorno apenas normalizam v.




On Sat, Apr 3, 2021 at 3:22 PM Pacini Bores  wrote:

> Olá pessoal, Encontrei uma resposta que não está entre as opções desta
> questão do Canguru.
>
> " um certo jogo tem um vencedor quando este atinge 3 pontos a frente do
> oponente. Dois jogadores A e B estão jogando e, num determinado momento, A
> está 1 ponto a frente de B. Os jogadores  têm probabilidades iguais de
> obter 1 ponto. Qual a probabilidade de A vencer o jogo ?
>
> (A) 1/2   (B) 2/3  (C) 3/4   (D) 4/5  (E) 5/6
>
>
>
> O que vocês acham ?
>
>  Pacini
>
>
> --
> Esta mensagem foi verificada pelo sistema de antivírus e
> acredita-se estar livre de perigo.
>

-- 
Esta mensagem foi verificada pelo sistema de antiv�rus e
 acredita-se estar livre de perigo.



Re: [obm-l] Re: transcendencia

2021-04-02 Por tôpico Carlos Gustavo Tamm de Araujo Moreira
Por outro lado, se v é algébrico e u é algébrico sobre o corpo Q(v) então u
é algébrico.
O meu exemplo é um pouco "roubado": parece que b satisfaz a equação
(a^2-2)b+a(a^2-2)=0, mas, como
a^2-2=0, essa equação é identicamente nula...
Abraços,
 Gugu

On Fri, Apr 2, 2021 at 4:57 PM Israel Meireles Chrisostomo <
israelmchrisost...@gmail.com> wrote:

> Muito obrigado professor gugu
>
> Em sex, 2 de abr de 2021 16:00, Carlos Gustavo Tamm de Araujo Moreira <
> g...@impa.br> escreveu:
>
>> Não. Se a=sqrt(2) e b=pi então a^3+b.a^2-2a-2b=0, por exemplo.
>>
>> Em sex, 2 de abr de 2021 15:31, Israel Meireles Chrisostomo <
>> israelmchrisost...@gmail.com> escreveu:
>>
>>>   Se u é um número transcendente e v é um número, se u,v são
>>> algebricamente dependentes então v é transcendente?
>>>
>>>
>>> Em sex., 2 de abr. de 2021 às 14:58, Israel Meireles Chrisostomo <
>>> israelmchrisost...@gmail.com> escreveu:
>>>
 Se a é um número transcendente e v é um número, se u,v são
 algebricamente dependentes então v é transcendente?

 --
 Israel Meireles Chrisostomo

>>>
>>>
>>> --
>>> Israel Meireles Chrisostomo
>>>
>>> --
>>> Esta mensagem foi verificada pelo sistema de antivírus e
>>> acredita-se estar livre de perigo.
>>
>>
>> --
>> Esta mensagem foi verificada pelo sistema de antivírus e
>> acredita-se estar livre de perigo.
>
>
> --
> Esta mensagem foi verificada pelo sistema de antivírus e
> acredita-se estar livre de perigo.

-- 
Esta mensagem foi verificada pelo sistema de antiv�rus e
 acredita-se estar livre de perigo.



Re: [obm-l] Re: transcendencia

2021-04-02 Por tôpico Israel Meireles Chrisostomo
Muito obrigado professor gugu

Em sex, 2 de abr de 2021 16:00, Carlos Gustavo Tamm de Araujo Moreira <
g...@impa.br> escreveu:

> Não. Se a=sqrt(2) e b=pi então a^3+b.a^2-2a-2b=0, por exemplo.
>
> Em sex, 2 de abr de 2021 15:31, Israel Meireles Chrisostomo <
> israelmchrisost...@gmail.com> escreveu:
>
>>   Se u é um número transcendente e v é um número, se u,v são
>> algebricamente dependentes então v é transcendente?
>>
>>
>> Em sex., 2 de abr. de 2021 às 14:58, Israel Meireles Chrisostomo <
>> israelmchrisost...@gmail.com> escreveu:
>>
>>> Se a é um número transcendente e v é um número, se u,v são
>>> algebricamente dependentes então v é transcendente?
>>>
>>> --
>>> Israel Meireles Chrisostomo
>>>
>>
>>
>> --
>> Israel Meireles Chrisostomo
>>
>> --
>> Esta mensagem foi verificada pelo sistema de antivírus e
>> acredita-se estar livre de perigo.
>
>
> --
> Esta mensagem foi verificada pelo sistema de antivírus e
> acredita-se estar livre de perigo.

-- 
Esta mensagem foi verificada pelo sistema de antiv�rus e
 acredita-se estar livre de perigo.



Re: [obm-l] Re: transcendencia

2021-04-02 Por tôpico Carlos Gustavo Tamm de Araujo Moreira
Não. Se a=sqrt(2) e b=pi então a^3+b.a^2-2a-2b=0, por exemplo.

Em sex, 2 de abr de 2021 15:31, Israel Meireles Chrisostomo <
israelmchrisost...@gmail.com> escreveu:

>   Se u é um número transcendente e v é um número, se u,v são
> algebricamente dependentes então v é transcendente?
>
>
> Em sex., 2 de abr. de 2021 às 14:58, Israel Meireles Chrisostomo <
> israelmchrisost...@gmail.com> escreveu:
>
>> Se a é um número transcendente e v é um número, se u,v são algebricamente
>> dependentes então v é transcendente?
>>
>> --
>> Israel Meireles Chrisostomo
>>
>
>
> --
> Israel Meireles Chrisostomo
>
> --
> Esta mensagem foi verificada pelo sistema de antivírus e
> acredita-se estar livre de perigo.

-- 
Esta mensagem foi verificada pelo sistema de antiv�rus e
 acredita-se estar livre de perigo.



Re: [obm-l] Artigo

2021-04-02 Por tôpico Pedro Júnior
Boa discussão!

Em ter, 30 de mar de 2021 17:16, Israel Meireles Chrisostomo <
israelmchrisost...@gmail.com> escreveu:

> Obrigado
>
> Em ter., 30 de mar. de 2021 às 16:20, Daniel Jelin 
> escreveu:
>
>> não sei ao certo, meu caro, mas, falando como professor (e leitor),
>> suponho que não. e não é tanto por ser muito ou pouco avançado. receio que
>> o assunto fuja às preocupações do ensino básico - mesmo que a sua prova
>> seja elementar. repara, nada contra provas matemáticas na escola, ao
>> contrário. acho importante mostrar para os alunos de onde vêm os teoremas,
>> claro, mas: apenas das propriedades que eles de fato usam; e apenas as
>> demonstrações que eles têm condição de acompanhar do princípio ao fim, sem
>> que isso se torne um fardo adicional. será o caso?
>>
>> claro, algumas provas podem interessar ao professor mesmo que ele não
>> tenha a intenção de levá-la a todos os alunos. a irracionalidade de pi,
>> talvez, pra ficar no mesmo campo. leria com gosto uma investigação sobre a
>> irracionalidade de pi, passo a passo, com as armas da matemática do ensino
>> médio. tem isso? sei lá eu. mas a transcendência de pi? que tipo de
>> questão, que problema (escolar) esbarra na transcendência de pi? uma
>> sugestão: se vc puder mostrar que o professor deveria, sim, se importar com
>> isso, que questões importantes passam por aí, opa, então beleza, aí fica
>> mto legal, aí tem tudo a ver.
>>
>> abs
>>
>> On Tue, Mar 30, 2021 at 2:14 PM Israel Meireles Chrisostomo <
>> israelmchrisost...@gmail.com> wrote:
>>
>>> Vcs acham que a revista RPM aceitaria uma prova para transcendência de
>>> pi, ou isso é algo avançado demais para revista?
>>>
>>> --
>>> Israel Meireles Chrisostomo
>>>
>>> --
>>> Esta mensagem foi verificada pelo sistema de antivírus e
>>> acredita-se estar livre de perigo.
>>
>>
>> --
>> Esta mensagem foi verificada pelo sistema de antivírus e
>> acredita-se estar livre de perigo.
>
>
>
> --
> Israel Meireles Chrisostomo
>
> --
> Esta mensagem foi verificada pelo sistema de antivírus e
> acredita-se estar livre de perigo.

-- 
Esta mensagem foi verificada pelo sistema de antiv�rus e
 acredita-se estar livre de perigo.



Re: [obm-l] Transcendentes

2021-04-01 Por tôpico Esdras Muniz
Isso aí é falso, basta vc pegar a série de Taylor do seno por exemplo e
aplicar o π.

Em qui, 1 de abr de 2021 18:50, Israel Meireles Chrisostomo <
israelmchrisost...@gmail.com> escreveu:

> Como provar que se u é um número transcendentes e a_k são números
> algébricos, para tô natural k, então $u^{m_0}a_0 + u^{m_1}a_1 + u^{m_2}a_2
> + ... + u^{m_n}a_n $ não pode ser zero.onde $m_k$ é um inteiro positivo.
>
> --
> Esta mensagem foi verificada pelo sistema de antivírus e
> acredita-se estar livre de perigo.

-- 
Esta mensagem foi verificada pelo sistema de antiv�rus e
 acredita-se estar livre de perigo.



Re: [obm-l] Artigo

2021-03-30 Por tôpico Israel Meireles Chrisostomo
Obrigado

Em ter., 30 de mar. de 2021 às 16:20, Daniel Jelin 
escreveu:

> não sei ao certo, meu caro, mas, falando como professor (e leitor),
> suponho que não. e não é tanto por ser muito ou pouco avançado. receio que
> o assunto fuja às preocupações do ensino básico - mesmo que a sua prova
> seja elementar. repara, nada contra provas matemáticas na escola, ao
> contrário. acho importante mostrar para os alunos de onde vêm os teoremas,
> claro, mas: apenas das propriedades que eles de fato usam; e apenas as
> demonstrações que eles têm condição de acompanhar do princípio ao fim, sem
> que isso se torne um fardo adicional. será o caso?
>
> claro, algumas provas podem interessar ao professor mesmo que ele não
> tenha a intenção de levá-la a todos os alunos. a irracionalidade de pi,
> talvez, pra ficar no mesmo campo. leria com gosto uma investigação sobre a
> irracionalidade de pi, passo a passo, com as armas da matemática do ensino
> médio. tem isso? sei lá eu. mas a transcendência de pi? que tipo de
> questão, que problema (escolar) esbarra na transcendência de pi? uma
> sugestão: se vc puder mostrar que o professor deveria, sim, se importar com
> isso, que questões importantes passam por aí, opa, então beleza, aí fica
> mto legal, aí tem tudo a ver.
>
> abs
>
> On Tue, Mar 30, 2021 at 2:14 PM Israel Meireles Chrisostomo <
> israelmchrisost...@gmail.com> wrote:
>
>> Vcs acham que a revista RPM aceitaria uma prova para transcendência de
>> pi, ou isso é algo avançado demais para revista?
>>
>> --
>> Israel Meireles Chrisostomo
>>
>> --
>> Esta mensagem foi verificada pelo sistema de antivírus e
>> acredita-se estar livre de perigo.
>
>
> --
> Esta mensagem foi verificada pelo sistema de antivírus e
> acredita-se estar livre de perigo.



-- 
Israel Meireles Chrisostomo

-- 
Esta mensagem foi verificada pelo sistema de antiv�rus e
 acredita-se estar livre de perigo.



Re: [obm-l] Artigo

2021-03-30 Por tôpico Daniel Jelin
não sei ao certo, meu caro, mas, falando como professor (e leitor), suponho
que não. e não é tanto por ser muito ou pouco avançado. receio que o
assunto fuja às preocupações do ensino básico - mesmo que a sua prova seja
elementar. repara, nada contra provas matemáticas na escola, ao contrário.
acho importante mostrar para os alunos de onde vêm os teoremas, claro, mas:
apenas das propriedades que eles de fato usam; e apenas as demonstrações
que eles têm condição de acompanhar do princípio ao fim, sem que isso se
torne um fardo adicional. será o caso?

claro, algumas provas podem interessar ao professor mesmo que ele não tenha
a intenção de levá-la a todos os alunos. a irracionalidade de pi, talvez,
pra ficar no mesmo campo. leria com gosto uma investigação sobre a
irracionalidade de pi, passo a passo, com as armas da matemática do ensino
médio. tem isso? sei lá eu. mas a transcendência de pi? que tipo de
questão, que problema (escolar) esbarra na transcendência de pi? uma
sugestão: se vc puder mostrar que o professor deveria, sim, se importar com
isso, que questões importantes passam por aí, opa, então beleza, aí fica
mto legal, aí tem tudo a ver.

abs

On Tue, Mar 30, 2021 at 2:14 PM Israel Meireles Chrisostomo <
israelmchrisost...@gmail.com> wrote:

> Vcs acham que a revista RPM aceitaria uma prova para transcendência de pi,
> ou isso é algo avançado demais para revista?
>
> --
> Israel Meireles Chrisostomo
>
> --
> Esta mensagem foi verificada pelo sistema de antivírus e
> acredita-se estar livre de perigo.

-- 
Esta mensagem foi verificada pelo sistema de antiv�rus e
 acredita-se estar livre de perigo.



[obm-l] Re: [obm-l] Transcendência

2021-03-30 Por tôpico Anderson Torres
Em seg., 29 de mar. de 2021 às 21:07, Israel Meireles Chrisostomo
 escreveu:
>
> Estou desconfiado de um resultado, mas não sei como prová-lo.o resultado é o 
> seguinte: dados dois números a,b  transcendentes e algebricamente dependentes 
> e c um número, se a,b e c são algebricamente dependentes, então c é 
> transcendente.é verdade esse resultado?se sim, como posso prová-lo?
>

Como exatamente dois números são algebricamente dependentes?

Você parece estar perdendo tempo em algo.

> --
> Esta mensagem foi verificada pelo sistema de antivírus e
> acredita-se estar livre de perigo.

-- 
Esta mensagem foi verificada pelo sistema de antiv�rus e
 acredita-se estar livre de perigo.


=
Instru��es para entrar na lista, sair da lista e usar a lista em
http://www.mat.puc-rio.br/~obmlistas/obm-l.html
=


Re: [obm-l] Transcende

2021-03-30 Por tôpico Anderson Torres
Em seg., 29 de mar. de 2021 às 23:17, Israel Meireles Chrisostomo
 escreveu:
>
> Como provar que dados u
> algébrico e v transcendente, qualquer combinação linear racional de u e v, 
> também será transcendente.

Sério?

Combinações lineares de algébricos são algébricas.
Se você não sabe disso ainda, então tem que voltar a estudar.

E combinações lineares são transitivas, no sentido que se C é
combinação de A e B então A é combinação de B e C.

>
> --
> Esta mensagem foi verificada pelo sistema de antivírus e
> acredita-se estar livre de perigo.

-- 
Esta mensagem foi verificada pelo sistema de antiv�rus e
 acredita-se estar livre de perigo.


=
Instru��es para entrar na lista, sair da lista e usar a lista em
http://www.mat.puc-rio.br/~obmlistas/obm-l.html
=


Re: [obm-l] Trascendencia

2021-03-29 Por tôpico Israel Meireles Chrisostomo
Obrigado!

Em seg, 29 de mar de 2021 21:15, Carlos Gomes 
escreveu:

> Rapaz o melhor lugar em Portugues é a RPM online ou a Matemática
> universitária. Em inglês, mas bem concorrida é a American Mathematical
> Monthly.
>
> https://pmo.sbm.org.br/
> https://rmu.sbm.org.br/
> https://www.tandfonline.com/toc/uamm20/current
>
> Em seg., 29 de mar. de 2021 às 16:11, Israel Meireles Chrisostomo <
> israelmchrisost...@gmail.com> escreveu:
>
>>
>> Acho que consigo provar a transcendência de pi, como faço para publicá-la?
>> --
>> Israel Meireles Chrisostomo
>>
>> --
>> Esta mensagem foi verificada pelo sistema de antivírus e
>> acredita-se estar livre de perigo.
>
>
> --
> Esta mensagem foi verificada pelo sistema de antivírus e
> acredita-se estar livre de perigo.

-- 
Esta mensagem foi verificada pelo sistema de antiv�rus e
 acredita-se estar livre de perigo.



Re: [obm-l] Trascendencia

2021-03-29 Por tôpico Carlos Gomes
Rapaz o melhor lugar em Portugues é a RPM online ou a Matemática
universitária. Em inglês, mas bem concorrida é a American Mathematical
Monthly.

https://pmo.sbm.org.br/
https://rmu.sbm.org.br/
https://www.tandfonline.com/toc/uamm20/current

Em seg., 29 de mar. de 2021 às 16:11, Israel Meireles Chrisostomo <
israelmchrisost...@gmail.com> escreveu:

>
> Acho que consigo provar a transcendência de pi, como faço para publicá-la?
> --
> Israel Meireles Chrisostomo
>
> --
> Esta mensagem foi verificada pelo sistema de antivírus e
> acredita-se estar livre de perigo.

-- 
Esta mensagem foi verificada pelo sistema de antiv�rus e
 acredita-se estar livre de perigo.



Re: [obm-l] Provas anteriores

2021-03-11 Por tôpico Marcelo de Moura Costa
https://www.obm.org.br/como-se-preparar/provas-e-gabaritos/




Em qua., 10 de mar. de 2021 às 19:57, carlos h Souza 
escreveu:

> Onde posso baixar provas anteriores da obm?/
>
> --
> Esta mensagem foi verificada pelo sistema de antivírus e
> acredita-se estar livre de perigo.

-- 
Esta mensagem foi verificada pelo sistema de antiv�rus e
 acredita-se estar livre de perigo.



Re: [obm-l] Amigo secreto ENEM

2021-02-23 Por tôpico Luiz Antonio Rodrigues
Olá, Anderson!
Boa noite!
Vou consultar o Google.
Muito obrigado pela dica!
Luiz

Em ter, 23 de fev de 2021 10:55 AM, Anderson Torres <
torres.anderson...@gmail.com> escreveu:

> Em qui., 28 de jan. de 2021 às 13:15, Luiz Antonio Rodrigues
>  escreveu:
> >
> > Olá, pessoal!
> > Boa tarde!
> > Estou acompanhando com interesse a discussão, mas gostaria de pedir uma
> indicação de site ou outro material que trate de permutações caóticas.
>
> Procure por derangements no Google.
>
> > Muito obrigado!
> > Abraços!
> > Luiz
> >
> > Em qui, 28 de jan de 2021 11:38 AM, Arthur Queiroz <
> arthurqu...@gmail.com> escreveu:
> >>
> >> Uma pergunta: você assume que o número de sorteios é !10. Mas e se, em
> meio ao sorteio, nossa permutação caótica seja tal que seja formado um
> ciclo indesejado? Digamos A->B->C->A. Como o sorteio continuará nesse caso?
> Será escolhida aleatoriamente uma pessoa de fora do ciclo para continuar?
> Isso não afetaria esse !10?
> >>
> >> Em ter, 26 de jan de 2021 17:26, Ralph Costa Teixeira <
> ralp...@gmail.com> escreveu:
> >>>
> >>> Deixa eu copiar o que escrevi em outro lugar... :D :D
> >>>
> >>> Primeiro: não fica claro do enunciado se "auto-sorteios" (alguém
> sortear o próprio nome) são permitidos ou não, e isto ALTERA a resposta. :(
> >>>
> >>> Vejamos possíveis respostas corretas:
> >>>
> >>> ---///---
> >>>
> >>> SE AUTO-SORTEIOS FOREM PERMITIDOS:
> >>> Em resumo, temos 1/10 de chance de A iniciar o sorteio, e 1/10 de
> chance de B terminar (1/10 sim, pois A *pode* terminar). Assim, a resposta
> seria 1/10*1/10*2=1/50.
> >>>
> >>> Com mais detalhes para justificar o segundo "1/10":
> >>> -- Número de sorteios possíveis = 10!
> >>> -- Número de sorteios que formam um único ciclo de tamanho 10 = 9!
> >>> -- Note que ter um ciclo de tamanho 10 equivale a terminar com quem
> inicia; portanto, a chance de o amigo secreto terminar com o mesmo que
> iniciou seria 9!/10!=1/10 (que é independente de quem começa).
> >>>
> >>> Assim:
> >>> -- Chance de A iniciar = 1/10;
> >>> Agora, DADO QUE A INICIOU:
> >>>  Chance de A terminar = 9!/10! = 1/10
> >>>  Portanto, chance de não terminar com A: 9/10
> >>>  Chance de B terminar (por simetria): (9/10) /9 = 1/10
> >>>
> >>> Isso nos dá 1/10*1/10 = 1/100 de chance do amigo secreto começar por A
> e terminar com B. Portanto a resposta seria o dobro, 1/50.
> >>>
> >>> ---///---
> >>> SE AUTO-SORTEIOS SÃO PROIBIDOS:
> >>> -- Número de sorteios (desarranjos) possíveis = !10 (vou escrever
> K=!10 daqui por diante);
> >>> -- Número de sorteios que formam um único ciclo de tamanho 10 = 9!
> >>> -- Portanto, a chance de o amigo secreto terminar com o mesmo que
> iniciou seria 9!/K (que é independente de quem começa).
> >>>
> >>> Assim:
> >>> -- Chance de A iniciar = 1/10;
> >>> Agora, DADO QUE A INICIOU:
> >>>  Chance de A terminar = 9!/K
> >>>  Portanto, chance de não terminar com A: 1-9!/K
> >>>  Chance de B terminar (por simetria): (1-9!/K) /9 = (K-9!)/(9K)
> >>>
> >>> Isso nos dá 1/10* (K-9!)/(9K) = (K-9!)/(90K) de chance do amigo
> secreto começar por A e terminar com B. Portanto a resposta seria o dobro,
> (K-9!)/(45K). Fazendo a conta com a ajuda do computador, achei 12001/741645.
> >>>
> >>> Abraço, Ralph.
> >>>
> >>>
> >>>
> >>> On Tue, Jan 26, 2021 at 1:45 PM Professor Vanderlei Nemitz <
> vanderma...@gmail.com> wrote:
> 
>  Oi, pessoal!
> 
>  Com certeza vocês estão acompanhando desde domingo as resoluções da
> questão do ENEM do amigo secreto.
>  Além da resposta proposta, 1/45, que parece não estar correta, já vi
> outras duas, 12001/741645 (ETAPA e ANGLO), que consideram também que o
> sorteio anterior para definir "quem presenteia quem", e 7/360, do vídeo a
> seguir:
> 
>  https://www.youtube.com/watch?v=c-t_BAMASKE
> 
>  Gostaria da opinião (e se possível, uma resolução) dos especialistas
> da lista (Ralph e cia :))
> 
>  Muito obrigado!
> 
> 
> 
> 
>
> --
> Esta mensagem foi verificada pelo sistema de antivírus e
>  acredita-se estar livre de perigo.
>
>
> =
> Instru�ões para entrar na lista, sair da lista e usar a lista em
> http://www.mat.puc-rio.br/~obmlistas/obm-l.html
> =
>

-- 
Esta mensagem foi verificada pelo sistema de antiv�rus e
 acredita-se estar livre de perigo.



Re: [obm-l] Amigo secreto ENEM

2021-02-23 Por tôpico Anderson Torres
Em qui., 28 de jan. de 2021 às 13:15, Luiz Antonio Rodrigues
 escreveu:
>
> Olá, pessoal!
> Boa tarde!
> Estou acompanhando com interesse a discussão, mas gostaria de pedir uma 
> indicação de site ou outro material que trate de permutações caóticas.

Procure por derangements no Google.

> Muito obrigado!
> Abraços!
> Luiz
>
> Em qui, 28 de jan de 2021 11:38 AM, Arthur Queiroz  
> escreveu:
>>
>> Uma pergunta: você assume que o número de sorteios é !10. Mas e se, em meio 
>> ao sorteio, nossa permutação caótica seja tal que seja formado um ciclo 
>> indesejado? Digamos A->B->C->A. Como o sorteio continuará nesse caso? Será 
>> escolhida aleatoriamente uma pessoa de fora do ciclo para continuar? Isso 
>> não afetaria esse !10?
>>
>> Em ter, 26 de jan de 2021 17:26, Ralph Costa Teixeira  
>> escreveu:
>>>
>>> Deixa eu copiar o que escrevi em outro lugar... :D :D
>>>
>>> Primeiro: não fica claro do enunciado se "auto-sorteios" (alguém sortear o 
>>> próprio nome) são permitidos ou não, e isto ALTERA a resposta. :(
>>>
>>> Vejamos possíveis respostas corretas:
>>>
>>> ---///---
>>>
>>> SE AUTO-SORTEIOS FOREM PERMITIDOS:
>>> Em resumo, temos 1/10 de chance de A iniciar o sorteio, e 1/10 de chance de 
>>> B terminar (1/10 sim, pois A *pode* terminar). Assim, a resposta seria 
>>> 1/10*1/10*2=1/50.
>>>
>>> Com mais detalhes para justificar o segundo "1/10":
>>> -- Número de sorteios possíveis = 10!
>>> -- Número de sorteios que formam um único ciclo de tamanho 10 = 9!
>>> -- Note que ter um ciclo de tamanho 10 equivale a terminar com quem inicia; 
>>> portanto, a chance de o amigo secreto terminar com o mesmo que iniciou 
>>> seria 9!/10!=1/10 (que é independente de quem começa).
>>>
>>> Assim:
>>> -- Chance de A iniciar = 1/10;
>>> Agora, DADO QUE A INICIOU:
>>>  Chance de A terminar = 9!/10! = 1/10
>>>  Portanto, chance de não terminar com A: 9/10
>>>  Chance de B terminar (por simetria): (9/10) /9 = 1/10
>>>
>>> Isso nos dá 1/10*1/10 = 1/100 de chance do amigo secreto começar por A e 
>>> terminar com B. Portanto a resposta seria o dobro, 1/50.
>>>
>>> ---///---
>>> SE AUTO-SORTEIOS SÃO PROIBIDOS:
>>> -- Número de sorteios (desarranjos) possíveis = !10 (vou escrever K=!10 
>>> daqui por diante);
>>> -- Número de sorteios que formam um único ciclo de tamanho 10 = 9!
>>> -- Portanto, a chance de o amigo secreto terminar com o mesmo que iniciou 
>>> seria 9!/K (que é independente de quem começa).
>>>
>>> Assim:
>>> -- Chance de A iniciar = 1/10;
>>> Agora, DADO QUE A INICIOU:
>>>  Chance de A terminar = 9!/K
>>>  Portanto, chance de não terminar com A: 1-9!/K
>>>  Chance de B terminar (por simetria): (1-9!/K) /9 = (K-9!)/(9K)
>>>
>>> Isso nos dá 1/10* (K-9!)/(9K) = (K-9!)/(90K) de chance do amigo secreto 
>>> começar por A e terminar com B. Portanto a resposta seria o dobro, 
>>> (K-9!)/(45K). Fazendo a conta com a ajuda do computador, achei 12001/741645.
>>>
>>> Abraço, Ralph.
>>>
>>>
>>>
>>> On Tue, Jan 26, 2021 at 1:45 PM Professor Vanderlei Nemitz 
>>>  wrote:

 Oi, pessoal!

 Com certeza vocês estão acompanhando desde domingo as resoluções da 
 questão do ENEM do amigo secreto.
 Além da resposta proposta, 1/45, que parece não estar correta, já vi 
 outras duas, 12001/741645 (ETAPA e ANGLO), que consideram também que o 
 sorteio anterior para definir "quem presenteia quem", e 7/360, do vídeo a 
 seguir:

 https://www.youtube.com/watch?v=c-t_BAMASKE

 Gostaria da opinião (e se possível, uma resolução) dos especialistas da 
 lista (Ralph e cia :))

 Muito obrigado!





-- 
Esta mensagem foi verificada pelo sistema de antiv�rus e
 acredita-se estar livre de perigo.


=
Instru��es para entrar na lista, sair da lista e usar a lista em
http://www.mat.puc-rio.br/~obmlistas/obm-l.html
=


Re: [obm-l] teoria de conjuntos

2021-02-20 Por tôpico enzo souza
A pergunta não chegou deu algum erro de envio :/

Em sáb, 20 de fev de 2021 21:59, carlos h Souza 
escreveu:

>
>
> --
> Esta mensagem foi verificada pelo sistema de antivírus e
> acredita-se estar livre de perigo.

-- 
Esta mensagem foi verificada pelo sistema de antiv�rus e
 acredita-se estar livre de perigo.



[obm-l] Re: [obm-l] Re: [obm-l] Re: [obm-l] Re: [obm-l] Re: [obm-l] Re: [obm-l] Re: [obm-l] Re: Equações funcionais

2021-02-16 Por tôpico Anderson Torres
Em ter., 16 de fev. de 2021 às 21:26, joao pedro b menezes
 escreveu:
>
> Eu sei, temos f(-1)= 0, f(0) = 1, e f é bijetora. Após trabalhar a equação 
> que cheguei na expressão:
> f( x + f(x) ) - f( f(x)) = x.  Queria saber se essa identidade, junto com a 
> do enunciado, é suficiente para provar a linearidade de f.
>

Seriosamente, não me parece útil perder tempo provando que isso é
linear. O processo que você levaria provando que f(x)=Ax+B basicamente
se resumiria a finalizar o problema.

Outra identidade que pode ser útil para você é provar que f(f(x)) -
f(x) = f(x) -x.
Essa, junto com a identidade acima que você provou, te deixam em 70%
do problema.

> --
> Esta mensagem foi verificada pelo sistema de antivírus e
> acredita-se estar livre de perigo.

-- 
Esta mensagem foi verificada pelo sistema de antiv�rus e
 acredita-se estar livre de perigo.


=
Instru��es para entrar na lista, sair da lista e usar a lista em
http://www.mat.puc-rio.br/~obmlistas/obm-l.html
=


[obm-l] Re: [obm-l] Re: [obm-l] Re: [obm-l] Re: [obm-l] Re: [obm-l] Re: [obm-l] Re: Equações funcionais

2021-02-16 Por tôpico joao pedro b menezes
Eu sei, temos f(-1)= 0, f(0) = 1, e f é bijetora. Após trabalhar a equação
que cheguei na expressão:
f( x + f(x) ) - f( f(x)) = x.  Queria saber se essa identidade, junto com a
do enunciado, é suficiente para provar a linearidade de f.

-- 
Esta mensagem foi verificada pelo sistema de antiv�rus e
 acredita-se estar livre de perigo.



[obm-l] Re: [obm-l] Re: [obm-l] Re: [obm-l] Re: [obm-l] Re: [obm-l] Re: Equações funcionais

2021-02-16 Por tôpico Anderson Torres
Em ter., 16 de fev. de 2021 às 20:43, joao pedro b menezes
 escreveu:
>
> Foi da OBM 2006, nível 3,  3° fase:
> “Determine todas as funções f: R -> R tais que
> f( xf(y) + f(x) ) = 2f(x) + xy

Isso dá bem mais informação!

Por exemplo essa função é sobrejetora. Afinal, qualquer número pode
ser escrito na forma 2f(x)+xy - faça por exemplo x=1 e y=z-2f(1).

Daí a ideia é resolver as equacoes f(A)=0 e f(B)=1.

> para todos x,y reais”
>
> --
> Esta mensagem foi verificada pelo sistema de antivírus e
> acredita-se estar livre de perigo.

-- 
Esta mensagem foi verificada pelo sistema de antiv�rus e
 acredita-se estar livre de perigo.


=
Instru��es para entrar na lista, sair da lista e usar a lista em
http://www.mat.puc-rio.br/~obmlistas/obm-l.html
=


[obm-l] Re: [obm-l] Re: [obm-l] Re: [obm-l] Re: [obm-l] Sequência Injetiva

2021-02-16 Por tôpico Anderson Torres
Nada como uma bijeção N -> Q para encerrar o dia!

Se pensar nas operacoes INC e REV, podemos usar um algoritmo assim:

- Se o número é maior que 1, usa DEC (inversa de INC)
- Se o número é menor que 1, usa INV
- Se o número é 1, pare

Como demonstrar que este procedimento sempre encerrará em 1, não
importando que número racional começou? Acho que no fundo isso é só
uma maneira de encodar fracoes continuas mesmo.

Em ter., 16 de fev. de 2021 às 20:35, Matheus Secco
 escreveu:
>
> Esse problema caiu na Olimpíada Iberoamericana de 2009 que eu participei. Foi 
> o problema 5 da prova e lá pedia para provar injetividade e sobrejetividade.
>
> Em qua, 17 de fev de 2021 00:16, Anderson Torres 
>  escreveu:
>>
>> Em dom., 14 de fev. de 2021 às 17:20, Claudio Buffara
>>  escreveu:
>> >
>> > Será que essa sequência é sobrejetiva (sobre os racionais positivos)?
>> > Porque como a(2^n) = n+1, ela certamente atinge todos os naturais, de modo 
>> > que é ilimitada, superiormente e inferiormente (já que a(2^n + 1) = 
>> > 1/(n+1) ).
>> > Mesmo que não seja, seria interessante descobrir que racionais positivos 
>> > ela não atinge.
>> > É suficiente provar que todos os racionais entre 0 e 1 são atingidos (no 
>> > caso, pelos termos de ordem ímpar), mas não sei se isso facilita.
>> > Vale uma exploração numérica, talvez com uma planilha.
>>
>>
>> Se eu não errei as contas, acredito que sim. Afinal basta reverter a
>> fracao continua.
>>
>> As operacoes parecem ser bem limitadas, contudo nao e necessario muito
>> mais que isso para gerar um racional qualquer:
>>
>> - Função INC: x -> x+1
>> - Função REV: x -> 1/x
>>
>> Talvez haja algum invariante que permita prever que cada operacao esta
>> fadada a cair em 1
>>
>> >
>> >
>> > Abs,
>> > Claudio.
>> >
>> > Enviado do meu iPhone
>> >
>> > Em 14 de fev. de 2021, à(s) 13:57, Anderson Torres 
>> >  escreveu:
>> >
>> > 
>> >
>> >
>> > Em sáb., 13 de fev. de 2021 às 17:56, Jeferson Almir 
>> >  escreveu:
>> >>
>> >> Amigos, peço ajuda em provar a injetividade dessa sequência que seria 
>> >> uma saída para provar a unica ocorrência do racional que aparece nela. 
>> >> Estou andando em círculos tentando montar uma possível indução.
>> >>
>> >>
>> >> Dado a sequência a_1 = 1 e a_2n = a_n  + 1 e a_2n+1 = 1/a_2n.
>> >>
>> >> Prove que para todo racional positivo que ocorre na sequência, ocorre 
>> >> uma única vez.
>> >>
>> >>
>> >
>> > Acho que e uma boa usar fracao continua aqui.
>> >
>> > Se a_n = [c0; c1, c2, ..., ck], temos entao a_1 = [1] e
>> >
>> > a_2n =Â [(1+c0); c1, c2, ..., ck] (chamemos isso de operacao E)
>> > a_2n+1 = [0; (1+c0), c1, c2, ..., ck] (chamemos isso de operacao O)
>> >
>> >
>> > A partir disso, acredito que a bijecao fica quase obvia, bastando 
>> > formalizar algumas inducoes marotas.
>> >
>> > Primeiramente, nenhuma representacao da forma [...,N,1] vai surgir dai a 
>> > partir de a_2. Isso pode ser demonstrado por inducao mesmo: ck=1 somente 
>> > no caso [1], e depois dele a funcao a_n so modifica o comeco da cadeia, 
>> > nunca o final dela.
>> >
>> > Assim sendo, temos certeza que nao tem como um racional aparecer uma vez 
>> > na forma canonica e outra na forma alternativa. E, por conseguinte, se 
>> > duas fracoes tem comprimentos diferentes, elas devem ser diferentes. E 
>> > fracoes com comprimentos iguais diferem se e somente se pelo menos um dos 
>> > componentes diferir.
>> >
>> > Agora, a funcao recursiva age de duas formas. Uma delas altera o 
>> > comprimento em 1, e a outra mantém. A que altera, só altera 
>> > acrescentando o 0 na cabeceira. A que não altera, incrementa a cabeceira.
>> >
>> > Desta forma, é possível gerar de maneira unica qualquer numero racional 
>> > comecando do 1.
>> >
>> > - Qualquer fracao de comprimento 1 pode ser gerada simplesmente aplicando 
>> > a operacao E tantas vezes quantas forem necessarias. E tambem nao e 
>> > possivel fazer isso de outra maneira, pois a operacao O aumentara o 
>> > comprimento de maneira irreversivel.
>> >
>> > - Dada uma fracao com comprimento K, temos duas sub inducoes para fazer:
>> >
>> > + A fracao tem comprimento K e comeca com 0.
>> >
>> > Â  Entao ela foi gerada por uma operacao O. O elemento que a gerou tinha 
>> > menos componentes, os quais satisfazem a hipotese de inducao.
>> >
>> > + A fracao tem comprimento K e comeca com algo maior que 0.
>> >
>> > Entao ela foi gerada por uma operacao E. A fracao da qual ela foi gerada 
>> > difere unicamente no primeiro elemento, o qual antes era menor. Assim 
>> > sendo, e possivel reduzir isso ate chegar no caso anterior.
>> >
>> > E isso demonstra recursivamente a unicidade e existencia!
>> >
>> >
>> >
>> >> --
>> >> Esta mensagem foi verificada pelo sistema de antivírus e
>> >> acredita-se estar livre de perigo.
>> >
>> >
>> > --
>> > Esta mensagem foi verificada pelo sistema de antivírus e
>> > acredita-se estar livre de perigo.
>> >
>> >
>> > --
>> > Esta mensagem foi verificada pelo sistema de 

[obm-l] Re: [obm-l] Re: [obm-l] Re: [obm-l] Re: [obm-l] Re: Equações funcionais

2021-02-16 Por tôpico joao pedro b menezes
Foi da OBM 2006, nível 3,  3° fase:
“Determine todas as funções f: R -> R tais que
f( xf(y) + f(x) ) = 2f(x) + xy
para todos x,y reais”

-- 
Esta mensagem foi verificada pelo sistema de antiv�rus e
 acredita-se estar livre de perigo.



[obm-l] Re: [obm-l] Re: [obm-l] Re: [obm-l] Sequência Injetiva

2021-02-16 Por tôpico Matheus Secco
Esse problema caiu na Olimpíada Iberoamericana de 2009 que eu participei.
Foi o problema 5 da prova e lá pedia para provar injetividade e
sobrejetividade.

Em qua, 17 de fev de 2021 00:16, Anderson Torres <
torres.anderson...@gmail.com> escreveu:

> Em dom., 14 de fev. de 2021 às 17:20, Claudio Buffara
>  escreveu:
> >
> > Será que essa sequência é sobrejetiva (sobre os racionais positivos)?
> > Porque como a(2^n) = n+1, ela certamente atinge todos os naturais, de
> modo que é ilimitada, superiormente e inferiormente (já que a(2^n + 1) =
> 1/(n+1) ).
> > Mesmo que não seja, seria interessante descobrir que racionais positivos
> ela não atinge.
> > É suficiente provar que todos os racionais entre 0 e 1 são atingidos (no
> caso, pelos termos de ordem ímpar), mas não sei se isso facilita.
> > Vale uma exploração numérica, talvez com uma planilha.
>
>
> Se eu não errei as contas, acredito que sim. Afinal basta reverter a
> fracao continua.
>
> As operacoes parecem ser bem limitadas, contudo nao e necessario muito
> mais que isso para gerar um racional qualquer:
>
> - Função INC: x -> x+1
> - Função REV: x -> 1/x
>
> Talvez haja algum invariante que permita prever que cada operacao esta
> fadada a cair em 1
>
> >
> >
> > Abs,
> > Claudio.
> >
> > Enviado do meu iPhone
> >
> > Em 14 de fev. de 2021, à(s) 13:57, Anderson Torres <
> torres.anderson...@gmail.com> escreveu:
> >
> > 
> >
> >
> > Em sáb., 13 de fev. de 2021 às 17:56, Jeferson Almir <
> jefersonram...@gmail.com> escreveu:
> >>
> >> Amigos, peço ajuda em provar a injetividade dessa sequência que seria
> uma saída para provar a unica ocorrência do racional que aparece nela.
> Estou andando em círculos tentando montar uma possível indução.
> >>
> >>
> >> Dado a sequência a_1 = 1 e a_2n = a_n  + 1 e a_2n+1 = 1/a_2n.
> >>
> >> Prove que para todo racional positivo que ocorre na sequência, ocorre
> uma única vez.
> >>
> >>
> >
> > Acho que e uma boa usar fracao continua aqui.
> >
> > Se a_n = [c0; c1, c2, ..., ck], temos entao a_1 = [1] e
> >
> > a_2n =Â [(1+c0); c1, c2, ..., ck] (chamemos isso de operacao E)
> > a_2n+1 = [0; (1+c0), c1, c2, ..., ck] (chamemos isso de operacao O)
> >
> >
> > A partir disso, acredito que a bijecao fica quase obvia, bastando
> formalizar algumas inducoes marotas.
> >
> > Primeiramente, nenhuma representacao da forma [...,N,1] vai surgir dai a
> partir de a_2. Isso pode ser demonstrado por inducao mesmo: ck=1 somente
> no caso [1], e depois dele a funcao a_n so modifica o comeco da cadeia,
> nunca o final dela.
> >
> > Assim sendo, temos certeza que nao tem como um racional aparecer uma vez
> na forma canonica e outra na forma alternativa. E, por conseguinte, se duas
> fracoes tem comprimentos diferentes, elas devem ser diferentes. E fracoes
> com comprimentos iguais diferem se e somente se pelo menos um dos
> componentes diferir.
> >
> > Agora, a funcao recursiva age de duas formas. Uma delas altera o
> comprimento em 1, e a outra mantém. A que altera, só altera acrescentando
> o 0 na cabeceira. A que não altera, incrementa a cabeceira.
> >
> > Desta forma, é possível gerar de maneira unica qualquer numeroÂ
> racional comecando do 1.
> >
> > - Qualquer fracao de comprimento 1 pode ser gerada simplesmente
> aplicando a operacao E tantas vezes quantas forem necessarias. E tambem
> nao e possivel fazer isso de outra maneira, pois a operacao O aumentara o
> comprimento de maneira irreversivel.
> >
> > - Dada uma fracao com comprimento K, temos duas sub inducoes para fazer:
> >
> > + A fracao tem comprimento K e comeca com 0.
> >
> > Â  Entao ela foi gerada por uma operacao O. O elemento que a gerou tinha
> menos componentes, os quais satisfazem a hipotese de inducao.
> >
> > + A fracao tem comprimento K e comeca com algo maior que 0.
> >
> > Entao ela foi gerada por uma operacao E. A fracao da qual ela foi gerada
> difere unicamente no primeiro elemento, o qual antes era menor. Assim
> sendo, e possivel reduzir isso ate chegar no caso anterior.
> >
> > E isso demonstra recursivamente a unicidade e existencia!
> >
> >
> >
> >> --
> >> Esta mensagem foi verificada pelo sistema de antivírus e
> >> acredita-se estar livre de perigo.
> >
> >
> > --
> > Esta mensagem foi verificada pelo sistema de antivírus e
> > acredita-se estar livre de perigo.
> >
> >
> > --
> > Esta mensagem foi verificada pelo sistema de antivírus e
> > acredita-se estar livre de perigo.
>
> --
> Esta mensagem foi verificada pelo sistema de antivírus e
>  acredita-se estar livre de perigo.
>
>
> =
> Instru�ões para entrar na lista, sair da lista e usar a lista em
> http://www.mat.puc-rio.br/~obmlistas/obm-l.html
> =
>

-- 
Esta mensagem foi verificada pelo sistema de antiv�rus e
 acredita-se estar livre de perigo.



[obm-l] Re: [obm-l] Re: [obm-l] Re: [obm-l] Re: Equações funcionais

2021-02-16 Por tôpico Anderson Torres
Eu gostaria de saber da origem desse problema...

Em dom., 14 de fev. de 2021 às 14:32, joao pedro b menezes <
joaopedrobmene...@gmail.com> escreveu:

> Obrigado pela resposta, mas ainda tenho umas dúvidas. Poderia dar um
> exemplo de tal função ou explicar como construí-la? E se f fosse somente
> injetora, mudaria alguma coisa?
>
>
> --
> Esta mensagem foi verificada pelo sistema de antivírus e
> acredita-se estar livre de perigo.

-- 
Esta mensagem foi verificada pelo sistema de antiv�rus e
 acredita-se estar livre de perigo.



[obm-l] Re: [obm-l] Re: [obm-l] Sequência Injetiva

2021-02-16 Por tôpico Anderson Torres
Em dom., 14 de fev. de 2021 às 17:20, Claudio Buffara
 escreveu:
>
> Será que essa sequência é sobrejetiva (sobre os racionais positivos)?
> Porque como a(2^n) = n+1, ela certamente atinge todos os naturais, de modo 
> que é ilimitada, superiormente e inferiormente (já que a(2^n + 1) = 1/(n+1) ).
> Mesmo que não seja, seria interessante descobrir que racionais positivos ela 
> não atinge.
> É suficiente provar que todos os racionais entre 0 e 1 são atingidos (no 
> caso, pelos termos de ordem ímpar), mas não sei se isso facilita.
> Vale uma exploração numérica, talvez com uma planilha.


Se eu não errei as contas, acredito que sim. Afinal basta reverter a
fracao continua.

As operacoes parecem ser bem limitadas, contudo nao e necessario muito
mais que isso para gerar um racional qualquer:

- Função INC: x -> x+1
- Função REV: x -> 1/x

Talvez haja algum invariante que permita prever que cada operacao esta
fadada a cair em 1

>
>
> Abs,
> Claudio.
>
> Enviado do meu iPhone
>
> Em 14 de fev. de 2021, à(s) 13:57, Anderson Torres 
>  escreveu:
>
> 
>
>
> Em sáb., 13 de fev. de 2021 às 17:56, Jeferson Almir 
>  escreveu:
>>
>> Amigos, peço ajuda em provar a injetividade dessa sequência que seria uma 
>> saída para provar a unica ocorrência do racional que aparece nela. Estou 
>> andando em círculos tentando montar uma possível indução.
>>
>>
>> Dado a sequência a_1 = 1 e a_2n = a_n  + 1 e a_2n+1 = 1/a_2n.
>>
>> Prove que para todo racional positivo que ocorre na sequência, ocorre uma 
>> única vez.
>>
>>
>
> Acho que e uma boa usar fracao continua aqui.
>
> Se a_n = [c0; c1, c2, ..., ck], temos entao a_1 = [1] e
>
> a_2n =Â [(1+c0); c1, c2, ..., ck] (chamemos isso de operacao E)
> a_2n+1 = [0; (1+c0), c1, c2, ..., ck] (chamemos isso de operacao O)
>
>
> A partir disso, acredito que a bijecao fica quase obvia, bastando formalizar 
> algumas inducoes marotas.
>
> Primeiramente, nenhuma representacao da forma [...,N,1] vai surgir dai a 
> partir de a_2. Isso pode ser demonstrado por inducao mesmo: ck=1 somente no 
> caso [1], e depois dele a funcao a_n so modifica o comeco da cadeia, nunca o 
> final dela.
>
> Assim sendo, temos certeza que nao tem como um racional aparecer uma vez na 
> forma canonica e outra na forma alternativa. E, por conseguinte, se duas 
> fracoes tem comprimentos diferentes, elas devem ser diferentes. E fracoes com 
> comprimentos iguais diferem se e somente se pelo menos um dos componentes 
> diferir.
>
> Agora, a funcao recursiva age de duas formas. Uma delas altera o comprimento 
> em 1, e a outra mantém. A que altera, só altera acrescentando o 0 na 
> cabeceira. A que não altera, incrementa a cabeceira.
>
> Desta forma, é possível gerar de maneira unica qualquer numero racional 
> comecando do 1.
>
> - Qualquer fracao de comprimento 1 pode ser gerada simplesmente aplicando a 
> operacao E tantas vezes quantas forem necessarias. E tambem nao e possivel 
> fazer isso de outra maneira, pois a operacao O aumentara o comprimento de 
> maneira irreversivel.
>
> - Dada uma fracao com comprimento K, temos duas sub inducoes para fazer:
>
> + A fracao tem comprimento K e comeca com 0.
>
> Â  Entao ela foi gerada por uma operacao O. O elemento que a gerou tinha 
> menos componentes, os quais satisfazem a hipotese de inducao.
>
> + A fracao tem comprimento K e comeca com algo maior que 0.
>
> Entao ela foi gerada por uma operacao E. A fracao da qual ela foi gerada 
> difere unicamente no primeiro elemento, o qual antes era menor. Assim sendo, 
> e possivel reduzir isso ate chegar no caso anterior.
>
> E isso demonstra recursivamente a unicidade e existencia!
>
>
>
>> --
>> Esta mensagem foi verificada pelo sistema de antivírus e
>> acredita-se estar livre de perigo.
>
>
> --
> Esta mensagem foi verificada pelo sistema de antivírus e
> acredita-se estar livre de perigo.
>
>
> --
> Esta mensagem foi verificada pelo sistema de antivírus e
> acredita-se estar livre de perigo.

-- 
Esta mensagem foi verificada pelo sistema de antiv�rus e
 acredita-se estar livre de perigo.


=
Instru��es para entrar na lista, sair da lista e usar a lista em
http://www.mat.puc-rio.br/~obmlistas/obm-l.html
=


[obm-l] Re: [obm-l] Somatório

2021-02-16 Por tôpico Artur Costa Steiner
Como a função x ---> 1/x3 , x > 0, é postiva e estritamente decrescente,
para todo inteiro positivo n temos que

Soma(1, n) 1/k^3 = 1 + Soma(2, n) 1/k^3 <  1 + Integral (2,n) 1/x^3 dx < 1
+ Integral (2, oo) 1/x^3 dx = 1 + [-1/(2x^2)] [2, oo) = 1 + 1/1/8 = 9/8 <
10/8 = 5/4


Em ter., 16 de fev. de 2021 07:23,  escreveu:

> Seja n um inteiro positivo. Prove que:
>
> Somatório(1/k^3)<5/4 , k=1 até n
> --
> Esta mensagem foi verificada pelo sistema de antivírus e
>  acredita-se estar livre de perigo.
>
>
> =
> Instruções para entrar na lista, sair da lista e usar a lista em
> http://www.mat.puc-rio.br/~obmlistas/obm-l.html
> =
>

-- 
Esta mensagem foi verificada pelo sistema de antiv�rus e
 acredita-se estar livre de perigo.



Re: [obm-l] Re: [obm-l] Sequência Injetiva

2021-02-14 Por tôpico Claudio Buffara
Será que essa sequência é sobrejetiva (sobre os racionais positivos)?
Porque como a(2^n) = n+1, ela certamente atinge todos os naturais, de modo que 
é ilimitada, superiormente e inferiormente (já que a(2^n + 1) = 1/(n+1) ).
Mesmo que não seja, seria interessante descobrir que racionais positivos ela 
não atinge.
É suficiente provar que todos os racionais entre 0 e 1 são atingidos (no caso, 
pelos termos de ordem ímpar), mas não sei se isso facilita.
Vale uma exploração numérica, talvez com uma planilha.

Abs,
Claudio.

Enviado do meu iPhone

> Em 14 de fev. de 2021, à(s) 13:57, Anderson Torres 
>  escreveu:
> 
> 
> 
> 
> Em sáb., 13 de fev. de 2021 às 17:56, Jeferson Almir 
>  escreveu:
>> Amigos, peço ajuda em provar a injetividade dessa sequência que seria uma 
>> saída para provar a unica ocorrência do racional que aparece nela. Estou 
>> andando em círculos tentando montar uma possível indução.
>> 
>> Dado a sequência a_1 = 1 e a_2n = a_n  + 1 e a_2n+1 = 1/a_2n.
>> Prove que para todo racional positivo que ocorre na sequência, ocorre uma 
>> única vez.
>> 
> 
> Acho que e uma boa usar fracao continua aqui.
> 
> Se a_n = [c0; c1, c2, ..., ck], temos entao a_1 = [1] e
> 
> a_2n =Â [(1+c0); c1, c2, ..., ck] (chamemos isso de operacao E)
> a_2n+1 = [0; (1+c0), c1, c2, ..., ck] (chamemos isso de operacao O)
> 
> 
> A partir disso, acredito que a bijecao fica quase obvia, bastando formalizar 
> algumas inducoes marotas.
> 
> Primeiramente, nenhuma representacao da forma [...,N,1] vai surgir dai a 
> partir de a_2. Isso pode ser demonstrado por inducao mesmo: ck=1 somente no 
> caso [1], e depois dele a funcao a_n so modifica o comeco da cadeia, nunca o 
> final dela.
> 
> Assim sendo, temos certeza que nao tem como um racional aparecer uma vez na 
> forma canonica e outra na forma alternativa. E, por conseguinte, se duas 
> fracoes tem comprimentos diferentes, elas devem ser diferentes. E fracoes com 
> comprimentos iguais diferem se e somente se pelo menos um dos componentes 
> diferir.
> 
> Agora, a funcao recursiva age de duas formas. Uma delas altera o comprimento 
> em 1, e a outra mantém. A que altera, só altera acrescentando o 0 na 
> cabeceira. A que não altera, incrementa a cabeceira.
> 
> Desta forma, é possível gerar de maneira unica qualquer numero racional 
> comecando do 1.
> 
> - Qualquer fracao de comprimento 1 pode ser gerada simplesmente aplicando a 
> operacao E tantas vezes quantas forem necessarias. E tambem nao e possivel 
> fazer isso de outra maneira, pois a operacao O aumentara o comprimento de 
> maneira irreversivel.
> 
> - Dada uma fracao com comprimento K, temos duas sub inducoes para fazer:
> 
> + A fracao tem comprimento K e comeca com 0.
> 
> Â  Entao ela foi gerada por uma operacao O. O elemento que a gerou tinha 
> menos componentes, os quais satisfazem a hipotese de inducao.
> 
> + A fracao tem comprimento K e comeca com algo maior que 0.
> 
> Entao ela foi gerada por uma operacao E. A fracao da qual ela foi gerada 
> difere unicamente no primeiro elemento, o qual antes era menor. Assim sendo, 
> e possivel reduzir isso ate chegar no caso anterior.
> 
> E isso demonstra recursivamente a unicidade e existencia!
> 
> 
> 
>> -- 
>> Esta mensagem foi verificada pelo sistema de antivírus e 
>> acredita-se estar livre de perigo.
> 
> -- 
> Esta mensagem foi verificada pelo sistema de antivírus e 
> acredita-se estar livre de perigo.

-- 
Esta mensagem foi verificada pelo sistema de antiv�rus e
 acredita-se estar livre de perigo.



[obm-l] Re: [obm-l] Re: [obm-l] Re: Equações funcionais

2021-02-14 Por tôpico joao pedro b menezes
Obrigado pela resposta, mas ainda tenho umas dúvidas. Poderia dar um
exemplo de tal função ou explicar como construí-la? E se f fosse somente
injetora, mudaria alguma coisa?

-- 
Esta mensagem foi verificada pelo sistema de antiv�rus e
 acredita-se estar livre de perigo.



[obm-l] Re: [obm-l] Re: Equações funcionais

2021-02-14 Por tôpico Anderson Torres
Em dom., 14 de fev. de 2021 às 11:30, joao pedro b menezes <
joaopedrobmene...@gmail.com> escreveu:

> Obs: f é bijetora
>
>>
>
Acho que nao basta. Se f(x)=y entao f(x+y)=x+f(y).

Com isso, poderiamos fazer uma funcao que nao aja linearmente em (0,1) mas
aja linearmente fora dele.



> --
> Esta mensagem foi verificada pelo sistema de antivírus e
> acredita-se estar livre de perigo.

-- 
Esta mensagem foi verificada pelo sistema de antiv�rus e
 acredita-se estar livre de perigo.



[obm-l] Re: [obm-l] Sequência Injetiva

2021-02-14 Por tôpico Anderson Torres
Em sáb., 13 de fev. de 2021 às 17:56, Jeferson Almir <
jefersonram...@gmail.com> escreveu:

> Amigos, peço ajuda em provar a injetividade dessa sequência que seria uma
> saída para provar a unica ocorrência do racional que aparece nela. Estou
> andando em círculos tentando montar uma possível indução.
>
>
> Dado a sequência a_1 = 1 e a_2n = a_n  + 1 e a_2n+1 = 1/a_2n.
>
> Prove que para todo racional positivo que ocorre na sequência, ocorre uma
> única vez.
>
>
Acho que e uma boa usar fracao continua aqui.

Se a_n = [c0; c1, c2, ..., ck], temos entao a_1 = [1] e

a_2n = [(1+c0); c1, c2, ..., ck] (chamemos isso de operacao E)
a_2n+1 = [0; (1+c0), c1, c2, ..., ck] (chamemos isso de operacao O)


A partir disso, acredito que a bijecao fica quase obvia, bastando
formalizar algumas inducoes marotas.

Primeiramente, nenhuma representacao da forma [...,N,1] vai surgir dai a
partir de a_2. Isso pode ser demonstrado por inducao mesmo: ck=1 somente no
caso [1], e depois dele a funcao a_n so modifica o comeco da cadeia, nunca
o final dela.

Assim sendo, temos certeza que nao tem como um racional aparecer uma vez na
forma canonica e outra na forma alternativa. E, por conseguinte, se duas
fracoes tem comprimentos diferentes, elas devem ser diferentes. E fracoes
com comprimentos iguais diferem se e somente se pelo menos um dos
componentes diferir.

Agora, a funcao recursiva age de duas formas. Uma delas altera o
comprimento em 1, e a outra mantém. A que altera, só altera acrescentando o
0 na cabeceira. A que não altera, incrementa a cabeceira.

Desta forma, é possível gerar de maneira unica qualquer numero racional
comecando do 1.

- Qualquer fracao de comprimento 1 pode ser gerada simplesmente aplicando a
operacao E tantas vezes quantas forem necessarias. E tambem nao e possivel
fazer isso de outra maneira, pois a operacao O aumentara o comprimento de
maneira irreversivel.

- Dada uma fracao com comprimento K, temos duas sub inducoes para fazer:

+ A fracao tem comprimento K e comeca com 0.

  Entao ela foi gerada por uma operacao O. O elemento que a gerou tinha
menos componentes, os quais satisfazem a hipotese de inducao.

+ A fracao tem comprimento K e comeca com algo maior que 0.

Entao ela foi gerada por uma operacao E. A fracao da qual ela foi gerada
difere unicamente no primeiro elemento, o qual antes era menor. Assim
sendo, e possivel reduzir isso ate chegar no caso anterior.

E isso demonstra recursivamente a unicidade e existencia!



-- 
> Esta mensagem foi verificada pelo sistema de antivírus e
> acredita-se estar livre de perigo.

-- 
Esta mensagem foi verificada pelo sistema de antiv�rus e
 acredita-se estar livre de perigo.



<    1   2   3   4   5   6   7   8   9   10   >